You are on page 1of 130

www.cgaspirants.

com

UNIT 1
Function of Complex Variable

1.1 INTRODUCTION

m
A complex number z is an ordered pair (x, y) of real numbers and is written as

o
z = x + iy, where i = − 1 .

.c
The real numbers x and y are called the real and Y
imaginary parts of z. In the Argand’s diagram, the complex
P (x, y)

polar coordinates of P, then r =


n
y
ts
number z is represented by the point P(x, y). If (r, θ) are the
x 2 + y 2 is called the modulus
r y
ra
of z and is denoted by | z |. Also θ = tan–1 is called the
x
argument of z and is denoted by arg. z. Every non-zero complex θ
number z can be expressed as
pi

O x M X
z = r (cos θ + i sin θ) = reiθ
If z = x + iy, then the complex number x – iy is called the conjugate of the complex
as

number z and is denoted by z .


Clearly, | z | = | z |,| z |2 = z z ,
g

z+z z−z
.c

Re(z) = , Im(z) = .
2 2i
w

1.2 DEFINITIONS
w

Let S be a non-empty set of complex numbers and δ be a positive real number.


1. Circle. |z – a| = r represents a circle C with centre at the point a and radius r.
w

2. Open disk. The set of points which satisfies the equation |z – z0| < δ defines an open disk
of radius δ with centre at z0 = (x0, y0). This set consists of all points which lie inside circle C.
3. Closed disk. The set of points which satisfies the equation |z – z0| ≤ δ defines a closed disk
of radius δ with centre at z0 = (x0, y0). This set consists of all points which lie inside and on the
boundary of circle C.
4. Annulus. The set of points which lie between two concentric circles C1 : |z – a| = r1 and
C2 : |z – a| = r2 defines an open annulus i.e., the set of points which satisfies the inequality r1
< |z – a| < r2.
The set of points which satisfies the inequality r1 ≤ |z – a| ≤ r2 defines a closed annulus.
It is to be noted that r1 ≤ |z – a| < r2 is neither open nor closed.

www.cgaspirants.com
www.cgaspirants.com

2 A TEXTBOOK OF ENGINEERING MATHEMATICS

5. Neighbourhood. δ-Neighbourhood of a point z0 is the set of all points z for which


|z – z0| < δ where δ is a positive constant. If we exclude the point z0 from the open disk
|z – z0| < δ then it is called the deleted neighbourhood of the point z0 and is written as
0 < |z – z0| < δ.
6. Interior and exterior points. A point z is an interior point of S if all the points in some
δ-neighbourhood of z are in S and an exterior point of S if they are outside S.
7. Boundary point. A point z is a boundary point of S if every δ-neighbourhood of z contains
at least one point of S and at least one point not in S. For example, the points on the circle
|z – z0| = r are the boundary points for the disk |z – z0| ≤ r.
8. Open and closed sets. A set S is open if every point of S is an interior point while a set S
is closed if every boundary point of S belongs to S. e.g. S = {z : |z – z0| < r} is open set while
S = {z : |z – z0| ≤ r} is closed set.

m
9. Bounded set. An open set S is bounded if ∃ a positive real number M such that
| z | ≤ M for all z ∈ S otherwise unbounded.

o
For example: the set S = {z : |z – z0| < r} is a bounded set while the set S = {z : |z – z0| > r} is

.c
an unbounded set.
10. Connected set. An open set S is connected if any two points z1 and z2 belonging to S can

ts
be joined by a polygonal line which is totally contained in S.
11. Domain. An open connected set is called a domain denoted by D.
n
12. Region. A region is a domain together with all, some or none of its boundary points. Thus
ra
a domain is always a region but a region may or may not be a domain.
13. Finite complex plane. The complex plane without the point z = ∞ is called the finite
complex plane.
pi

14. Extended complex plane. The complex plane to which the point z = ∞ has been added is
called the extended complex plane.
as

1.3 FUNCTION OF A COMPLEX VARIABLE


g

If x and y are real variables, then z = x + iy is called a complex variable. If corresponding to


.c

each value of a complex variable z(= x + iy) in a given region R, there correspond one or more
values of another complex variable w (= u + iv), then w is called a function of the complex
w

variable z and is denoted by


w = f(z) = u + iv
w

For example, if w = z2 where z = x + iy and w = f(z) = u + iv


then u + iv = (x + iy) = (x2 – y2) + i(2xy)
2
w

⇒ u = x2 – y2 and v = 2xy
Thus u and v, the real and imaginary parts of w, are functions of the real variables x and y.
∴ w = f(z) = u(x, y) + iv(x, y)
If to each value of z, there corresponds one and only one value of w, then w is called a
single-valued function of z. If to each value of z, there correspond more than one values of w,
then w is called a multi-valued function of z. For example, w = z is a multi-valued function.
To represent w = f(z) graphically, we take two Argand diagrams: one to represent the
point z and the other to represent w. The former diagram is called the XOY-plane or the
z-plane and the latter UOV-plane or the w-plane.

www.cgaspirants.com
www.cgaspirants.com

FUNCTION OF COMPLEX VARIABLE 3

1.4 LIMIT OF f(z)

A function f(z) tends to the limit l as z tends to z0 along any Y


path, if to each positive arbitrary number ε, however small, Z
there corresponds a positive number δ, such that
| f(z) – l | < ε whenever 0 < | z– z0 | < δ
and we write Lt f(z) = l, where l is finite
z → z0
Z0
Note. In real variables, x → x0 implies that x approaches x0 along
the number line, either from left or from right. In complex
variables, z → z0 implies that z approaches z0 along any path,
straight or curved, since the two points representing z and z0 in a

m
O X
complex plane can be joined by an infinite number of curves.

1.5 CONTINUITY OF f(z)

o
.c
A single-valued function f(z) is said to be continuous at a point z = z0 if f(z0) exists, lim f(z)
z → z0

exists and Lt f(z) = f(z0).


z → z0
n ts
A function f(z) is said to be continuous in a region R of the z-plane if it is continuous at
every point of the region. A function f(z) which is not continuous at z0 is said to be discontinuous
ra

at z0.
If the function f(z) = u + iv is continuous at z0 = x0 + iy0 then the real functions u and v
are also continuous at the point (x0, y0). Therefore, we can discuss the continuity of a complex
pi

valued function by studying the continuity of its real and imaginary parts. If f(z) and g(z) are
f ( z)
as

continuous at a point z0 then the functions f(z) ± g(z), f(z) g(z) and , where g(z0) ≠ 0 are also
g( z)
continuous at z0.
g

If f(z) is continuous in a closed region S then it is bounded in S i.e., |f(z)| ≤ M ∀ z ∈ S.

FG IJ
.c

Also, the function f(z) is continuous at z = ∞ if the function f 1 is continuous at ξ = 0


ξ HK
w

1.6 DERIVATIVE OF f(z)


w

Let w = f(z) be a single-valued function of the variable z(= x + iy), then the derivative or
w

differential co-efficient of w = f(z) is defined as

dw f ( z + δz) − f ( z)
= f ′ ( z) = Lt
dz δz → 0 δz
provided the limit exists, independent of the manner in which δz → 0.

1.7 ANALYTIC FUNCTION [G.B.T.U. 2012, M.T.U. 2012, U.P.T.U. 2014]

A function f(z) is said to be analytic at a point z0 if it is one-valued and differentiable not only
at z0 but at every point of some neighbourhood of z0. For example: ex (cos y + i sin y). A function
f(z) is said to be analytic in a certain domain D if it is analytic at every point of D.

www.cgaspirants.com
www.cgaspirants.com

4 A TEXTBOOK OF ENGINEERING MATHEMATICS

The terms ‘regular’, ‘holomorphic’ and ‘monogenic’ are also sometimes used as synony-
mous with the term analytic.

A function f(z) is said to be analytic at z = ∞ if the function f FG 1IJ is analytic at z = 0.


H zK
Here it should be noted that analyticity implies differentiability but not vice versa. For
example, the function f(z) = |z|2 is differentiable only at z = 0 and nowhere else therefore f(z)
is differentiable at z = 0 but not analytic anywhere.
A function f(z) may be differentiable in a domain except for a finite number of points.
These points are called singular points or singularities of f(z) in that domain.

1.8 ENTIRE FUNCTION

m
A function f(z) which is analytic at every point of the finite complex plane is called an entire
function. Since the derivative of a polynomial exists at every point, a polynomial of any degree

o
is an entire function. Rational functions with non-zero denominators are also entire functions.

.c
1.9 NECESSARY AND SUFFICIENT CONDITIONS FOR f(z) TO BE ANALYTIC
n
The necessary and sufficient conditions for the function
w = f(z) = u(x, y) + iv(x, y)
ts
[M.T.U. 2012, U.P.T.U. (C.O.) 2008]
ra

to be analytic in a region R, are


∂u ∂u ∂v ∂v
(i) , , , are continuous functions of x and y in the region R.
pi

∂x ∂y ∂x ∂y
∂u ∂v ∂u ∂v
(ii) = , =− .
as

∂x ∂y ∂y ∂x
The conditions in (ii) are known as Cauchy-Riemann equations or briefly C-R
equations.
g

Proof. (a) Necessary Condition. Let w = f(z) = u(x, y) + iv(x, y) be analytic in a region R, then
.c

dw
= f ′(z) exists uniquely at every point of that region.
w

dz
Let δx and δy be the increments in x and y respectively. Let δu, δv and δz be the corre-
w

sponding increments in u, v and z respectively. Then,


f ( z + δz) − f ( z) (u + δu) + i(v + δv) − (u + iv)
w

f ′(z) = Lt = Lt
δz → 0 δz δz → 0 δz
FG δu + i δvIJ
= Lt
δz → 0 H δz δz K ...(1)

Since the function w = f(z) is analytic in the region R, the limit (1) must exist independ-
ent of the manner in which δz → 0, i.e., along whichever path δx and δy → 0.
First, let δz → 0 along a line parallel to x-axis so that δy = 0 and δz = δx.
[since z = x + iy, z + δz = (x + δx) + i(y + δy) and δz = δx + iδy]
FG δu + i δv IJ = ∂u + i ∂v
∴ From (1), f ′(z) = Lt
δx → 0 H δx δx K ∂x ∂x ...(2)

Now, let δz → 0 along a line parallel to y-axis so that δx = 0 and δz = iδy.

www.cgaspirants.com
www.cgaspirants.com

FUNCTION OF COMPLEX VARIABLE 5

f ′(z) = Lt
FG δu + i δv IJ = 1 ∂u + ∂v
∴ From (1),
δy → 0 H i δy i δy K i ∂y ∂y
∂v ∂u 1
= −i ...(3) ∵ =−i
∂y ∂y i
∂u ∂v ∂v ∂u
From (2) and (3), we have +i = −i
∂x ∂x ∂y ∂y

∂u ∂v ∂u ∂v
Equating the real and imaginary parts, = and =− (U.P.T.U. 2015)
∂x ∂y ∂y ∂x

Hence the necessary condition for f(z) to be analytic is that the C-R equations must be

m
satisfied.
(b) Sufficient Condition. Let f(z) = u + iv be a single-valued function possessing partial

o
∂u ∂u ∂v ∂v
derivatives , , , at each point of a region R and satisfying C-R equations.

.c
∂x ∂y ∂x ∂y
∂u ∂v ∂u ∂v
i.e., =
∂x ∂y
and
∂y
=−
∂x
.
ts
We shall show that f (z) is analytic, i.e., f ′(z) exists at every point of the region R.
n
By Taylor’s theorem for functions of two variables, we have, on omitting second and
ra

higher degree terms of δx and δy.


f(z + δz) = u(x + δx, y + δy) + iv(x + δx, y + δy)
LM FG ∂u δx + ∂u δyIJ OP + i LMv(x, y) + FG ∂v δx + ∂v δyIJ OP
pi

= u( x, y) +
N H ∂x ∂y K Q N H ∂x ∂ y K Q
as

= [u(x, y) + iv(x, y)] + G


F ∂u + i ∂vIJ δx + FG ∂u + i ∂vIJ δy
H ∂x ∂x K H ∂y ∂y K
g

F ∂u + i ∂vIJ δx + FG ∂u + i ∂vIJ δy
.c

= f(z) + G
H ∂x ∂x K H ∂y ∂y K
w

f(z + δz) – f(z) = G


F ∂u + i ∂vIJ δx + FG ∂u + i ∂vIJ δy
or
H ∂x ∂x K H ∂y ∂y K
w

=G
F ∂u + i ∂v IJ δx + FG − ∂v + i ∂u IJ δy | Using C-R equations
w

H ∂x ∂x K H ∂x ∂x K
F ∂u + i ∂v IJ δx + FG ∂u + i ∂vIJ iδy
= GH
∂x K H ∂x ∂x K |∵ –1=i 2
∂x

=G
F ∂u + i ∂v IJ (δx + iδy) = FG ∂u + i ∂v IJ δz | ∵ δx + iδy = δz
H ∂x ∂x K H ∂x ∂x K
f ( z + δz) − f ( z) ∂u ∂v
⇒ = +i
δz ∂x ∂x
f ( z + δz) − f ( z) ∂u ∂v
∴ f ′(z) = Lt = +i
δz → 0 δz ∂x ∂x

www.cgaspirants.com
www.cgaspirants.com

6 A TEXTBOOK OF ENGINEERING MATHEMATICS

∂u ∂v
Thus f ′(z) exists, because , exist.
∂x ∂x
Hence f(z) is analytic.
Note 1. The real and imaginary parts of an analytic function are called conjugate functions. Thus, if
f(z) = u(x, y) + iv (x, y) is an analytic function, then u(x, y) and v(x, y) are conjugate functions. The relation
between two conjugate functions is given by C-R equations.
Note 2. When a function f(z) is known to be analytic, it can be differentiated in the ordinary way as if z
is a real variable.
Thus, f(z) = z2 ⇒ f ′(z) = 2z
f(z) = sin z ⇒ f ′(z) = cos z etc.

1.10 CAUCHY-RIEMANN EQUATIONS IN POLAR COORDINATES (U.P.T.U. 2008)

m
Let (r, θ) be the polar coordinates of the point whose cartesian coordinates are (x, y), then

o
x = r cos θ, y = r sin θ,
z = x + iy = r (cos θ + i sin θ) = reiθ

.c
∴ u + iv = f(z) = f(reiθ) ...(1)
Differentiating (1) partially w.r.t. r, we have
∂u
∂r
+i
∂v
∂r
= f ′ (reiθ) . eiθ
n ts ...(2)
Differentiating (1) partially w.r.t. θ, we have
ra

∂u ∂v ∂u FG∂v IJ
∂θ
+i
∂θ
= f ′ (reiθ) . ireiθ = ir
∂r
+i
H∂r K | Using (2)
pi

∂v ∂u
=–r + ir
∂r ∂r
as

Equating real and imaginary parts, we get


∂u ∂v ∂v ∂u
=−r and =r
g

∂θ ∂r ∂θ ∂r
.c

∂u 1 ∂v ∂v 1 ∂u
or = and =− , which is the polar form of C-R equations.
∂r r ∂θ ∂r r ∂θ
w
w

1.11 DERIVATIVE OF w, i.e., f ′(z) IN POLAR COORDINATES

w = f(z)
w

dw ∂u ∂v ∂ ∂w
∴ = f ′ ( z) = +i = (u + iv) =
dz ∂x ∂x ∂x ∂x
∂w ∂r ∂w ∂θ
= +
∂r ∂x ∂θ ∂x
∵ r 2 = x 2 + y2
= cos θ
∂w FG
∂u IJ
∂v sin θ ∴ ∂r/∂x = cos θ as x = r cos θ
∂r

H
∂θ
+i
∂θ K
r
and θ = tan −1
yFG IJ
∂w F ∂v ∂u I sin θ x H K
= cos θ − G− r + ir J
∂r H ∂r K r
∂θ − sin θ
∂r ∴ = as y = r sin θ
∂x r

www.cgaspirants.com
www.cgaspirants.com

FUNCTION OF COMPLEX VARIABLE 7

= cos θ
∂w
−i
∂uFG
+i
∂v IJ sin θ = cos θ
∂w
– i sin θ
∂w
∂r ∂r H ∂r K ∂r ∂r

dw ∂w
⇒ = (cos θ – i sin θ) ...(1)
dz ∂r

∂w
which is the result in terms of .
∂r
dw ∂w ∂r ∂w ∂θ
. .
FG ∂u + i ∂v IJ cos θ − ∂w . sin θ
Again,
dz
= +
∂r ∂x ∂θ ∂x
=
H ∂ r ∂r K ∂θ r
F 1 ∂v − i ∂u IJ cos θ − sin θ ∂w = – i FG ∂u + i ∂v IJ cos θ − sin θ ∂w
= G

m
H r ∂θ r ∂θ K r ∂θ r H ∂θ ∂θ K r ∂θ

o
i ∂w sin θ ∂w
=– cos θ −
r ∂θ r ∂θ

.c
dw i ∂w
⇒ = − (cos θ – i sin θ)
dz r

which is the result in terms of


∂w
.
n ∂θ
ts
∂θ
ra

1.12 HARMONIC FUNCTION [M.T.U. 2014, G.B.T.U. 2012, U.P.T.U. 2007, 2009]
pi

A function of x, y which possesses continuous partial derivatives of the first and second orders
and satisfies Laplace’s equation is called a Harmonic function.
as

1.13 THEOREM
g

If f(z) = u + iv is an analytic function then u and v are both harmonic functions.


.c

Proof. Let f(z) = u + iv be analytic in some region of the z-plane, then u and v satisfy C-R
equations.
w

∂u ∂v
∴ = ...(1)
∂x ∂y
w

∂u ∂v
and =− ...(2)
w

∂y ∂x
Differentiating eqn. (1) partially w.r.t. x and eqn. (2) w.r.t. y, we get
∂ 2u ∂ 2v
= ...(3)
∂x 2 ∂x∂y

∂ 2u ∂ 2v
and = − ...(4)
∂y 2 ∂y∂x
2 2
Assuming ∂ v = ∂ v and adding equations (3) and (4), we get
∂x∂y ∂y∂x

www.cgaspirants.com
www.cgaspirants.com

8 A TEXTBOOK OF ENGINEERING MATHEMATICS

∂ 2u ∂ 2u
+ =0 ...(5)
∂x 2 ∂y 2
Now, differentiating eqn. (1) partially w.r.t. y and eqn. (2) w.r.t. x, we get
∂ 2u ∂ 2 v
= ...(6)
∂y∂x ∂y 2
∂ 2u ∂2v
and =− 2 ...(7)
∂x∂y ∂x
∂ 2u ∂ 2u
Assuming = and subtracting eqn. (7) from eqn. (6), we get
∂y∂x ∂x∂y

m
∂2v ∂2v
+ =0 ...(8)
∂x 2 ∂y 2

o
Equations (5) and (8) show that the real and imaginary parts u and v of an analytic
function satisfy the Laplace’s equation. Hence u and v are harmonic functions.

.c
Note. Here u and v are called conjugate harmonic functions.

1.14 ORTHOGONAL SYSTEM


n ts [M.T.U. 2012, U.P.T.U. 2009]

Every analytic function f(z) = u + iv defines two families of curves u(x, y) = c1 and v(x, y) = c2,
ra
which form an orthogonal system.
Consider the two families of curves
u(x, y) = c1 ...(1)
pi

and v(x, y) = c2 ...(2)


Differentiating eqn. (1) w.r.t. x, we get
as

∂u Y
∂u ∂u dy dy ∂x
+ . = 0 or =− = m1 (say)
g

∂x ∂y dx dx ∂u
.c

∂y v (x, y) = c2
∂v
w

dy
Similarly, from eqn. (2), we get = − x = m2 (say)

dx ∂v
w

∂y
∂u ∂v
w

. u (x, y) = c1
∂x ∂x
∴ m 1 m2 = ...(3)
∂u ∂v O
. X
∂y ∂y
Since f(z) is analytic, u and v satisfy C-R equations
∂u ∂v ∂u ∂v
i.e., = and =−
∂x ∂y ∂y ∂x
∂v ∂v
.
∂y ∂x
∴ From (3), m1 m2 = =–1
∂v ∂v
− .
∂x ∂y

www.cgaspirants.com
www.cgaspirants.com

FUNCTION OF COMPLEX VARIABLE 9

Thus, the product of the slopes of the curves (1) and (2) is –1. Hence the curves intersect
at right angles, i.e., they form an orthogonal system.

1.15 THEOREM (U.P.T.U. 2008)

An analytic function with constant modulus is constant.


Proof. Let f(z) = u + iv be an analytic function with constant modulus. Then,
| f(z) | = | u + iv | = constant
⇒ u 2 + v2 = constant = c (say)
Squaring both sides, we get
u 2 + v2 = c 2 ...(1)

m
Differentiating eqn. (1) partially w.r.t. x, we get
∂u ∂v

o
2u + 2v =0
∂x ∂x

.c
∂u ∂v
⇒ u +v =0 ...(2)
∂x ∂x
Again, differentiating eqn. (1) partially w.r.t. y, we get

2u
∂u
∂y
+ 2v
∂v
n
∂y
=0
ts
ra
∂u ∂v
⇒ u +v =0
∂y ∂y
FG IJ FG IJ
pi

∂v ∂u ∂u ∂v ∂v ∂u
and

∂x
+v
H
u −
∂x
=0
K H K ...(3) ∵
∂y
=−
∂x
=
∂y ∂x
as

Squaring and adding eqns. (2) and (3), we get


R|F ∂u I + F ∂vI U|
2 2

(u2 + v 2) S|GH ∂x JK GH ∂x JK V| = 0
g

T W
.c

FG ∂u IJ + FG ∂v IJ = 0
2 2
u2 + v 2 = c2 ≠ 0

H ∂x K H ∂x K |∵
w

∂u ∂v
⇒ | f ′(z) |2 = 0 ∵ f ′ ( z) = +i
w

∂x ∂x
⇒ | f ′(z) | = 0
w

⇒ f(z) is constant.

1.16 APPLICATION OF ANALYTIC FUNCTIONS TO FLOW PROBLEMS

Since the real and imaginary parts of an analytic function satisfy the Laplace’s equation in
two variables, these conjugate functions provide solutions to a number of field and flow problems.
For example, consider the two dimensional irrotational motion of an incompressible
fluid, in planes parallel to xy-plane.

Let V be the velocity of a fluid particle, then it can be expressed as

V = vx i + v y j ...(1)

www.cgaspirants.com
www.cgaspirants.com

10 A TEXTBOOK OF ENGINEERING MATHEMATICS

Since the motion is irrotational, there exists a scalar function φ(x, y), such that
→ ∂φ ∂φ
i+
V = ∇φ(x, y) = j ...(2)
∂x ∂y
∂φ ∂φ
From (1) and (2), we have vx = and vy = ...(3)
∂x ∂y
The scalar function φ(x, y), which gives the velocity components, is called the velocity
potential function or simply the velocity potential.

Also the fluid being incompressible, div V = 0


FG i ∂ + j ∂ IJ (v i + v j) = 0
H ∂x ∂y K x y

m
∂vx ∂vy
⇒ + =0 ...(4)
∂x ∂y

o
Substituting the values of vx and vy from (3) in (4), we get

.c
FG IJ
∂ ∂φ ∂ ∂φ FG IJ ∂2φ ∂2φ
H K+ = 0 or
H K + =0
n ∂x ∂x ∂y ∂y
ts ∂x 2 ∂y 2
Thus, the function φ is harmonic and can be treated as real part of an analytic function
w = f(z) = φ(x, y) + i ψ (x, y)
ra
For interpretation of conjugate function ψ (x, y), the slope at any point of the curve
ψ (x, y) = c′ is given by
∂φ
pi

∂ψ
dy ∂y
= − ∂x = | By C-R equations
dx ∂ψ ∂φ
as

∂y ∂x
vy
g

= | By (3)
vx
.c

This shows that the resultant velocity vx 2 + vy 2 of the fluid particle is along the tangent
w

to the curve ψ (x, y) = c′ i.e., the fluid particles move along this curve. Such curves are known
as stream lines and ψ (x, y) is called the stream function. The curves represented by
w

φ (x, y) = c are called equipotential lines.


Since φ(x, y) and ψ(x, y) are conjugate functions of analytic function w = f(z), the
w

equipotential lines φ (x, y) = c and the stream lines ψ (x, y) = c′, intersect each other orthogonally.
dw ∂φ ∂ψ ∂φ ∂φ
Now, = +i = −i | By C-R equations
dz ∂x ∂x ∂x ∂y
= vx – ivy | By (3)
dw
∴ The magnitude of resultant velocity = = vx 2 + vy 2
dz
The function w = f(z) which fully represents the flow pattern is called the complex potential.

www.cgaspirants.com
www.cgaspirants.com

FUNCTION OF COMPLEX VARIABLE 11

In the study of electrostatics and gravitational fields, the curves φ(x, y) = c and ψ (x, y) = c′
are called equipotential lines and lines of force respectively. In heat flow problems, the
curves φ (x, y) = c and ψ (x, y) = c′ are known as isothermals and heat flow lines respectively.

1.17 DETERMINATION OF THE CONJUGATE FUNCTION

If f(z) = u + iv is an analytic function where both u(x, y) and v(x, y) are conjugate functions,
then we determine the other function v when one of these say u is given as follows:
∵ v = v (x, y)
∂v ∂v
∴ dv = dx + dy
∂x ∂y

m
∂u ∂u
⇒ dv = – dx + dy ...(1) | By C-R eqns.
∂y ∂x

o
∂u ∂u
M=– , N=

.c
∂y ∂x
∂M ∂ 2u ∂N ∂ 2 u

ts
∴ = − 2 and =
∂y ∂y ∂x ∂ x 2
∂M ∂N
n
Now, = gives
∂y ∂x
ra

∂ 2u ∂ 2u
– =
∂y 2 ∂x 2
pi

∂ 2u ∂ 2u
or + =0
∂x 2 ∂y 2
as

which is true as u being a harmonic function satisfies Laplace’s equation.


∴ dv is exact.
g

∴ dv can be integrated to get v.


.c

However, if we are to construct f(z) = u + iv when only u is given, we first of all find v by
above procedure and then write f(z) = u + iv.
w

∂v ∂v
Similarly, if we are to determine u and only v is given then we use du = dx − dy
∂y ∂x
w

and integrate it to find u. Consequently f(z) = u + iv can also be determined.


w

1.18 MILNE’S THOMSON METHOD

With the help of this method, we can directly construct f(z) in terms of z without first finding
out v when u is given or u when v is given.
z = x + iy
z = x – iy
1 1
⇒ x = (z + z ) and y = (z – z )
2 2i
∴ f(z) = u(x, y) + iv(x, y)

=u
RS
z+ z z− z
,
UV
+ iv
RS
z+ z z−z
,
UV ...(1)
T2 2i W T 2 2i W

www.cgaspirants.com
www.cgaspirants.com

12 A TEXTBOOK OF ENGINEERING MATHEMATICS

Relation (1) is an identity in z and z . Putting z = z, we get


f(z) = u(z, 0) + iv(z, 0) ...(2)
Now, f(z) = u + iv
∂u ∂v ∂u ∂u
⇒ f ′(z) = +i = −i | By C-R eqns.
∂x ∂x ∂x ∂y
= φ1(x, y) – i φ2(x, y)
∂u ∂u
where φ 1 ( x, y) = and φ 2 ( x, y) =
∂x ∂y
Now, f ′(z) = φ1 (z, 0) – i φ2(z, 0) | Replacing x by z and y by 0
Integrating, we get

m
f(z) = {φ1(z, 0) – i φ2(z, 0)} dz + c | c is an arbitrary constant.

o
Hence the function is constructed directly in terms of z.

.c
Similarly if v(x, y) is given, then

f(z) = z [ψ1(z, 0) + iψ2(z, 0)] dz + c


n ts ψ 1 ( x, y) =
∂v
∂y
and ψ 2 ( x, y) =
∂v
∂x

Milne’s Thomson method can easily be grasped by going through the steps involved in
ra

following various cases.


Case I. When only real part u(x, y) is given.
pi

To construct analytic function f(z) directly in terms of z when only real part u is given,
we use the following steps:
as

∂u
1. Find
∂x
g

2. Write it as equal to φ1(x, y)


.c

∂u
3. Find
∂y
w

4. Write it as equal to φ2(x, y)


5. Find φ1(z, 0) by replacing x by z and y by 0 in φ1(x, y).
w

6. Find φ2(z, 0) by replacing x by z and y by 0 in φ2(x, y).


7. f(z) is obtained by the formula
w

f(z) =
z {φ 1 ( z, 0) – iφ 2 ( z, 0)} dz + c directly in terms of z.
Case II. When only imaginary part v(x, y) is given.
To construct analytic function f(z) directly in terms of z when only imaginary part v is
given, we use the following steps :
∂v
1. Find
∂y
2. Write it as equal to ψ1(x, y)
∂v
3. Find
∂x

www.cgaspirants.com
www.cgaspirants.com

FUNCTION OF COMPLEX VARIABLE 13

4. Write it as equal to ψ2(x, y)


5. Find ψ1(z, 0) by replacing x by z and y by 0 in ψ1(x, y)
6. Find ψ2(z, 0) by replacing x by z and y by 0 in ψ2(x, y)
7. f(z) is obtained by the formula

f(z) = z
Case III. When u – v is given.
{ ψ 1 ( z, 0) + iψ 2 ( z, 0)} dz + c directly in terms of z.

To construct analytic function f(z) directly in terms of z when u – v is given, we follow


the following steps:
1. f(z) = u + iv ...(1)
2. i f(z) = iu – v ...(2)

m
3. Add (1) and (2) to get
(1 + i) f(z) = (u – v) + i(u + v)

o
or, F(z) = U + iV

.c
where F(z) = (1 + i) f(z), U = u – v and V = u + v
4. Since u – v is given hence U(x, y) is given

5. Find
∂U
∂x
6. Write it as equal to φ1(x, y)
n ts
ra

∂U
7. Find
∂y
pi

8. Write it as equal to φ2(x, y)


9. Find φ1(z, 0)
as

10. Find φ2(z, 0)


11. F(z) is obtained by the formula

z
g

F(z) = { φ1 ( z , 0) – iφ 2 ( z , 0)} dz + c
.c

F( z)
12. f(z) is determined by f(z) = directly in terms of z.
1+ i
w

Case IV. When u + v is given.


w

To construct analytic function f(z) directly in terms of z when u + v is given, we follow


the following steps:
w

1. f(z) = u + iv ...(1)
2. i f(z) = iu – v ...(2)
3. Add (1) and (2) to get
(1 + i) f(z) = (u – v) + i(u + v)
⇒ F(z) = U + iV
where, F(z) = (1 + i) f(z), U = u – v and V = u + v
4. Since u + v is given hence V(x, y) is given
∂V
5. Find
∂y
6. Write it as equal to ψ1(x, y)

www.cgaspirants.com
www.cgaspirants.com

14 A TEXTBOOK OF ENGINEERING MATHEMATICS

∂V
7. Find
∂x
8. Write it as equal to ψ2(x, y)
9. Find ψ1(z, 0)
10. Find ψ2(z, 0)
11. F(z) is obtained by the formula
F(z) = z { ψ 1 ( z, 0) + iψ 2 ( z, 0)} dz + c
F( z)
12. f(z) is determined by f(z) = directly in terms of z.
1+ i

m
EXAMPLES

o
Example 1. Find the values of c1 and c2 such that the function

.c
f(z) = x2 + c1y2 – 2xy + i (c2x2 – y2 + 2xy)
is analytic. Also find f ′(z). (U.K.T.U. 2011)
Sol. Here
Comparing (1) with f(z) = u(x, y) + iv(x, y), we get
n
u(x, y) = x2 + c1y2 – 2xy
ts
f(z) = (x2 + c1y2 – 2xy) + i (c2x2 – y2 + 2xy) ...(1)

...(2)
ra
and 2 2
v(x, y) = c2x – y + 2xy ...(3)
For the function f(z) to be analytic, it should satisfy Cauchy-Riemann equations.
∂u
pi

∂u
Now from (2), = 2x – 2y and = 2c1y – 2x
∂x ∂y
∂v ∂v
as

Also, from (3), = 2c2x + 2y and = – 2y + 2x


∂x ∂y
Cauchy-Riemann eqns. are
g

∂u ∂v
.c

=
∂x ∂y
⇒ 2x – 2y = – 2y + 2x which is true.
w

∂u ∂v
and =−
w

∂y ∂x
⇒ 2c1y – 2x = – 2c2x – 2y ...(4)
w

Comparing the coefficients of x and y in eqn. (4), we get


2c1 = – 2 ⇒ c1 = – 1
and – 2 = – 2c2 ⇒ c2 = 1
Hence c1 = – 1 and c2 = 1
∂u ∂v
Now, f ′(z) = +i = 2x – 2y + i(2c2x + 2y)
∂x ∂x
= 2x – 2y + i(2x + 2y) | ∵ c2 = 1
= 2(x + iy) + 2i(x + iy)
= 2z + 2iz = 2(1 + i)z.
Example 2. Find p such that the function f(z) expressed in polar coordinates as
f(z) = r2 cos 2θ + ir2 sin pθ is analytic.

www.cgaspirants.com
www.cgaspirants.com

FUNCTION OF COMPLEX VARIABLE 15

Sol. Let f(z) = u + iv, then u = r2 cos 2θ, v = r2 sin pθ


∂u ∂v
= 2r cos 2θ, = 2r sin pθ
∂r ∂r
∂u ∂v
= – 2r2 sin 2θ, = pr2 cos pθ
∂θ ∂θ
∂u 1 ∂v ∂v 1 ∂u
For f(z) to be analytic, = and =−
∂r r ∂θ ∂r r ∂θ
∴ 2r cos 2θ = pr cos pθ and 2r sin pθ = 2r sin 2θ
Both these equations are satisfied if p = 2.
Example 3. (i) Prove that the function sinh z is analytic and find its derivative.

m
(U.K.T.U. 2010)
(ii) Show that f(z) = log z is analytic everywhere in the complex plane except at the origin

o
FG 1IJ .
and that its derivative is
H zK

.c
Sol. (i) Here f(z) = u + iv = sinh z = sinh (x + iy) = sinh x cos y + i cosh x sin y

∂u
n
= cosh x cos y,
∂u
ts
u = sinh x cos y and v = cosh x sin y

= – sinh x sin y
∂x ∂y
ra

∂v ∂v
= sinh x sin y, = cosh x cos y
pi

∂x ∂y
∂u ∂v ∂u ∂v
= =−
as

∴ and
∂x ∂y ∂y ∂x
Thus C-R equations are satisfied.
g

∂u ∂u ∂v ∂v
, ,
.c

Since sinh x, cosh x, sin y and cos y are continuous functions, and are
∂x ∂y ∂x ∂y
also continuous functions satisfying C-R equations.
w

Hence f(z) is analytic everywhere.


w

∂u ∂v
Now f ′(z) = +i = cosh x cos y + i sinh x sin y = cosh (x + iy) = cosh z.
∂x ∂x
w

(ii) Here f(z) = u + iv = log z = log (x + iy)


Let x = r cos θ and y = r sin θ so that
x + iy = r (cos θ + i sin θ) = reiθ

1 FG y IJ
log (x + iy) = log (r eiθ) = log r + iθ =
2
log (x2 + y2) + i tan–1 H xK
Separating real and imaginary parts, we get
1 FG y IJ
u=
2
log (x2 + y2) and v = tan–1
H xK

www.cgaspirants.com
www.cgaspirants.com

16 A TEXTBOOK OF ENGINEERING MATHEMATICS

∂u x ∂u y
Now, = 2 ,
2 ∂y
= 2
∂x x +y x + y2

∂v −y ∂v x
and = 2 2
, = 2
∂x x +y ∂y x + y2
We observe that the Cauchy-Riemann equations
∂u ∂v ∂u ∂v
= and =–
∂x ∂y ∂y ∂x
are satisfied except when x2 + y2 = 0 i.e., when x = 0, y = 0
Also derivatives are continuous except at origin.

m
Hence the function f(z) = log z is analytic everywhere in the complex plane except at the
origin.

o
∂u ∂v x − iy x − iy 1 1

.c
Also, f ′(z) = +i = 2 = = =
∂x ∂x x + y2 ( x + iy)( x − iy) x + iy z
Example 4. Show that the function ex (cos y + i sin y) is holomorphic and find its
derivative.
Sol.
n
f(z) = ex cos y + i ex sin y = u + iv
ts
Here, u = ex cos y, v = ex sin y
ra

∂u ∂v
= ex cos y = ex sin y
∂x ∂x
pi

∂u ∂v
= – ex sin y = ex cos y
as

∂y ∂y

∂u ∂v ∂u ∂v
Since, = and =−
g

∂x ∂y ∂y ∂x
.c

hence, C-R equations are satisfied. Also first order partial derivatives of u and v are continuous
everywhere. Therefore f(z) is analytic.
w

∂u ∂v
Now, f ′(z) = +i = ex cos y + i ex sin y
w

∂x ∂x
= ex (cos y + i sin y) = ex . eiy = ex+iy = ez
w

Example 5. If n is real, show that rn (cos nθ + i sin nθ) is analytic except possibly when
r = 0 and that its derivative is
nrn–1 [cos (n – 1) θ + i sin (n – 1) θ].
Sol. Let w = f(z) = u + iv = rn (cos nθ + i sin nθ)
Here, u = rn cos nθ, v = rn sin nθ
∂u ∂v
then, = nrn–1 cos nθ = nrn–1 sin nθ
∂r ∂r
∂u ∂v
= – nrn sin nθ = nrn cos nθ
∂θ ∂θ

www.cgaspirants.com
www.cgaspirants.com

FUNCTION OF COMPLEX VARIABLE 17

∂u 1 ∂v ∂v 1 ∂u
Thus, we see that, = and =−
∂r r ∂θ ∂r r ∂θ
∴ Cauchy-Riemann equations are satisfied. Also first order partial derivatives of u
and v are continuous everywhere.
dw
Hence f(z) is analytic if f ′(z) or exists for all finite values of z.
dz
dw ∂w
We have, = (cos θ – i sin θ)
dz ∂r
= (cos θ – i sin θ) . nrn–1 (cos nθ + i sin nθ)
= nrn–1 [cos (n – 1) θ + i sin (n – 1) θ]

m
This exists for all finite values of r including zero, except when r = 0 and n ≤ 1.
Example 6. Show that if f(z) is analytic and Re f(z) = constant then f(z) is a constant.

o
(U.P.T.U. 2006)
Sol. Since the function f(z) = u (x, y) + iv (x, y) is analytic, it satisfies the Cauchy-

.c
Riemann equations

Also,
∂u ∂v
=
∂x ∂y
and
n ∂u
∂y
=−
∂v
∂x ts
Re f(z) = constant, therefore u(x, y) = c1
ra

∂u ∂u
∴ =0= .
∂x ∂y
pi

∂v ∂v
Using C-R equations, =0=
∂x ∂y
as

Hence v(x, y) = c2 = a real constant


Therefore f(z) = u(x, y) + iv(x, y) = c1 + ic2 = a complex constant.
g

F I.
.c

y
Example 7. Given that u(x, y) = x2 – y2 and v(x, y) = – GH x 2
+y 2 JK
w

Prove that both u and v are harmonic functions but u + iv is not an analytic function of z.
w

Sol. u = x 2 – y2
∂u ∂ 2u
w

= 2x ⇒ =2
∂x ∂x 2
∂u ∂ 2u
= – 2y ⇒ =–2
∂y ∂y 2

∂ 2u ∂ 2u
Since + =0 Hence u(x, y) is harmonic.
∂x 2 ∂y 2
−y
Also, v=
x + y2
2

∂v 2 xy ∂ 2v 2 y3 − 6 x 2 y
= 2 ⇒ =
∂x (x + y2 )2 ∂x 2 (x 2 + y2 )3

www.cgaspirants.com
www.cgaspirants.com

18 A TEXTBOOK OF ENGINEERING MATHEMATICS

∂v y2 − x 2 ∂2v 6 x 2 y − 2 y3
= 2 ⇒ 2 =
∂y (x + y2 )2 ∂y (x 2 + y2 )3
∂ 2v ∂ 2v
Since + = 0. Hence v(x, y) is also harmonic.
∂x 2 ∂y 2
∂u ∂v ∂v ∂u
But, ≠ and ≠–
∂x ∂y ∂x ∂y
Therefore u + iv is not an analytic function of z.
Example 8. If φ and ψ are functions of x and y satisfying Laplace’s equation, show that
s + it is analytic, where

m
∂φ ∂ψ ∂φ ∂ψ
s= − and t = + .
∂y ∂x ∂x ∂y

o
[U.K.T.U. 2010, G.B.T.U. (C.O.) 2011]

.c
Sol. Since φ and ψ are functions of x and y satisfying Laplace’s equations,
∂2φ ∂2φ

and
∴ +
∂x 2 ∂y 2
∂2ψ ∂2ψ
+
=0n
= 0.
ts ...(1)

...(2)
∂x 2 ∂y 2
ra

For the function s + it to be analytic,


pi

∂s ∂t
= ...(3)
∂x ∂y
as

∂s ∂t
and =− ...(4)
∂y ∂x
must satisfy.
g

∂s FG
∂ ∂φ ∂ψ IJ
∂2φ ∂2 ψ
.c

= −
Now,
H
∂x ∂x ∂y ∂x
=
K −
∂x∂y ∂x 2
...(5)
w

∂t ∂ F ∂φ ∂ψ I 2 2
= G + J = ∂∂y∂φx + ∂∂yψ
∂y ∂y H ∂x ∂y K 2 ...(6)
w

∂s ∂ F ∂φ ∂ψ I ∂ φ ∂ ψ
2 2
G − J = − ∂y∂x
w

= ...(7)
∂y ∂y H ∂y ∂x K ∂y 2

∂t ∂ F ∂φ ∂ψ I 2 2
and = G + J = ∂∂xφ + ∂∂x∂ψy .
∂x ∂x H ∂x ∂y K 2
...(8)

From (3), (5) and (6), we have


∂2φ ∂2ψ ∂2φ ∂2ψ ∂2ψ ∂2ψ
− = + ⇒ + =0
∂x∂y ∂x 2 ∂y∂x ∂y 2 ∂x 2 ∂y 2
which is true by (2).

www.cgaspirants.com
www.cgaspirants.com

FUNCTION OF COMPLEX VARIABLE 19

Again from (4), (7) and (8), we have

∂2φ ∂2 ψ ∂2φ ∂2 ψ ∂2φ ∂2φ


− =− 2 − ⇒ + =0
∂y 2
∂y∂x ∂x ∂x∂y ∂x 2 ∂y 2
which is also true by (1).
Hence the function s + it is analytic.

xy 2 (x + iy)
Example 9. Verify if f(z) = , z ≠ 0 ; f(0) = 0 is analytic or not?
x 2 + y4
[U.P.T.U. (C.O.) 2008]

m
xy 2 ( x + iy)
Sol. u + iv = ;z≠0

o
x2 + y4

.c
x 2 y2 xy 3
∴ u= , v =

At the origin,
∂u
= lim
∂x x→0
x2 + y4

x
x2 + y4

u( x, 0) − u(0, 0)
n
= lim
x→0
0−0
x
=0
ts
ra

∂u u (0, y) − u (0, 0) 0−0


= lim = lim =0
∂y y→0 y y → 0 y
pi

∂v v( x, 0) − v(0, 0) 0−0
= lim = lim =0
as

∂x x→0 x x → 0 x
∂v v(0, y) − v(0, 0) 0−0
= lim = lim
g

=0
∂y y→0 y y→ 0 y
.c

∂u ∂v ∂u − ∂v
Since = and =
w

∂x ∂y ∂y ∂x
Hence Cauchy-Riemann equations are satisfied at the origin.
w

f ( z) − f (0) LM
xy 2 ( x + iy) OP 1 xy 2
w

But f ′(0) = lim = lim − 0 . = lim


z→ 0 z x→ 0
y→ 0 N
x2 + y4 Q x + iy x→0 x 2 + y 4
y→ 0

Let z → 0 along the real axis y = 0, then


f ′(0) = 0
Again let z → 0 along the curve x = y2 then
x2 1
f ′(0) = lim 2 2
=
x +x
x→0 2
which shows that f ′(0) does not exist since the limit is not unique along two different paths.
Hence f(z) is not analytic at origin although Cauchy-Riemann equations are satisfied there.

www.cgaspirants.com
www.cgaspirants.com

20 A TEXTBOOK OF ENGINEERING MATHEMATICS

Example 10. Show that the function defined by f(z) = | xy| is not regular at the origin,
although Cauchy-Riemann equations are satisfied there. [G.B.T.U. (C.O.) 2011]
Sol. Let f(z) = u(x, y) + iv(x, y) = | xy| then u(x, y) = | xy|, v(x, y) = 0
At the origin (0, 0), we have
∂u u( x, 0) − u(0, 0) 0−0
= Lt = Lt =0
∂x x → 0 x x→0 x
∂u u(0, y) − u(0, 0) 0−0
= Lt = Lt =0
∂y y → 0 y y→0 y
∂v v( x, 0) − v(0, 0) 0−0
= Lt = Lt =0

m
∂x x → 0 x x→0 x
∂v v(0, y) − v(0, 0) 0−0
= Lt = Lt =0

o
∂y y → 0 y y→0 y

.c
∂u ∂v ∂u ∂v
Clearly, = , =−
∂x ∂y ∂y ∂x
Hence C-R equations are satisfied at the origin.

Now f ′(0) = Lt
z→0
f ( z) − f (0)
n
z
= Lt
z→0
| xy| − 0
x + iy
ts
ra

If z → 0 along the line y = mx, we get

|mx 2 | |m|
pi

f ′(0) = Lt = Lt
x → 0 x(1 + im) x → 0 1 + im
as

Now this limit is not unique since it depends on m. Therefore, f ′(0) does not exist.
Hence the function f(z) is not regular at the origin.
Example 11. Prove that the function f(z) defined by
g

x 3 (1 + i) − y 3 (1 − i)
.c

f(z) = , z ≠ 0 and f(0) = 0


x2 + y2
w

is continuous and the Cauchy-Riemann equations are satisfied at the origin, yet f ′(0) does not
exist. (U.P.T.U. 2015)
w

( x 3 − y 3 ) + i( x 3 + y 3 )
Sol. Here, f(z) = ,z≠0
w

x2 + y2
x 3 − y3 x 3 + y3
Let f(z) = u + iv = +i ,
x2 + y2 x 2 + y2

x 3 − y3 x 3 + y3
then u= , v =
x 2 + y2 x 2 + y2
Since z ≠ 0 ⇒ x ≠ 0, y ≠ 0
∴ u and v are rational functions of x and y with non-zero denominators. Thus, u, v and
hence f(z) are continuous functions when z ≠ 0. To test them for continuity at z = 0, on changing
u, v to polar co-ordinates by putting x = r cos θ, y = r sin θ, we get

www.cgaspirants.com
www.cgaspirants.com

FUNCTION OF COMPLEX VARIABLE 21

u = r(cos3 θ – sin3 θ) and v = r (cos3 θ + sin3 θ)


When z → 0, r → 0
∴ Lt u = Lt r (cos3 θ – sin3 θ) = 0
z→0 r→0

Similarly, Lt v = 0
z→0

∴ Lt f(z) = 0 = f(0)
z→0
⇒ f(z) is continuous at z = 0.
Hence f(z) is continuous for all values of z.
At the origin (0, 0), we have

m
∂u u( x, 0) − u(0, 0) x−0
= Lt = Lt =1
∂x x → 0 x x→0 x

o
∂u u(0, y) − u(0, 0) − y−0
= Lt = Lt =–1
∂y y → 0 y y

.c
y→0

∂v v( x, 0) − v(0, 0) x−0
= Lt = Lt =1
∂x x → 0
∂v
= Lt
∂y y → 0
n
y
x
v(0, y) − v(0, 0)
x→0

= Lt
y→0
x
y−0
y
=1
ts
ra

∂u ∂v ∂u ∂v
∴ = and =−
∂x ∂y ∂y ∂x
pi

Hence C-R equations are satisfied at the origin.


f ( z) − f (0) ( x 3 − y 3 ) + i( x 3 + y 3 ) − 0
Now f ′(0) = Lt = Lt
as

z→0 z z→0 ( x 2 + y 2 )( x + iy)


Let z → 0 along the line y = x, then
g

0 + 2ix 3 i i(1 − i) 1 + i
.c

f ′(0) = Lt 3
= = = ...(1)
x→0 2 x (1 + i) 1 + i 2 2
w

Also, let z → 0 along the x-axis (i.e. y = 0), then


x 3 + ix 3
w

f ′(0) = Lt =1+i ...(2)


x→0 x3
Since the limits (1) and (2) are different, f ′(0) does not exist.
w

Example 12. Show that the function f (z) = e − z −4 , z ≠ 0 and f(0) = 0 is not analytic at
z = 0, although Cauchy-Riemann equations are satisfied at this point. [U.P.T.U. (C.O.) 2008]
−4 −4
−z − ( x + iy)
Sol. Here, f(z) = e =e
1
.
( x − iy) 4 −
R|S ( x − iy )
4 U|V

( x + iy) ( x − iy) 4
4 |T ( x + y )
2 2 4
|W
=e = e
1
− 2 2 4 [( x 4 + y 4 − 6 x 2 y 2 ) − 4 ixy( x 2 − y 2 )]
(x + y )
=e

www.cgaspirants.com
www.cgaspirants.com

22 A TEXTBOOK OF ENGINEERING MATHEMATICS

LM x OP
PQ LMcos 4 xy ( x 2 − y 2 ) + i sin 4 xy ( x 2 − y 2 ) OP
4
+ y 4 − 6 x 2 y2

⇒ u + iv = e
MN ( x 2 + y2 ) 4

N ( x2 + y2 )4 ( x 2 + y2 ) 4 Q
Lx
−M
4
+ y4 − 6 x2 y2 OP

M
u= e N
( x 2 + y2 ) 4 PQ cos 4 xy ( x 2 − y 2 )
(x2 + y2 )4

LM x 4
+ y4 − 6 x 2 y2 OP
and v= e
MN ( x 2 + y2 )4 PQ sin 4 xy ( x 2 − y 2 )
(x2 + y2 )4
∂u u ( x, 0) − u (0, 0)
At z = 0, = lim
∂x x → 0 x

m
−4
e− x −0 1
= lim = lim

o
−4
x→0 x x→0 xe x

.c
1 1
= lim
x→0 L 1
x M1 +
1 OP = lim x + 1
x→0 1
=0

∂u
n N x
u(0, y) − u(0, 0) − e− y
4
+
2x8
+
ts Q
−4
x 3
+
2 x7
+

= lim = lim =0
ra
∂y y→0 y y→0 y
∂v v( x, 0) − v(0, 0) 0
= lim = lim =0
pi

∂x x → 0 x x → 0 x
∂v v(0, y) − v(0, 0) 0
as

and = lim = lim = 0.


∂y y → 0 y y → 0 y
Hence Cauchy-Riemann Conditions are satisfied at z = 0.
g

−4
f ( z) − f (0) e− z
.c

But f ′(0) = lim = lim


z→0 z z→0 z
iπ / 4 −4
w

e − (re )
= lim ; if z → 0 along z = reiπ/4
r→0 re iπ / 4
w

−4
er
= lim=∞
re iπ / 4
w

r→0
which shows that f ′(z) does not exist at z = 0. Hence f(z) is not analytic at z = 0.
Example 13. (i) Examine the nature of the function
R| x y (x + iy) , z ≠ 0U|
2 5

f ( z) = S x + y
4 10 V| in the region including the origin.
|T0 , z = 0W
3R| x y(y − ix) , z ≠ 0U| f(z) − f(0)
(ii) If f(z) = S x + y6 2 V , prove that → 0 as z → 0 along any radius
|T 0 , z = 0 |W
z
vector but not as z → 0 in any manner and also that f(z) is not analytic at z = 0.
[G.B.T.U. 2013, U.K.T.U. 2010]

www.cgaspirants.com
www.cgaspirants.com

FUNCTION OF COMPLEX VARIABLE 23

x 2 y 5 ( x + iy)
Sol. (i) Here, u + iv = ;z≠0
x 4 + y 10

x 3 y5 x 2 y6
∴ u= 4 10 , v =
x +y x + y 10
4

∂u u( x, 0) − u(0, 0) 0 −0
At the origin, = lim = lim =0
∂x x → 0 x x→0 x
∂u u(0, y) − u(0, 0) 0 −0
= lim = lim =0
∂y y → 0 y y→0 y
∂v ∂v

m
Similarly, =0=
∂x ∂y
∂u ∂v ∂u ∂v

o
Since = and =−
∂x ∂y ∂y ∂x

.c
Hence Cauchy-Riemann equations are satisfied at the origin
LM
x 2 y 5 ( x + iy) OP
But f ′(0) = lim
z→0
n f ( z) − f (0)

x y
z
2 5
= lim
x→0
ts
4
x +y
y→0 N 10
−0 .
Q
1
x + iy

= xlim
ra

→0 x 4 + y 10
y→0
Let z → 0 along the radius vector y = mx, then
pi

m5 x 7 m5 x 3
f ′(0) = lim = lim =0
x → 0 x 4 + m 10 x 10 x → 0 1 + m 10 x 6
as

Again let z → 0 along the curve y5 = x2


1 x4
g

f ′(0) = xlim 4 4
=
x +x
→0 2
.c

which shows that f ′(0) does not exist. Hence f(z) is not analytic at origin although Cauchy-
Riemann equations are satisfied there.
w

(ii)
f ( z) − f (0)
=
LM
x 3 y( y − ix)
− 0 .
1
=
OP
− ix 3 y( x + iy)
.
1
= – i
x3 y
w

z N
x 6 + y2 x + iy Q(x6 + y2 ) x + iy x 6 + y2
Let z → 0 along radius vector y = mx then,
w

f ( z) − f (0) − ix 3 (mx) − imx 2


lim = lim 6 = lim 4 =0
z→0 z x → 0 x + m2 x 2 x → 0 x + m2

f ( z) − f (0)
Hence → 0 as z → 0 along any radius vector.
z
Now let z → 0 along a curve y = x3 then,
f ( z) − f (0) − ix 3 . x 3 − i
lim = lim 6 =
z→0 z x → 0 x + x6 2
f ( z) − f (0)
Hence does not tend to zero as z → 0 along the curve y = x3.
z
We observe that f ′(0) does not exist hence f(z) is not analytic at z = 0.

www.cgaspirants.com
www.cgaspirants.com

24 A TEXTBOOK OF ENGINEERING MATHEMATICS

Example 14. Show that the following functions are harmonic and find their harmonic
conjugate functions.
1
(i) u = log(x2 + y2) (U.P.T.U. 2015) (ii) v = sinh x cos y.
2
1
Sol. (i) u= log (x2 + y2) ...(1)
2
∂u 1 1 x
= . 2 2
. 2x = 2
∂x 2 x + y x + y2

∂ 2u ( x 2 + y2 ) . 1 − x . 2 x y2 − x2
= = ...(2)
∂x 2 (x 2 + y2 )2 (x2 + y2 )2

m
∂u 1 1 y
Also, = . 2 . 2y = 2

o
2
∂y 2 x + y x + y2

.c
∂ 2u ( x 2 + y2 ) . 1 − y . 2 y x 2 − y2
= = ...(3)
∂y 2 (x2 + y2 )2 (x2 + y2 )2

∂ 2u ∂ 2u
+
∂x 2 ∂y 2
= 0.
n ts [From (2) and (3)]
ra
Since u satisfies Laplace’s equation hence u is a harmonic function.
∂v ∂v
Let dv = dx + dy
pi

∂x ∂y
FG ∂u IJ dx + FG ∂u IJ dy
H ∂y K H ∂x K
= − [Using C-R equations]
as

F − y I dx + F x I dy
g

=G
H x + y JK GH x + y JK
2 2 2 2
.c

x dy − y dx L F yI O
= d Mtan G J P
−1

N H xKQ
w

= 2 2
(x + y )
F yI
w

v = tan G J + c
Integration yields,
H xK
–1 | c is a constant
w

which is the required harmonic conjugate function of u.


(ii) v = sinh x cos y ...(1)
2
∂v ∂ v
= cosh x cos y ⇒ = sinh x cos y ...(2)
∂x ∂x 2
∂v ∂2v
= – sinh x sin y ⇒ = – sinh x cos y ...(3)
∂y ∂y 2
∂2v ∂2v
Since, + =0
∂x 2 ∂y 2
Hence v is harmonic.

www.cgaspirants.com
www.cgaspirants.com

FUNCTION OF COMPLEX VARIABLE 25

∂u ∂u ∂v ∂v
Now, du = dx + dy = dx − dy
∂x ∂y ∂y ∂x
= – sinh x sin y dx – cosh x cos y dy
= – [sinh x sin y dx + cosh x cos y dy]
= – d (cosh x sin y).
Integration yields, u = – cosh x sin y + c | c is a constant
4 2 2 4
Example 15. (i) Show that the function u(x, y) = x – 6x y + y is harmonic. Also find
the analytic function f(z) = u(x, y) + iv(x, y). (U.P.T.U. 2007)
(ii) Show that the function u = x3 – 3xy2 is harmonic and find the corresponding analytic
function. [U.P.T.U. (C.O.) 2008]

m
x
(iii) Show that e cos y is a harmonic function, find the analytic function of which it is real
part. [U.P.T.U. (C.O.) 2008]

o
Sol. (i) u = x4 – 6x2y2 + y4

.c
∂u ∂ 2u
∴ = 4x3 – 12xy2 ⇒ = 12x2 – 12y2
∂x ∂x 2

∂u
∂y
= – 12 x2y + 4y3 ⇒
n ts
∂ 2u
∂y 2
= – 12x2 + 12y2
ra

∂ 2u ∂ 2u
Since, + =0 ∴ u(x, y) is a harmonic function.
∂x 2 ∂y 2
pi

Now, let dv =
∂v
dx +
∂v FG
dy = −
∂u
dx +
IJ
∂u
dy | By C-R eqns.
∂x ∂y H∂y K
∂x
as

= (12x2y – 4y3) dx + (4x3 – 12xy2) dy


= (12x2y dx + 4x3dy) – (4y3 dx + 12xy2 dy)
g

= d(4x3y) – d(4xy3)
.c

Integration yields, v = 4x3y – 4xy3 + c


w

Hence f(z) = u + iv = x4 – 6x2y2 + y4 + i(4x3y – 4xy3 + c)


= (x + iy)4 + c1 = z4 + c1 | where c1 = ic
w

(ii) u = x3 – 3xy2
∂u ∂ 2u
w

∴ = 3x2 – 3y2 ⇒ = 6x
∂x ∂x 2
∂u ∂ 2u
= – 6xy ⇒ = – 6x
∂y ∂y 2
∂ 2u ∂ 2u
Since, + =0 ∴ u is a harmonic function.
∂x 2 ∂y 2

Now, dv =
dv
dx +
∂v FG IJ
dy = − ∂u dx + ∂u dy | By C-R eqns.
dx ∂y H∂y K
∂x

www.cgaspirants.com
www.cgaspirants.com

26 A TEXTBOOK OF ENGINEERING MATHEMATICS

= 6xy dx + (3x2 – 3y2) dy = (6xy dx + 3x2 dy) – 3y2 dy


= d(3x2y) – d(y3)
Integration yields,
v = 3x2y – y3 + c
∴ f(z) = u + iv = x3 – 3xy2 + i(3x2y – y3 + c)
= (x + iy)3 + ic = z3 + c1 (where c1 = ic)
(iii) Let u= ex cos y

∂u ∂ 2u
∴ = ex cos y ⇒ = ex cos y
∂x ∂x 2

m
∂u ∂ 2u
= – ex sin y ⇒ = – ex cos y
∂y

o
∂y 2

.c
∂ 2u ∂ 2u
Since + =0 ∴ u is a harmonic function.
∂x 2 ∂y 2

Let dv =
∂v
n
∂x
dx +
∂v
∂y
dy = −
∂u
∂y
FG
dx +
H
∂u
∂x
dy
tsIJ
K
FG IJ
H K | By C-R eqns.
ra

= e x sin y dx + ex cos y dy = d (e x sin y)


Integration yields,
pi

v = e x sin y + c
Hence f(z) = u + iv = e x cos y + i (e x sin y + c)
as

= e x(cos y + i sin y) + c1 | where c1 = ic


= ex + iy + c1 = ez + c1.
g

y
Example 16. (i) In a two-dimensional fluid flow, the stream function is ψ = – ,
x + y2
2
.c

find the velocity potential φ. [M.T.U. 2014]


w

(ii) An electrostatic field in the xy-plane is given by the potential function φ = 3x2y – y3,
find the stream function and hence find complex potential. (G.B.T.U. 2011, 2013)
w

y
Sol. (i) ψ=– ...(1)
w

x + y2
2

∂ψ 2 xy ∂ψ y2 – x 2
= 2 , = 2
∂x (x + y2 )2 ∂y ( x + y 2 ) 2
∂φ ∂φ ∂ψ ∂ψ
We know that, dφ = dx + dy = dx – dy
∂x ∂y ∂y ∂x
( y2 − x 2 ) 2 xy
= dx − dy
(x2 + y2 )2 (x2 + y2 )2
( x 2 + y 2 ) dx − 2 x 2 dx − 2 xy dy
=
( x 2 + y2 )2

www.cgaspirants.com
www.cgaspirants.com

FUNCTION OF COMPLEX VARIABLE 27

( x 2 + y 2 ) d( x) − x(2 x dx + 2 y dy)
=
(x 2 + y2 )2
( x 2 + y 2 ) d( x) − xd ( x 2 + y 2 ) x F I
=
(x2 + y2 )2
=d GH
x 2 + y2
. JK
x
Integration yields, φ= 2 + c where c is a constant.
x + y2
(ii) Let ψ (x, y) be the stream function.
∂ψ ∂ψ ∂φ FG
∂φ IJ FG IJ
dψ =
∂x
dx +
∂y
dy = −
∂y
dx +
H
∂x
dy
K H K

m
= {– (3x2 – 3y2)} dx + 6xy dy
= – 3x2 dx + (3y2 dx + 6xy dy)

o
= – d (x3) + 3d (xy2)

.c
Integrating, we get ψ = – x3 + 3xy2 + c |c is a constant
Complex potential is given by

or,
n
w = –i[x3 – iy3 + 3ix2y – 3xy2 – c]
ts
w = φ + iψ = 3x2y – y3 + i(–x3 + 3xy2 + c)

or, w = –i [(x + iy)3 – c]


ra

⇒ w = –iz3 + c1 | where c1 = ic
Example 17. (i) If u = ex(x cos y – y sin y) is a harmonic function, find an analytic
pi

function f (z) = u + iv such that f (1) = e.


(ii) Determine an analytic function f(z) in terms of z whose real part is e–x(x sin y – y cos y).
as

[M.T.U. 2012, G.B.T.U. 2011, U.P.T.U. 2006, 2008, 2014]


Sol. (i) We have, x
u = e (x cos y – y sin y)
g

∂u
= ex(x cos y – y sin y) + ex cos y = φ1 (x, y) |say
.c

∂x
∂u
w

= ex [– x sin y – y cos y – sin y] = φ2 (x, y) |say


∂y
∴ φ1(z, 0) = ez z + ez = (z + 1) ez
w

φ2 (z, 0) = 0
w

By Milne’s Thomson method,


f(z) = z {φ 1 ( z, 0) − i φ 2 ( z, 0)} dz + c | c is a constant

= z( z + 1) e z dz + c = (z – 1) ez + ez + c = zez + c
f(1) = e + c
...(1)
|From (1)
e=e+c |f(1) = e (given)
⇒ c=0
∴ From (1), f(z) = zez.
(ii) u = e–x(x sin y – y cos y)
∂u
= e–x sin y – e–x (x sin y – y cos y) = φ1 (x, y) | say
∂x

www.cgaspirants.com
www.cgaspirants.com

28 A TEXTBOOK OF ENGINEERING MATHEMATICS

∂u
= e–x(x cos y – cos y + y sin y) = φ2(x, y) | say
∂y
∴ φ1 (z, 0) = 0 and φ2(z, 0) = e–z(z – 1)
By Milne’s Thomson method,

f(z) = zl q
φ 1 ( z, 0) − iφ 2 ( z, 0) dz + c

=–i zL e − z ( z − 1) dz + c

NM
= − i ( z − 1) (− e − z ) −

= – i [(1 – z) e–z – e–z] + c


z OP
(− e − z ) dz + c
Q

m
⇒ f(z) = ize–z + c | where c is a constant
Example 18. (i) Determine the analytic function whose real part is e2x (x cos 2y – y sin 2y).

o
(ii) Find an analytic function whose imaginary part is e–x(x cos y + y sin y).

.c
(U.P.T.U. 2009)
Sol. (i) Let f(z) = u + iv be the required analytic function.
Here,


∂u
∂x
u = e2x (x cos 2y – y sin 2y)
n ts
= e2x (2x cos 2y – 2y sin 2y + cos 2y) = φ1 (x, y) | say
ra
∂u
and = – e2x (2x sin 2y + sin 2y + 2y cos 2y) = φ2 (x, y) | say
∂y
Now, φ1 (z, 0) = e2z (2z + 1)
pi

φ2 (z, 0) = – e2z (0) = 0


By Milne’s Thomson method,
as

f(z) = z {φ 1 ( z, 0) − i φ 2 ( z, 0)} dz + c = z e 2 z (2 z + 1) dz + c

z
g

e2 z e2 z
= (2z + 1) − 2. dz + c |Integrating by parts
.c

2 2
e 2 z 1 2z
w

= (2z + 1) − e +c
2 2
f(z) = ze2z + c where c is an arbitrary constant.
w

(ii) Let f(z) = u + iv be the required analytic function.


w

Here v = e–x(x cos y + y sin y)


∂v
= e–x (– x sin y + y cos y + sin y) = ψ1(x, y) | say
∂y
∂v
= e–x cos y – e–x (x cos y + y sin y) = ψ2 (x, y) | say
∂x
∴ ψ1 (z, 0) = 0
ψ2(z, 0) = e–z – e–z z = (1 – z) e–z
By Milne’s Thomson method,

f(z) = z ψ 1 ( z, 0) + i ψ 2 ( z, 0) dz + c = i z (1 − z) e − z dz + c

www.cgaspirants.com
www.cgaspirants.com

FUNCTION OF COMPLEX VARIABLE 29

LM
= i (1 − z) (− e − z ) −
N z (− 1) (− e − z ) dz + c OP
Q
= i [(z – 1) e–z + e–z] + c
⇒ f(z) = ize–z + c
Example 19. (i) Let f(z) = u(r, θ) + iv(r, θ) be an analytic function. If u = – r3 sin 3θ, then
construct the corresponding analytic function f(z) interms of z.

FG 1IJ
(ii) Find the analytic function f(z) = u + iv, given that v = r −
H r K
sin θ ; r ≠ 0

Sol. (i) u = – r3 sin 3θ

m
∂u ∂u
= – 3r2 sin 3θ, = – 3r3 cos 3θ
∂r ∂θ

o
∂v ∂v 1 ∂u FG ∂u IJ FG IJ
we know that dv = dr + dθ = − H
dr + r dθ K H K

.c
∂r ∂θ r ∂θ ∂r
= (3r2 cos 3θ) dr – (3r3 sin 3θ) dθ

Integration yields,
dv = d (r3 cos 3θ)
n ts
v = r3 cos 3θ + c
ra

∴ f(z) = u + iv = – r3 sin 3θ + ir3 cos 3θ + ic


pi

= ir3 (cos 3θ + i sin 3θ) + c1 | c1 = ic


= i(reiθ)3 + c1
as

⇒ f(z) = iz3 + c1 |∵ z = reiθ

FG 1IJ sin θ
H rK
v= r−
g

(ii)
.c

∂v F 1I
= G 1 + J sin θ,
∂v 1 FG IJ
H rK = r−
H
cos θ
K
w

∂r 2 ∂θ r
we know that,
w

∂u ∂u 1 ∂v FG ∂v IJ FG IJ
du = dr + dθ =
r ∂θ H
dr + − r
∂r
dθK H K
w

∂r ∂θ

FG IJ cos θ dr – FG r + 1IJ sin θ dθ


1
= 1−
H r K 2 H rK
F1 I
du = d (r cos θ) + d G cos θJ

Hr K
F 1I
u = G r + J cos θ + c
Integration yields,
H rK
F 1I F 1I
f(z) = u + iv = G r + J cos θ + c + i G r − J sin θ

H rK H rK

www.cgaspirants.com
www.cgaspirants.com

30 A TEXTBOOK OF ENGINEERING MATHEMATICS

1 –iθ
= reiθ + e +c
r
1
⇒ f (z) = z +
+ c.
z
Example 20. If u – v = (x – y) (x2 + 4xy + y2) and f(z) = u + iv is an analytic function of
z = x + iy, find f(z) in terms of z.
Sol. Here, f(z) = u + iv
∴ if(z) = iu – v
Adding (1 + i) f(z) = (u – v) + i(u + v)
Let (1 + i) f(z) = F(z), u – v = U, u + v = V, then

m
F(z) = U + iV

o
Now, U = u – v = (x – y) (x2 + 4xy + y2)
∂U

.c
⇒ = x2 + 4xy + y2 + (x – y)(2x + 4y) = 3x2 + 6xy – 3y2 = φ1(x, y)
∂x
| say

and
∂U
∂y
n ts
= – (x2 + 4xy + y2) + (x – y)(4x + 2y) = 3x2 – 6xy – 3y2 = φ2(x, y)
| say
ra

Now, φ1(z, 0) = 3z2, φ2(z, 0) = 3z2


By Milne’s Thomson method,

z
pi

F(z) = z [φ 1 ( z, 0) – iφ 2 ( z, 0)] dz + c = [3 z 2 − i(3 z 2 )] dz + c


as

F(z) = (1 – i) z3 + c
⇒ (1 + i) f(z) = (1 – i) z3 + c
g

FG 1 − i IJ z c FG IJ
− 2i 3 FG where c c IJ
.c

3
or, f(z) =
H 1 + iK +
1+ i
=
H K
2
z + c1
H 1 =
1+ i K
w

or, f(z) = – iz3 + c1.


2 sin 2x
w

Example 21. If u + v = 2y
and f(z) = u + iv is an analytic function of
e + e −2y − 2 cos 2x
w

z = x + iy, find f(z) in terms of z.


Sol. Let f(z) = u + iv ...(1)
Multiplying both sides by i
i f(z) = iu – v ...(2)
Adding (1) and (2), we get
(1 + i) f(z) = (u – v) + i(u + v) ...(3)
⇒ F(z) = U + iV ...(4)
where F(z) = (1 + i) f(z) ...(5)
U=u–v and V=u+v ...(6)

www.cgaspirants.com
www.cgaspirants.com

FUNCTION OF COMPLEX VARIABLE 31

It means that we have been given

sin 2 x
V= ...(7) | ∵ e2y + e–2y = 2 cosh 2y
cosh 2 y − cos 2 x

∂V − 2 sin 2 x sinh 2 y
Now = = ψ1(x, y) | say
∂y (cosh 2 y − cos 2 x) 2

∂V 2 cos 2 x (cosh 2 y − cos 2 x) − 2 sin 2 2 x


and =
∂x (cosh 2 y − cos 2 x) 2

2 cos 2 x cosh 2 y − 2

m
= = ψ2(x, y) | say
(cosh 2 y − cos 2 x) 2

o
∴ ψ1(z, 0) = 0

.c
2(cos 2 z − 1) −2 −2
ψ2(z, 0) = 2
= = = – cosec2 z
(1 − cos 2 z) 1 − cos 2 z 1 − 1 + 2 sin 2 z
By Milne’s Thomson method, we have
n ts
F(z) = ∫ {ψ1(z, 0) + i ψ2(z, 0)} dz + c
ra

= ∫ – i cosec2 z dz + c = i cot z + c
Replacing F(z) by (1 + i) f(z), from eqn. (5), we get
pi

(1 + i) f(z) = i cot z + c
i c
as

⇒ f(z) = cot z +
1+ i 1+ i
1
c
∴ f(z) = (1 + i) cot z + c1, where c1 = .
g

2 1+ i
.c

cos x + sin x − e − y
Example 22. If f(z) = u + iv is an analytic function of z and u – v = ,
2 cos x − 2 cosh y
w

1 LM z OP π FG IJ
prove that f(z) =
2
1 − cot
N 2 Q
when f
2 H K
= 0.
w

Sol. Let f(z) = u + iv ...(1)


w

∴ i f(z) = iu – v
Add, (1 + i) f(z) = (u – v) + i(u + v) ...(2)
⇒ F(z) = U + iV ...(3)
where u – v = U, u + v = V and (1 + i) f(z) = F(z).

cos x + sin x − e − y
We have, u–v=
2 cos x − 2 cosh y
cos x + sin x − cosh y + sinh y
or, U= [∵ e–y = cosh y – sinh y]
2 cos x − 2 cosh y
1 sin x + sinh y
= + ...(4)
2 2(cos x − cosh y)

www.cgaspirants.com
www.cgaspirants.com

32 A TEXTBOOK OF ENGINEERING MATHEMATICS

Diff. (4) w.r.t. x partially, we get


LM
∂U 1 (cos x − cosh y) cos x − (sin x + sinh y)(− sin x)
=
OP
∂x 2 N (cos x − cosh y) 2 Q
φ1(x, y) =
LM OP
1 1 − cosh y cos x + sinh y sin x
N2 (cos x − cosh y) 2 Q
1 L 1 − cos z O
φ (z, 0) = M
1 P= 1 .
2 N (cos z − 1) Q 2(1 − cos z) 2 ...(5)

Diff. (4) partially w.r.t. y, we get

m
LM
∂U 1 (cos x − cosh y) . cosh y − (sin x + sinh y)(− sinh y)
=
OP
∂y 2 N (cos x − cosh y) 2 Q

o
1 L cos x cosh y + sin x sinh y − 1 O

.c
φ (x, y) = M PQ
2
2N (cos x − cosh y) 2

∴ 2
1 L cos z − 1 O
φ (z, 0) = M P
2 N (cos z − 1) Q
1 F −1 I
= .G J.
2 H 1 − cos z K
n
2
ts ...(6)
ra

By Milne’s Thomson Method,


F(z) = zL [φ 1 ( z, 0) − i φ 2 ( z, 0)] dz + c
pi

= z MN 1
.
1 i
+ .
1
dz + c
OP
Q
as

2 (1 − cos z) 2 1 − cos z

=
1+ i 1
z
dz + c =
1+ i
cosec 2 ( z/2) dz + c z
g

2
2 2 sin z/2 4
FG IJ FG IJ
.c

1 + i (− cot z/2) 1+ i z
=
H
4
.
1 K
+c =–
FG IJ
2
cot + c
2 H K
H K
w

2
1+ i FG z IJ
w

or, (1 + i) f(z) = –
2 H
cot + c
2 K
1 z c
w

⇒ f(z) = – cot + ...(7)


2 2 1+ i
FG π IJ = − 1 cot π + c
f
H 2K 2 4 1 + i [From (7)]

1 c c 1
0=– + ⇒ = ...(8)
2 1+ i 1+ i 2

1 z 1 1 FG
z IJ
∴ From (7), f(z) = –
2
cot + =
2 2 2
1 − cot
2H.
K [Using (8)]

www.cgaspirants.com
www.cgaspirants.com

FUNCTION OF COMPLEX VARIABLE 33

Example 23. (i) If f(z) is a regular function of z, prove that


F∂ 2
∂2 I |f(z)|
GH ∂x 2
+
∂y 2 JK 2 = 4 | f ′(z)|2. (U.P.T.U. 2007, 2015)

(ii) If f(z) is a harmonic function of z, show that

RS ∂ |f(z)|UV + RS ∂ |f(z)|UV
2 2
= f ′ (z) 2
(U.P.T.U. 2009)
T ∂x W T ∂y W
Sol. (i) Let f(z) = u + iv so that |f(z)| = u 2 + v2
or |f(z)|2 = u2 + v2 = φ(x, y) (say)

m
∂φ ∂u ∂v
∴ = 2u + 2v
∂x ∂x ∂x

o
∂2φ LM
∂ 2u ∂u FG IJ 2
FG IJ
∂ 2v ∂v
2 OP

.c
∂x 2
=2 u 2 +
MN
∂x ∂x H K +v
H K
∂x 2
+
∂x PQ
L ∂ u F ∂u I OP
Similarly,
∂ φ
∂y
2
= 2 Mu
MN ∂y + GH ∂y JK
2
n 2

2
2
+v
∂y
+G J ts
∂ v F ∂v I
2

H ∂y K 2
2

PQ
Adding, we get
ra

∂2φ ∂2φ LM F
∂ 2u ∂ 2u ∂u I FG IJ + F ∂u I 2 2
F ∂ v + ∂ v I + FG ∂v IJ + F ∂v I OP
2 2 2 2
+ = 2 u
MN GH+ 2 + JK H K GH ∂y JK H ∂x ∂y JK H ∂x K GH ∂y JK PQ
+ vG ...(1)
pi

2 2 2 2 2
∂x ∂y ∂x ∂y ∂x

Since f(z) = u + iv is a regular function of z, u and v satisfy C-R equations and Laplace’s
as

equation.
∂u ∂v ∂u ∂v ∂ 2u ∂ 2u ∂ 2v ∂ 2v
= , =− and + = 0 = +
g


∂x ∂y ∂y ∂x ∂x 2 ∂y 2 ∂x 2 ∂y 2
.c

∴ From (1), we get


LM F IJ + FG ∂vIJ FG ∂vIJ + FG ∂u IJ OP
w

2 2 2 2
∂2φ ∂2φ ∂u
MN GH K H ∂x K H ∂x K H ∂x K PQ
2
+ 2 =2 0+ +0+
∂x ∂y ∂x
w

LF ∂u I 2
F ∂vI O 2
= 4 MG J +G J P
w

MNH ∂x K H ∂x K PQ ...(2)

Now f(z) = u + iv

∂u ∂v
|f ′(z)|2 =
FG ∂u IJ + FG ∂v IJ
2 2
∴ f ′(z) =
∂x
+i
∂x
and
H ∂x K H ∂x K
From (2), we get
F∂ 2
∂2 I φ = 4 |f ′(z)| F∂ 2
∂2 I |f(z) |
GH ∂x 2
+
∂y 2
JK 2 or GH ∂x 2
+
∂y 2
JK 2 = 4 |f ′(z)|2.

www.cgaspirants.com
www.cgaspirants.com

34 A TEXTBOOK OF ENGINEERING MATHEMATICS

(ii) We have, f(z) = u + iv ...(1)

∴ |f(z)| = u 2 + v2 ...(2)
Partially differentiating eqn. (2) w.r.t. x and y, we get
∂u ∂v
∂ 1 2 2 −1/2 ∂u ∂vFG IJ u
∂x
+v
∂x
∂x
|f ( z)| = (u + v )
2
2u
∂x
+ 2v
∂x H
=
K |f ( z)|
...(3)

∂u ∂v
u +v
∂ ∂y ∂y
Similarly, |f ( z)| = ...(4)
∂y |f ( z)|

m
Squaring and adding (3) and (4), we get

FG u ∂u + v ∂v IJ + FG u ∂u + v ∂vIJ
2 2

o
RS |f (z)|UV + S |f (z)|V = H ∂x ∂x K H ∂y ∂y K

2
R ∂ U 2

.c
T ∂x W T ∂y W |f ( z)| 2

FG u ∂u + v ∂vIJ + FG − u ∂v + v ∂u IJ
H ∂x ∂x K H ∂x ∂x K
=
n 2

|f ( z)|2
ts 2

| Using C-R eqns.


ra

(u 2 + v2 )
LMFG ∂u IJ + FG ∂v IJ OP
2 2

MNH ∂x K H ∂x K PQ
pi

=
|f ( z)|2
as

FG ∂u IJ + FG ∂v IJ
2 2
=
H ∂x K H ∂x K |∵ |f(z)|2 = u2 + v2
g

∂u ∂v
= |f ′(z)|2 ∵ f ′ ( z) = +i
.c

∂x ∂x
w

ASSIGNMENT
w

1. (i) Determine a, b, c, d so that the function f(z) = (x2 + axy + by2) + i(cx2 + dxy + y2) is analytic.
(ii) Find the constants a, b, c such that the function f(z) where
w

f(z) = – x2 + xy + y2 + i (ax2 + bxy + cy2) is analytic. Express f(z) in terms of z.


(M.T.U. 2013)
(iii) Find the value of the constants a and b such that the following function f(z) is analytic.
f(z) = cos x (cosh y + a sinh y) + i sin x (cosh y + b sinh y)
1 px
(iv) Determine p such that the function f(z) = log (x2 + y2) + i tan–1 is an analytic function.
2 y
Also find f ′(z). (M.T.U. 2012)
2. Show that
(a) f(z) = xy + iy is everywhere continuous but is not analytic.
(b) f(z) = z + 2 z is not analytic anywhere in the complex plane.
(c) f(z) = z | z | is not analytic anywhere. (U.K.T.U. 2010)

www.cgaspirants.com
www.cgaspirants.com

FUNCTION OF COMPLEX VARIABLE 35

3. Discuss the analyticity of the following functions:


1
(i) sin z (ii) cosh z (iii) (iv) z3.
z
4. (i) Define analytic function. Discuss the analyticity and differentiability of f(z) = | z |4 at z = 0.
(G.B.T.U. 2012)
(ii) Define anlaytic function. Discuss the analyticity of f(z) = Re (z3) in the complex plane.
(U.P.T.U. 2014)
∂u 1 ∂v ∂v 1 ∂u
5. Show that the polar form of Cauchy-Riemann equations are = , =− . Deduce
∂r r ∂θ ∂r r ∂θ
∂ 2u 1 ∂u 1 ∂ 2u
that 2
++ 2 = 0.
∂r r ∂r r ∂θ2

m
6. (i) Show that an analytic function f(z), whose derivative is identically zero, is constant.
(ii) It is given that a function f(z) and its conjugate f ( z) are both analytic. Determine the function

o
f(z).
(iii) Show that if f(z) is analytic and Im f (z) = constant then f (z) is a constant.

.c
ux uy
(iv) Show that if f(z) is differentiable at a point z, then |f ′(z)|2 = v vy

ts
x

x3 y5 ( x + iy)
7. (i) Show that the function f(z) defined by f(z) = , z ≠ 0, f(0) = 0, is not analytic at the
n
x6 + y10
origin even though it satisfies Cauchy-Riemann equations at the origin. (G.B.T.U. 2011)
ra

(ii) Show that for the function


R| (z) ,
2
pi

f(z) = S| z z≠0
T 0, z=0
as

the Cauchy-Riemann equations are satisfied at the origin. Does f ′(0) exist?
(iii) Show that for the function
R| 2 xy (x + iy) ,
g

z≠0
f(z) = S| x + y
2 2
.c

T 0, z=0
w

the C-R equations are satisfied at origin but derivative of f(z) does not exist at origin.
8. (i) If u is a harmonic function then show that w = u2 is not a harmonic function unless u is a
w

constant.
(ii) If f(z) is an analytic function, show that |f (z)| is not a harmonic function.
w

9. (i) Show that the function u (x, y) = 2x + y3 – 3x2y is harmonic. Find its conjugate harmonic
function v(x, y) and the corresponding analytic function f(z).
(ii) Show that the function v(x, y) = ex sin y is harmonic. Find its conjugate harmonic function
u(x, y) and the corresponding analytic function f(z).
(iii) Define a harmonic function and conjugate harmonic function. Find the harmonic conjugate of
the function u(x, y) = 2x (1 – y). (U.P.T.U. 2009)
(iv) Show that the function u = e –2xy 2 2
sin (x – y ) is harmonic. (U.K.T.U. 2011)
(v) Show that u(x, y) = x3 – 4xy – 3xy2 is harmonic. Find its harmonic conjugate v(x, y) and the
corresponding analytic function f(z) = u + iv. (G.B.T.U. 2013)
10. (i) Show that the function u(r, θ) = r2 cos 2θ is harmonic. Find its conjugate harmonic function
and the corresponding analytic function f(z).
(ii) Determine constant ‘b’ such that u = ebx cos 5y is harmonic.

www.cgaspirants.com
www.cgaspirants.com

36 A TEXTBOOK OF ENGINEERING MATHEMATICS

(iii) Define Harmonic function. Show that the function v = log (x2 + y2) + x – 2y is harmonic. Also
find the analytic function f(z) = u + iv. (G.B.T.U. 2012)
(iv) Show that v(x, y) = e–x (x cos y + y sin y) is harmonic. Find its harmonic conjugate.
(U.P.T.U. 2014)
11. Determine the analytic function f(z) in terms of z whose real part is
1
(i) log (x2 + y2) (U.K.T.U. 2011) (ii) cos x cosh y
2
(iii) e–x (x cos y + y sin y) ; f (0) = 1 (iv) (x – y)(x2 + 4xy + y2) (G.B.T.U. 2012)
sin 2 x sin 2 x
(v) (vi) .
cosh 2 y − cos 2 x cosh 2 y + cos 2 x
12. Find the regular function f(z) in terms of z whose imaginary part is
x− y

m
(i) 2 (ii) cos x cosh y (iii) sinh x cos y
x + y2
x

o
(iv) 6xy – 5x + 3 (v) 2 + cosh x cos y. (vi) ex (x sin y + y cos y)
x + y2
(U.P.T.U. 2015)

.c
13. Prove that u = x2 – y2 – 2xy – 2x + 3y is harmonic. Find a function v such that f(z) = u + iv is
analytic. Also express f(z) in terms of z.
14.
stream function.
n ts
(i) An electrostatic field in the xy-plane is given by the potential function φ = x2 – y2, find the

(ii) If the potential function is log (x2 + y2), find the flux function and the complex potential
ra
function.

15. (i) In a two dimensional fluid flow, the stream function is ψ = tan–1
FG y IJ , find the velocity
H xK
pi

potential φ.
x
(ii) If w = φ + iψ represents the complex potential for an electric field and ψ = x2 – y 2 + 2 ,
as

determine the function φ.


x + y2
(iii) If u = (x – 1)3 – 3xy2 + 3y2, determine v so that u + iv is a regular function of x + iy.
g

[U.K.T.U. 2010]
F∂ 2
∂2 I |Re f(z)|
.c

16. If f(z) is an analytic function of z, prove that GH ∂x 2


+
∂y 2 JK 2 = 2 |f ′(z)|2.
w

(G.B.T.U. 2012)
17. Find an analytic function f(z) = u(r, θ) + iv(r, θ) such that v(r, θ) = r2 cos 2θ – r cos θ + 2.
w

18. If f(z) = u + iv is an analytic function, find f(z) in terms of z if


x
w

(i) u – v = ex (cos y – sin y) (ii) u + v = , when f(1) = 1


x 2 + y2
[U.P.T.U. (C.O.) 2008]
y
e − cos x + sin x π FG IJ
3−i
(iii) u – v =
cosh y − cos x
when f
2
=
H K
2
.

19. (i) If f(z) = u + iv is an analytic function of z = x + iy and u + v = (x + y) (2 – 4xy + x2 + y2), then


construct f(z) in terms of z.
(ii) If f(z) = u + iv is an analytic function of z = x + iy and u – v = e –x [(x – y) sin y – (x + y) cos y], then
construct f(z) in terms of z. [U.P.T.U. (C.O.) 2009]
20. If f = u + iv is analytic show that g = – v + iu is also analytic. Also show that u and – v are
conjugate harmonic.

www.cgaspirants.com
www.cgaspirants.com

FUNCTION OF COMPLEX VARIABLE 37

21. Show that the function


z
(i) f (z) = is analytic at z = ∞. (ii) f(z) = z is not analytic at z = ∞.
z+1

z+z z−z
22. If f (z) = u(x, y) + iv (x, y) where x = ,y= is continuous as a function of two variables z
2 2i
∂f
and z then show that = 0 is equivalent to the Cauchy-Riemann equations.
∂z
LMHint. ∂f = F ∂u ∂x + ∂u ∂y I + i F ∂v ∂x + ∂v ∂y I OP
MN ∂z GH ∂x ∂z ∂y ∂z JK GH ∂x ∂z ∂y ∂z JK PQ

m
∂ 2u
23. (i) Show that a harmonic function satisfies the formal differential equation =0
∂z ∂ z

o
F∂ 2
∂2 I log |f ′(z)| = 0. Further, if |f ′(z)|
(ii) If w = f(z) is a regular function of z, prove that GH ∂x + JK

.c
2 2
∂y
is the product of a function of x and function of y, show that f ′(z) = exp. (αz2 + βz + γ) where α
is real and β, γ are complex constants.

(i) 3u + v = 3 sin x cosh y + cos x sinh y


n ts
24. If f (z) = u + iv is an analytic function of z = x + iy, find f (z) in terms of z if
(ii) u – 2v = cos x cosh y + 2 sin x sinh y
(iii) 2u – v = ex (2 cos y – sin y)
ra

25. (i) If f ′(z) = f(z) for all z, then show that f(z) = ke z, where k is an arbitrary constant.
(ii) Find an analytic function f(z) such that Re [f ′(z)] = 3x2 – 4y – 3y2 and f(1 + i) = 0
pi

(iii) Let f(z) = u + iv and g(z) = v + iu be analytic functions for all z. Let f(0) = 1 and g(0) = i. Obtain
the value of h(z) at z = 1 + i where h(z) = f ′(z) + g′(z) + 2f(z) g(z).
as

(iv) If f(z) = u + iv is an analytic function of z and φ is a function of u and v, then show that

FG ∂φ IJ + FG ∂φ IJ = LMFG ∂φ IJ + FG ∂φ IJ OP|f ′ (z)|


2 2 2 2
2
H ∂x K H ∂y K MNH ∂u K H ∂v K PQ
g
.c

Answers
1 1 1
w

1. (i) a = 2, b = – 1, c = – 1, d = 2 (ii) a = − , b = – 2, c = ; f(z) = − (2 + i) z2


2 2 2
1
w

(iii) a = – 1, b = – 1 (iv) p = – 1, f ′(z) =


z
6. (ii) constant function 7. (ii) No
w

9. (i) v = 2y – 3xy2 + x3 + c ; f(z) = 2z + iz3 + ic (ii) u = ex cos y + c ; f(z) = ez + c


(iii) v(x, y) = x2 – y2 + 2y + c (v) v(x, y) = 2x2 – 2y2 + 3x2y – y3 + c, f(z) = z3 + 2iz2 + c
10. (i) v = r2 sin 2θ + c ; f(z) = z2 + ic (ii) b = ± 5
(iii) f(z) = – 2z + i(2 log z + z) + c (iv) u(x, y) = e–x (x sin y – y cos y) + c
11. (i) log z + c (ii) cos z + c (iii) 1 + ze–z
(iv) (1 – i) z3 + c (v) cot z + c (vi) tan z + c
1+ i
12. (i) +c (ii) i cos z + c (iii) i sinh z + c
z
i
(iv) 3z2 – 5iz + c (v) + i cosh z + c (vi) z ez + c
z

www.cgaspirants.com
www.cgaspirants.com

38 A TEXTBOOK OF ENGINEERING MATHEMATICS

13. v = x2 – y2 + 2xy – 2y – 3x + c, f(z) = (1 + i)z2 – (2 + 3i)z + ic


FG y IJ + c, 2 log z + ic
14. (i) ψ = 2xy + c (ii) 2 tan–1
H xK
1 y
15. (i) log (x2 + y 2) + c (ii) – 2xy + +c (iii) v = 3y (1 + x2) – y3
2 x + y2
2

17. i(z2 – z + 2) + c
1 FG i + 1IJ z 1
18. (i) ez + c (ii)
1+ i Hz K (iii) cot + (1 – i)
2 2
21. (i) 2z + iz3 + c (ii) ize–z + c
24. (i) f (z) = sin z + c (ii) f (z) = cos z + c (iii) f (z) = ez + c

m
25. (ii) f(z) = z3 + 2iz2 + 6 – 2i (iii) 2i

1.19 LINE INTEGRAL IN THE COMPLEX PLANE

o
z
.c
b
In case of real variable, the path of integration of f ( x) dx is always along the x-axis from
a

x = a to x = b. But in case of a complex function f(z), the path of the definite integral
can be along any curve from z = a to z = b.
n ts za
b
f ( z) dz
ra
Let f(z) be a continuous function of the complex variable z = x + iy defined at all points
of a curve C having end points A and B. Divide the curve C into n parts at the points
A = P0(z0), P1(z1), ......, Pi(zi), ......, Pn(zn) = B.
pi

Let δzi = zi – zi–1 and ξi be any point on the arc Pi–1 Pi. Then the limit of the sum
n

∑ f (ξ ) δz
as

i i as n → ∞ and each δzi → 0, if it exists, is called the line integral of f(z)

z
i=1
g

along the curve C. It is denoted by f ( z) dz .


C
.c

In case the points P0 and Pn coincide so that


Y Pn = B
C is a closed curve, then this integral is called

z
w

Pi +1
contour integral and is denoted by f ( z) dz .
C
w

If f(z) = u(x, y) + iv(x, y), then since dz = dx + i dy, Pi


we have

z z
w

Pi – 1
f ( z) dz = (u + iv)(dx + i dy) P2
C C P1

= z
C
(udx − vdy) + i z
C
which shows that the evaluation of the line integral
(vdx + udy)
P0 = A

O X
of a complex function can be reduced to the evalua-
tion of two line integrals of real functions.
Moreover, the value of the integral depends on the path of integration unless the
integrand is analytic.

www.cgaspirants.com
www.cgaspirants.com

FUNCTION OF COMPLEX VARIABLE 39

When the same path of integration is used in each integral, then

za
b
f ( z) dz = −

If c is a point on the arc joining a and b, then


z
b
a
f ( z) dz

za
b
f ( z) dz = z c

a
f ( z) dz + z
c
b
f ( z) dz .

EXAMPLES

z 1+ i

m
Example 1. Evaluate (x − y + ix 2 ) dz .
0
(a) along the straight line from z = 0 to z = 1 + i

o
(b) along the real axis from z = 0 to z = 1 and then along a line parallel to imaginary axis

.c
from z = 1 to z = 1 + i.
(c) along the imaginary axis from z = 0 to z = i and then along a line parallel to real axis
from z = i to z = 1 + i.
Sol. (a) Along the straight line OP joining O(z = 0) and P(z = 1 + i), y = x, dy = dx and x
varies from 0 to 1.
n ts
z z
ra
1+ i 1+ i
∴ ( x − y + ix 2 ) dz = (x – y + ix2)(dx + i dy)
0 0

z z
pi

1 1
= ( x − x + ix 2 )(dx + idx) = ix 2 (1 + i) dx
0 0
as

Fx I 3 1
1 1
= (i – 1) G J =− + i.
H 3K 0
3 3
g

(b) Along the path OAP where A is z = 1


.c

z 1+ i
( x − y + ix 2 ) dz
w

z ( x − y + ix 2 ) dz + z
w

= (x – y + ix2) dz ...(1)
OA AP

Now along OA, y = 0, dz = dx and x varies from 0 to 1.


w

z z LM x OP
2 1
1 x3 1 1
∴ ( x − y + ix 2 ) dz = ( x + ix 2 ) dx = +i = + i
OA 0
N2 3 Q 0
2 3
Also along AP, x = 1, dz = idy and y varies from 0 to 1

∴ zAP
2 L
N z0
1
( x − y + ix ) dz = (1 − y + i) idy = M(− 1 + i) y − i
y O
2
P
Q
1 1
=–1+i– i=–1+ i
2 2
2 1

Hence from (1),


z 0
F1 1 I F 1 I 1 5
1+ i
( x − y + ix ) dz = G + iJ + G − 1 + iJ = − + i .
H2 3 K H 2 K 2 6
2

www.cgaspirants.com
www.cgaspirants.com

40 A TEXTBOOK OF ENGINEERING MATHEMATICS

(c) Along the path OBP where B is z = i

z 0
1+ i
( x − y + ix 2 ) dz =

Now along OB, x = 0, dz = idy and y varies from 0 to 1


z OB
( x − y + ix 2 ) dz + zBP
(x – y + ix2) dz ...(2)

∴ zOB
( x − y + ix 2 ) dz = z 1

0
(− y) idy = − i
LM y OP
2 1

N2Q 0
=−
1
2
i

Also, along BP, y = 1, dz = dx and x varies from 0 to 1

∴ z ( x − y + ix 2 ) dz = z LM x − x + i x OP
1

N2
( x − 1 + ix 2 ) dx =
3Q
2 3 1
=−
1 1
+ i
2 3

m
BP 0
0

Hence from (2), z 1+ i


2
1 F 1 1I 1 1
(x – y + ix ) dz = – i + G − + iJ = − − i .
H 2 3K 2 6

o
0 2

.c
Note. The values of the integral are different along the three different paths.

Example 2. Evaluate

(a) y = x
z
(b) y = x2.
0
1+ i
n (x 2 − iy) dz along the paths
ts (G.B.T.U. 2010)
Sol. (a) Along the line y = x,
ra

dy = dx so that dz = dx + idx = (1 + i) dx

z 1+ i
( x 2 − iy) dz = z 1
( x 2 − ix)(1 + i) dx
pi

0 0

Lx
= (1 + i) M
3
x2 OP 1
FG 1 − 1 iIJ = 5 − 1 i .
as

N3
−i
2 Q 0
= (1 + i)
H3 2 K 6 6
(b) Along the parabola y = x2, dy = 2x dx so that
g

Y
dz = dx + 2ix dx = (1 + 2ix) dx
.c

and x varies from 0 to 1.

z z
w

1+ i 1 P (1, 1)
∴ ( x 2 − iy) dz = ( x 2 − ix 2 ) (1 + 2ix) dx
0 0
w

L x + i x OP 4 1
x

3
=

= (1 − i) M
w

N3 2Q
2
x

0
=
y

F1 1 I 5 1
= (1 – i) GH + iJK = + i . O
3 2 6 6 X

Example 3. Evaluate z0
2+i
(z) 2 dz , along

(a) the real axis from z = 0 to z = 2 and then along a line parallel to y-axis from z = 2 to
z = 2 + i. (U.P.T.U. 2009, U.K.T.U. 2011)
(b) along the line 2y = x. (U.P.T.U. 2009)

www.cgaspirants.com
www.cgaspirants.com

FUNCTION OF COMPLEX VARIABLE 41

Sol. ( z )2 = (x – iy)2 = (x2 – y2) – 2ixy Y


(a) Along the path OAP where A is (2, 0) and P is
(2, 1).

z
P (2, 1)
2+ i
2
( z ) dz

z z
0 x
=
= ( x 2 − y 2 − 2ixy) dz + ( x 2 − y 2 − 2ixy) dz 2y
OA AP
...(1)
Now, along OA, y = 0, dz = dx and x varies from 0
to 2 O A (2, 0) X

z z 2 LM x OP 3 2
8

m
∴ ( x 2 − y 2 − 2ixy) dz = x 2 dx = =
OA 0
N3Q 0
3
Also, along AP, x = 2, dz = idy and y varies from 0 to 1

o
z zL 1

.c
∴ ( x 2 − y 2 − 2ixy) dz = (4 – y2 – 4iy) idy
AP 0

y3 OP 1

MN
= 4iy − i
3
+ 2 y2
Q ts
= 4i −
1
3
i+2=2+
11
3
i

z
0
n
8 11 14 11 2+i
2
Hence from (1), we have ( z ) dz = + 2 + i= + i.
ra
0 3 3 3 3
(b) Along the line OP, 2y = x, dx = 2dy
so that dz = 2dy + i dy = (2 + i) dy
pi

and y varies from 0 to 1.

z 2+ i
( z ) 2 dz = z 2+ i
( x 2 − y 2 − 2ixy) dz = z 1
(4 y 2 − y 2 − 4iy 2 ) (2 + i) dy
as


0 0 0

= (2 + i)(3 – 4i) z 1
y 2 dy = (10 − 5i)
LM y OP
3 1
10 5
− i.
g

=
0
N3Q 0
3 3
.c

Example 4. Integrate f(z) = x2 + ixy from A(1, 1) to B(2, 4) along the curve x = t, y = t2.
Sol. Equations of the path of integration are x = t, y = t2
w

∴ dx = dt, dy = 2t dt
At A(1, 1), t = 1 and at B(2, 4), t = 2
w

∴ z B
f ( z) dz = z B
( x 2 + ixy)(dx + idy) = z 2
(t2 + it3)(dt + 2it dt)
w

A A 1

=
z1
2
(t − 2t ) dt + i 3t dt = M −
2 4
z
L t 2t OP + i LM 3t OP
N3 5 Q N 4 Q
1
2
3
3 5 2

1
4 2

1
F 8 64 IJ − FG 1 − 2 IJ + i FG 12 − 3 IJ = − 151 + 45 i .
=G −
H 3 5 K H 3 5K H 4 K 5 4
Example 5. Prove that

(i)
z dz
z−a
= 2πi

z
C

(ii) (z − a) n dz = 0 [n is an integer ≠ –1] where C is the circle | z – a | = r.


C
(G.B.T.U. 2011)

www.cgaspirants.com
www.cgaspirants.com

42 A TEXTBOOK OF ENGINEERING MATHEMATICS

Sol. The equation of the circle C is


|z–a|=r or z – a = reiθθ
where θ varies from 0 to 2π as z describes C once in the anti-clockwise direction.
Also dz = ireiθ dθ.

(i)
zC
dz
z−a
= z0
2π ire iθ dθ
re iθ
=i z 2π

0
dθ = 2πi

(ii)
z
C
( z − a) n dz = z0

r n e niθ . ire iθ dθ

= ir
n+1
z 2π
e i( n + 1)θ dθ

m
0

LM e OP
i( n + 1)θ 2π

o
= irn+1 [∵ n ≠ – 1]
N i(n + 1) Q 0

.c
r n+ 1 i2(n+1)π
= [e – 1]

= 0.
n+1
[∵ ts
ei2(n+1)π = cos 2(n + 1)π + i sin 2(n + 1)π = 1]

z
n
Example 6. Evaluate the integral |z|dz , where c is the contour
ra
c
(i) The straight line from z = – i to z = i
(ii) The left half of the unit circle | z | = 1 from z = – i to z = i.
pi

Sol. (i) The straight line from z = – i to z = i is x = 0


i.e., z = iy so that dz = idy

z z z z
as

1 0 1
∴ |z|dz = |iy|i dy = i (− y) dy + i y dy
c −1 −1 0
g

Fy I
=–iG J
2 0
Fy I
+iG J
2 1
FG 1IJ + i FG 1IJ = i.
=–i −
H 2K H 2K
.c

H 2K −1
H 2K 0
(ii) For a point on the unit circle | z | = 1,
w

z = eiθ
w

∴ dz = ieiθ dθ.
3π π
The points z = – i and i correspond to θ = and θ = respectively.
w

2 2

z z F I π/2
idθ = GH e JK
π/2 iθ

∴ | z| dz = 1. e = eiπ/2 – e3iπ/2
c 3π / 2 3π / 2

π π 3π 3π
= cos + i sin − cos − i sin = 0 + i – 0 – i(– 1) = 2i.
2 2 2 2
Example 7. Evaluate the integral
Sol. Here, c ≡ |z| = 1
z c
log z dz , where c is the unit circle | z | = 1.
...(1)

z c
log zdz = z c
log ( x + iy) dz

www.cgaspirants.com
www.cgaspirants.com

FUNCTION OF COMPLEX VARIABLE 43

= z LMN
c
1
2
log ( x 2 + y 2 ) + i tan −1
y
x
dz
OP
Q

On the unit circle,


=i

z = eiθ
z c
tan −1
FG y IJ dz.
H xK ...(2) (∵ x2 + y2 = 1)

∴ dz = ieiθ dθ.
Now (2) becomes,

z c
log z dz = i z 2π

0
tan–1 (tan θ) ieiθ dθ = –
z
0

θ e iθ d θ

LF e I OP LM F I OP
z
2π 2π

m
= – MG θ
iθ 2π e iθ 2π 2 πi 1 e iθ
MNH i JK − 0
0
1.
i
dθ = –
PQ i
e −
MN
i i GH JK P
0Q

o
=– M
L 2π e + e OP

.c
2 πi 2 πi
− 1 = 2πie2πi + 1 – e2πi = 2πi |∵ e2πi = 1
Ni Q
n ASSIGNMENT ts
z
ra
3+ i
1. Evaluate z2 dz , along
0
pi

x
(a) the line y = (b) the real axis to 3 and then vertically to 3 + i
3
as

(c) the parabola x = 3y2.

2. Find the value of the integral


z 1+ i
( x − y − ix2 ) dz , along real axis from z = 0 to z = 1 and then
g

0
along a line parallel to imaginary axis from z = 1 to z = 1 + i. [G.B.T.U. (C.O.) 2011]
.c

3. Evaluate
z 4 + 2i
z dz along the curve given by z = t2 + it.
w

4. (i) Evaluate
z | z |2 dz around the square with vertices at (0, 0), (1, 0), (1, 1) and (0, 1).
w

z
w

(ii) Show that ( z + 1) dz = 0 where C is the boundary of the square whose vertices are at the
C
points z = 0, z = 1, z = 1 + i and z = i.

5. (a) Evaluate
z C
( x + y) dx + x 2 y dy

(i) along y = x2 having (0, 0), (3, 9) as end points.


(ii) along y = 3x between the same points.

(b) Evaluate

(i) along y = x2
z (1, 1)

(0, 0)
(3 x2 + 4 xy + 3 y2 ) dx + 2(x2 + 3xy + 4y2)dy

(ii) along y = x
Does the value of the integral depend upon the path?

www.cgaspirants.com
www.cgaspirants.com

44 A TEXTBOOK OF ENGINEERING MATHEMATICS

6. (i) Evaluate
z C
( y − x − 3 x 2 i) dz where C is the straight line from z = 0 to z = 1 + i.

(ii) Evaluate
z 2 + 3i

1− i
( z2 + z) dz along the line joining the points (1, – 1) and (2, 3).

7. (i) Evaluate
z C
( z − z2 ) dz where C is the upper half of the circle | z | = 1. What is the value of

this integral if C is lower half of the given circle?

(ii) Evaluate the integral


zC
( z − z2 ) dz where C is the upper half of the circle | z – 2 | = 3. What is

the value of the integral if C is the lower half of the circle? (M.T.U. 2013)

m
[Hint: z = 2 + 3eiθ]

8. Prove that
1
z
dz = − πi or πi according as C is the semi-circular arc | z | = 1 from z = – 1

o
C z
to z = 1 above or below the real axis.

.c
9. Evaluate z 2+ i
(2 x + iy + 1) dz along
1− i
(a) the straight line joining (1 – i) to (2 + i) ts
(b) the curve x = t + 1, y = 2t2 – 1.

z
n
10. Evaluate the line integral (3 y2 dx + 2 y dy) where C is the circle x2 + y2 = 1 counter clockwise
ra
C
from (1, 0) to (0, 1). Y

z
z FGH IJK
pi

11. Evaluate the integral I = dz where C is the


z C C2
boundary of the half annulus as given in figure 1.

z
as

C1
2
12. Evaluate the integral z dz where C is the arc of
C
g

π
the circle |z| = 2 from θ = 0 to θ = . –3 –2 O 2 3 X
.c

3 A B C D

13. Evaluate
z 2z + 3
dz where C is
Fig. 1
w

C z
(i) upper half of the circle |z| = 2 in clockwise direction.
w

(ii) lower half of the circle |z| = 2 in anticlockwise direction.


(iii) the circle |z| = 2 in anticlockwise direction.

z
w

14. Evaluate the integral Re ( z2 ) dz from 0 to 2 + 4i along the line segment joining the points
C
(0, 0) and (2, 4).

15. Evaluate
z0
3+i
( z)2 dz along the real axis from z = 0 to z = 3 and then along a line parallel to

imaginary axis from z = 3 to z = 3 + i. (G.B.T.U. 2013)


16. Integrate f(z) = Re (z) from z = 0 to z = 1 + 2i.
(i) along straight line joining z = 0 to z = 1 + 2i.
(ii) along the real axis from z = 0 to z = 1 and then along a line parallel to imaginary axis from
z = 1 to z = 1 + 2i. (U.P.T.U. 2014)

www.cgaspirants.com
www.cgaspirants.com

FUNCTION OF COMPLEX VARIABLE 45

Answers
26 26 26 3 i
1. (a) 6 + i (b) 6 + i (c) 6 + i 2. +
3 3 3 2 6
8
3. 10 – i 4. (i) – 1 + i
3
5. (a) (i) 256.5 (ii) 200.25 (b) (i) 26/3 (ii) 26/3 ; No
1 2 2
6. (i) 1 – i (ii) (64i – 103) 7. (i) ;– (ii) 66, – 66
6 3 3
25 4
9. (a) 4 + 8i (b) 4 + i 10. – 1 11.
3 3
− 16
12. 13. (i) 8 – 3πi (ii) 8 + 3πi (iii) 6πi

m
3
26 1 + 2i 1
14. – 8 (1 + 2i) 15. 12 + i 16. (i) (ii) + 2i.

o
3 2 2

.c
1.20 SIMPLY AND MULTIPLY CONNECTED DOMAINS

ts
A domain in which every closed curve can be shrunk to a point without passing out of the
region is called a simply connected domain. If a domain is not simply connected, then it is
called multiply connected domain.
n
ra
1.21 SIMPLY AND MULTIPLY CONNECTED REGIONS

A curve is called simple closed curve if it does not cross itself (Fig. 1). A curve which crosses
pi

itself is called a multiple curve (Fig. 2).


A region is called simply connected if every closed curve in the region encloses points of
as

the region only, i.e., every closed curve lying in it can be contracted indefinitely without pass-
ing out of it. A region which is not simply connected is called a multiply connected region. In
plain terms, a simply connected region is one which has no holes. Figure 3 shows a multiply
g

connected region R enclosed between two separate curves C1 and C2. (There can be more than
.c

two separate curves). We can convert a multiply connected region into a simply connected one,
by giving it one or more cuts (e.g. along the dotted line AB).
w
w

R2 R
w

B
R C
R1 C2

C
C1

Fig. 1 Fig. 2 Fig. 3

Remark 1. Jordan arc is a continuous arc without multiple points.


Remark 2. Contour is a Jordan curve consisting of continuous chain of a finite number of regular arcs.

www.cgaspirants.com
www.cgaspirants.com

46 A TEXTBOOK OF ENGINEERING MATHEMATICS

1.22 CAUCHY'S INTEGRAL THEOREM [M.T.U. 2013, 2014; G.B.T.U. (C.O.) 2011]

Statement. If f(z) is an analytic function and f ′(z) is continuous at each point within and on a
simple closed curve C, then

z C
f ( z) dz = 0 .

C
Proof. Let R be the region bounded by the curve C. R
Let f(z) = u (x, y) + iv(x, y), then

z f ( z) dz = z (u + iv) (dx + idy)

z z
C C

m
= (udx − vdy) + i (vdx + udy) ...(1)
C C

o
∂u ∂u ∂v ∂v
Since f ′(z) is continuous, the partial derivatives , , , are also continuous

.c
∂x ∂y ∂x ∂y
in R. Hence by Green’s Theorem, we have

zC
f ( z) dz = zz FGH
n R

∂v ∂u

∂x ∂y
IJ
dx dy + i
K ts zz FGH
R
∂u ∂v

∂x ∂y
IJ
dx dy
K ...(2)

Now f(z) being analytic at each point of the region R, by Cauchy-Riemann equations, we
ra
have
∂u ∂v ∂u ∂v
= and =−
∂x ∂y ∂y ∂x
pi

Thus, the two double integrals in (2) vanish.

z
as

Hence f ( z) dz = 0.
C
However Cauchy with the help of Goursat developed the revised form of Cauchy’s
g

fundamental theorem which states that

z
.c

“If f(z) is analytic and one valued within and on a simple closed contour C then f ( z) dz = 0.”
C
w

Goursat showed that for the truth of the original theorem, the assumption of continuity
of f ′(z) is unnecessary and Cauchy’s theorem holds if f(z) is analytic within and on C.
w

Corollary. If f(z) is analytic in a region R and P, Q are two


points in R, then z Q
f ( z) dz is independent of the path joining P Q
w

R
P
and Q and lying entirely in R.
B
Let PAQ and PBQ be any two paths joining P and Q.
By Cauchy’s theorem,

z
A
P
f ( z) dz = 0

z z
PAQBP

⇒ f ( z) dz + f ( z) dz = 0

z z
PAQ QBP

⇒ f ( z) dz − f ( z) dz = 0

z z
PAQ PBQ

Hence f ( z) dz = f ( z) dz.
PAQ PBQ

www.cgaspirants.com
www.cgaspirants.com

FUNCTION OF COMPLEX VARIABLE 47

1.23 EXTENSION OF CAUCHY'S THEOREM TO MULTIPLY CONNECTED REGION

If f(z) is analytic in the region R between two simple closed


curves C1 and C2, then

z C1
f(z) dz = z
C2
f(z) dz
A B
C2
C1
when integral along each curve is taken in anti-clockwise
direction.

Proof. z f ( z) dz = 0

m
where the path of integration is along AB and curve C2 in clockwise direction and along BA
and along C1 in anti-clockwise direction.

z z z z

o
f ( z) dz + f ( z) dz + f ( z) dz + f ( z) dz = 0

.c
AB C2 BA C1

or zC2
f ( z) dz + z C1
f ( z) dz = 0
n ts ∵ z
AB
f ( z) dz = − zBA
f ( z) dz

Reversing the direction of the integral around C2, we get

z z
ra

f ( z) dz = f ( z) dz
C1 C2
pi

However if a closed curve C contains non-intersecting closed curves C1, C2, ....., Cn,
then by introducing cross-cuts, it can be shown that

z z z z
as

f ( z) dz = f ( z) dz + f ( z) dz + + f ( z) dz .
C C1 C2 Cn
g

C
.c
w
w

C1 C6
w

C5
C2
C4
C3

www.cgaspirants.com
www.cgaspirants.com

48 A TEXTBOOK OF ENGINEERING MATHEMATICS

EXAMPLES

Example 1. Evaluate
Sol.
z C
(x 2 − y 2 + 2ixy) dz , where C is the contour | z | = 1.
f(z) = x2 – y2 + 2ixy = (x + iy)2 = z2 is analytic everywhere within and on | z | = 1.
∴ By Cauchy’s integral theorem, z
C
f ( z) dz = 0.

Example 2. Evaluate zC
y = a(1 – cos θ) between (0, 0) and (2πa, 0).
(3z 2 + 4z + 1) dz where C is the arc of the cycloid x = a(θ – sin θ),

Sol. Here, f(z) = 3z2 + 4z + 1 is analytic everywhere so that the integral is independent

m
of the path of integration and depends only on the end points z1 = 0 + i0 and z2 = 2πa + i0.

z z L O 2 πa
2 πa
2
(3 z + 4 z + 1) dz = (3 z 2
+ 4 z + 1) dz = M z 3
+ 2z 2
+ zP

o
∴ = 2πa (4π2a2 + 4πa + 1).
C 0 N Q 0

.c
Example 3. Evaluate: z 2z + 5 2
dz , where C is the square with vertices at

1 + i, 2 + i, 2 + 2i, 1 + 2i.
C (z + 2) 3 (z 2 + 4)
n
2z2 + 5
ts
ra
Sol. Here, f(z) = Y
( z + 2) 3 ( z 2 + 4)
(2 + 2i)
Singularities are given by (1 + 2i)
pi

D E
(z + 2)3 (z2 + 4) = 0
z = – 2 (order 3), ± 2i (simple poles)
as

C
Since the singularities donot lie inside the contour
C hence by Cauchy’s integral theorem,

z
g

A B
2z2 + 5 (1 + i) (2 + i)
dz = 0.
.c

C ( z + 2) 3 ( z 2 + 4)

z O X
w

Example 4. Evaluate (5z 4 − z 3 + 2) dz around


C
w

(i) unit circle |z| = 1


(ii) square with vertices (0, 0), (1, 0), (1, 1), (1, 0)
w

(iii) curve consisting of the parabola y = x2 from (0, 0) to (1, 1) and y2 = x from (1, 1) to
(0, 0).
Sol. f(z) = 5z4 – z3 + 2 is analytic everywhere. So by Cauchy integral theorem,

z
C
f ( z) dz = 0

∴ For all given curves, z C


(5 z 4 − z 3 + 2) dz = 0.

Example 5. Verify Cauchy theorem by integrating eiz along the boundary of the triangle
with the vertices at the points 1 + i, – 1 + i and – 1 – i. [G.B.T.U. 2012, 2013; M.T.U. 2012)

www.cgaspirants.com
www.cgaspirants.com

FUNCTION OF COMPLEX VARIABLE 49

Sol. The boundary of triangle C consists of three lines C1, C2 and C3. So,

I= z
C
e iz dz

= z
C1
e iz dz + z C2
e iz dz + zC3
e iz dz = I1 + I2 + I3 ...(1)

Along C1: AB: y = 1, z = x + iy = x + i


∴ dz = dx

I1 = z
C1
e iz dz =
z1
−1
e i( x + i) dx

m
−1
= e (ix − 1) dx
1

o
1 e ix F I −1
e − i−1 − ei−1
= GH JK =

.c
e i 1
i
Along C2 : BE:


x=–1
z = x + iy = – 1 + iy
dz = i dy
n ts
z z
ra
−1
I2 = e iz dz = e i( −1+ iy) i dy
C2 1
pi

= ie
−i
z1
−1
e − y dy = i e–i − e − y e j −1

1
as

1 − i+ 1
= – i (e–i+1 – e–1–i) =
i
e − e −1 − i e j
g

Along C3 : EA: y = x, z = x + iy = (1 + i) x
.c

∴ dz = (1 + i) dx

z z
w

1
I3 = e iz dz = e i(1+ i) x (1 + i) dx
C3 −1
w

L e OP
= (1 + i) M
i(1+ i) x 1
Fe i− 1
− e − i+ 1 I
GH JK
w

=
N i(1 + i) Q −1
i

1 − i−1
From (1), I = I1 + I2 + I3 = e − e i − 1 + e − i + 1 − e −1− i + e i − 1 − e − i + 1 = 0
i
Hence Cauchy’s theorem is verified.
Example 6. Can the Cauchy-integral theorem be applied for evaluating the following
integrals? Hence evaluate these integrals.

(i) z
C
2
e sin z dz; C ≡ |z| = 1 (ii) z
C
tan z dz; C ≡ |z| = 1

(iii) z
C
ez
z2 + 9
dz; C ≡ |z| = 2

www.cgaspirants.com
www.cgaspirants.com

50 A TEXTBOOK OF ENGINEERING MATHEMATICS

Sol. (i) Let I=


2
z
C
2
e sin z dz

sin z
The integrand f(z) = e is analytic for all z and f ′(z) is continuous inside C. Hence,
Cauchy integral theorem can be applied.
∴ I=0
(ii) Let I= z
C
tan z dz

sin z π
The integrand f(z) = tan z = is analytic for all z except at the points z = ± ,
cos z 2

, ...... . All these points lie outside C. Also f ′(z) is continuous inside C. Hence Cauchy

m
±
2
integral theorem is applicable.

o
∴ I=0

z
.c
ez
(iii) Let I= dz
C z2 + 9

The integrand f(z) =


z2 + 9
ez
ts
is analytic everywhere except at the points z = ± 3i. These
n
points lie outside c and f ′(z) is continuous inside C. Hence Cauchy integral theorem is applica-
ra

ble and I = 0.

ASSIGNMENT
pi

1. (i) State Cauchy-integral theorem for an analytic function. Verify this theorem by integrating
as

the function z3 + iz along the boundary of the rectangle with vertices +1, –1, i, – i.
(U.P.T.U. 2015)
g

(ii) Verify Cauchy’s integral theorem for f (z) = z2 taken over the boundary of a square with vertices
at ± 1 ± i in counter-clockwise direction.

z
.c

2. Using Cauchy’s integral theorem, evaluate f ( z) dz , where f(z) is


C
w

(i) ez (ii) sin z (iii) cos z


(iv) zn ; n = 0, 1, 2, 3, ...... and C is any simple closed path.
w

3. Evaluate: (i)
z z2 − z + 1
dz ; C ≡ |z – 1| =
1
(ii)
z 1
dz ; C ≡ 1 < |z | < 2
w

C z−2 2 C z2 ( z2 + 9)
4. (i) Verify Cauchy’s theorem for f (z) = z3 taken over the boundary of the rectangle with vertices at
– 1, 1, 1 + i, – 1 + i.
(ii) Verify Cauchy’s theorem by integrating z3 along the boundary of a square with vertices at
1 + i, 1 – i, –1 + i and –1 – i. (U.P.T.U. 2014)
5. Evaluate:

(i)
zC
e− z
z+1
dz , where C is the circle |z| =
1
2

(ii)
zC
z2 + 5
z−3
dz , where C is the circle |z| = 1

(iii)
zC
3 z2 + 7 z + 1
z+1
dz , where C is the circle |z + i| = 1

www.cgaspirants.com
www.cgaspirants.com

FUNCTION OF COMPLEX VARIABLE 51

6. Evaluate the following integrals:

(i)
z
C
z3 + z + 1
z2 − 3 z + 2
dz , where C is the ellipse 4x2 + 9y2 = 1

(ii)
z
C
z+4
z2 + 2 z + 5
dz , where C is the circle |z + 1| = 1

(iii)
z
z2 − z + 1
z−1
dz , where C is the circle |z| =
1
2

z
C
dz
7. Evaluate I = around a triangle with vertices at (0, 0), (1, 0) and (0, 1).
C z−2

8. State and prove Cauchy’s integral theorem. Hence evaluate


z z 2 + 5z + 6
dz where C: |z|=
3

m
C z − 2 2
[M.T.U. 2014, G.B.T.U. (C.O.) 2011]

z e3iz

o
9. Evaluate dz , where C is the circle |z – π| = 3.2. (U.P.T.U. 2007)
( z + π) 3
C

.c
10. (i) Verify Cauchy’s theorem for the function f(z) = 3z2 + iz – 4 along the perimeter of square with
vertices 1 ± i, –1 ± i. (G.B.T.U. 2011)

ts
(ii) Verify Cauchy’s theorem for the function f(z) = 4z2 + iz – 3 along the positively oriented
square with vertices (1, 0), (– 1, 0), (0, 1) and (0, – 1). (M.T.U. 2012)
(iii) Verify Cauchy’s theorem for f(z) = z2 + 3z + 2 where c is the perimeter of square with vertices
n
1 ± i, – 1 ± i. (G.B.T.U. 2012)
ra

Answers
2. 0 in all cases 3. (i) 0 (ii) 0
pi

5. (i) 0 (ii) 0 (iii) 0


6. (i) 0 (ii) 0 (iii) 0
as

7. 0 8. 0 9. 0.

1.24 CAUCHY’S INTEGRAL FORMULA


g

(M.T.U. 2012, U.P.T.U. 2006, 2007, 2009, 2014; G.B.T.U. 2011, 2013)
.c

Statement. If f(z) is analytic within and on a closed curve C and a is any point within C, then
w

1
z f ( z)
w

f(a) = dz .
2πi C z−a
w

f ( z)
Proof. Consider the function , which is analytic at every
z−a C
point within C except at z = a. Draw a circle C1 with a as centre
f ( z) C1
and radius ρ such that C1 lies entirely inside C. Thus is
z−a
analytic in the region between C and C1. aρ
∴ By Cauchy’s theorem, we have

z C
f ( z)
z−a
dz = z
C1
f ( z)
z−a
dz ...(1)

www.cgaspirants.com
www.cgaspirants.com

52 A TEXTBOOK OF ENGINEERING MATHEMATICS

Now, the equation of circle C1 is


| z – a | = ρ or z – a = ρeiθ
so that dz = iρeiθ dθ

∴ zC1
f ( z)
z−a
dz = z 0
2π f (a + ρe iθ )
ρe iθ
. iρe iθ dθ = i z
0

f (a + ρe iθ ) dθ

Hence by (1), we have


f ( z)
C z−a
dz = i z

0
f (a + ρe iθ ) dθ

In the limiting form, as the circle C1 shrinks to the point a, i.e.,


z ...(2)

ρ → 0, then from (2),

z f ( z)
z−a
dz = i z 2π
f (a) dθ = if (a) z 2π
dθ = 2πif (a)

m
C 0 0

Hence f(a) =
1
z f ( z)
dz

o
2πi C z−a

.c
Aliter: About the point z = a, describe a small circle γ of radius r
f ( z)
lying entirely within C. Consider the function
z−a
.

This function is analytic in the region between C and γ.


n
Hence by Cauchy’s theorem for multiply connected region, we
ts a
ra
γ
have
f (z)
dz = zf (z)
dz z C
pi

C z −a γ z −a

⇒ z f (z)
dz − z f (a)
dz = z f ( z ) − f (a)
dz
as

C z−a γ z−a γ z−a

⇒ z f (z)
dz − f ( a ) z dz
= z f ( z ) − f (a)
dz
g

C z−a γ z−a γ z−a

z z z
.c

f (z) f ( z ) − f (a) dz
dz − 2πif ( a ) = dz ∵ = 2 πi
⇒ γz−a
z−a z−a
w

C γ
since|z − a |= r on γ

z z
w

f (z) f ( z ) − f (a)
⇒ dz − 2πif ( a ) = dz
C z−a γ z−a

z
w

|f ( z ) − f ( a )|
≤ |dz |
γ |z − a |


ε
ε
r γ
| dz| z ∵ f ( z) is continuous at z = a
∴ | f ( z) − f (a)|< ε
≤ . 2πr and| z − a|= r for z on γ
r
≤ 2πε → 0 as ε → 0

⇒ z C
f ( z)
z−a
dz – 2πif(a) = 0

⇒ f(a) =
1
2πi z
C
f ( z)
z−a
dz

www.cgaspirants.com
www.cgaspirants.com

FUNCTION OF COMPLEX VARIABLE 53

1.25 CAUCHY’S INTEGRAL FORMULA FOR THE DERIVATIVE OF AN ANALYTIC


FUNCTION [U.P.T.U. (C.O.) 2009, 2010]

If a function f(z) is analytic in a region D, then its derivative at any point z = a of D is also
analytic in D and is given by

f ′(a) =
1
2πi z C
f(z)
(z − a) 2
dz

where C is any closed contour in D surrounding the point z = a.


Proof. Let a + h be a point in the neighbourhood of the point a. Then by Cauchy’s Integral
Formula

m
1 f ( z)
f(a) = dz
2πi C z−a

o
1 f ( z)
f(a + h) = dz

.c
2πi C z−a−h

∴ f(a + h) – f(a) =
1
2πi
n z RST
C
1

1
z−a−h z−a
f ( z) dz =
ts
h
2πi
UV
W z
C
f ( z) dz
( z − a − h) ( z − a)

f (a + h) − f (a) 1
z f ( z) dz
ra
⇒ =
h 2πi C ( z − a − h) ( z − a)
Take limit as h → 0

z
pi

f (a + h) − f (a) 1 f ( z) dz
Lt = Lt
h→0 h h → 0 2πi C ( z − a − h) ( z − a)
as

⇒ f ′(a) =
1
2πi z f ( z)
( z − a) 2
dz ...(1)
g

C
.c

Since a is any point of the region D, so by (1) it is clear that f ′(a) is analytic in D. Thus,
the derivative of an analytic function is also analytic.
w

1.26 THEOREM
w

If a function f(z) is analytic in a domain D, then at any point z = a of D, f(z) has derivatives of
all orders, all of which are again analytic functions in D, their values are given by
w

f n(a) =
n!
2πi z f(z)
C (z − a) n + 1
dz

where C is any closed contour in D surrounding the point z = a.


Proof. We shall prove this theorem by Mathematical Induction.
Let the theorem be true for n = m. Then

f m(a) =
m!
2πi z C
f ( z)
( z − a) m + 1
dz is true.


f m (a + h) − f m (a) m ! 1
h
=
2 πi h
LM
N zC
f ( z) dz
( z − a − h) m+1
− z
C
f ( z) dz
( z − a) m+1
UV
W

www.cgaspirants.com
www.cgaspirants.com

54 A TEXTBOOK OF ENGINEERING MATHEMATICS

R|F h I U
=
m! 1
. .
2 πi h zC
1
( z − a) m + 1
S|GH 1 − z − a JK − 1|V| f (z) dz
T
− ( m + 1)

W
=
m! 1
. .
2πi h zC
1
( z − a) m +1
R|S(m + 1) h + (m + 1) (m + 2) h
|T z−a 2! ( z − a)
2

2
+
U|V f (z) dz
|W
Take limit as h → 0

h→0
f m (a + h) − f m (a) (m + 1) !
lim
h
=
2πi
f ( z)
z
C ( z − a) m + 2
dz

⇒ f m+1(a) =
(m + 1) ! f ( z)
z dz

m
2πi C ( z − a) m + 2

Hence the theorem is true for n = m + 1 if the theorem is true for n = m. But we know by
Cauchy’s Integral formula for the derivative of a function that the theorem is true for n = 1.

o
Hence the theorem must be true for n = 2, 3, 4, ...... and so on i.e., for all +ve integral values

.c
of n. Thus,

f n(a) =
n!
2 πi
n z C
f ( z)
( z − a) n + 1
dz
ts ...(1)

Since a is any point of the region D, so by (1) it is clear that f n(a) is analytic in D. Thus
ra

the derivatives of f(z) of all orders are analytic if f(z) is analytic.


Thus, if a function of a complex variable has a first derivative in a simply connected
pi

region, all its higher derivatives exist in that region. This property is not exhibited by the functions
of real variables.
as

1.27 CAUCHY’S INEQUALITY

If f(z) is analytic within a circle C given by | z – a | = R and if | f(z) | ≤ M on C, then


g
.c

Mn !
| f n(a) | ≤ .
Rn

z
w

n! f ( z) dz
Proof. f n(a) = 2πi
( z − a) n + 1
w

n!
z f ( z) dz
w

∵ z – a = R e iθ
⇒ | f n(a) | = n+ 1
2 πi C ( z − a) ∴ dz = i R e iθ dθ
n!
≤ |2πi| z | f ( z)|| dz|
C |( z − a) n + 1 |
∴ | dz|=|i R e iθ dθ|
= R dθ


n! M
2π R n + 1 z0

R dθ

n! M Mn !
≤ n + 1 2πR ≤ .
2π R Rn

www.cgaspirants.com
www.cgaspirants.com

FUNCTION OF COMPLEX VARIABLE 55

EXAMPLES

Example 1. Evaluate zC
e− z
z+1
dz , where C is the circle | z | = 2

Sol. f(z) = e–z


is an analytic function.
The point a = – 1 lies inside the circle | z | = 2.
∴ By Cauchy’s integral formula,

z C
e− z
z+1
dz = 2πi(e–z)z= –1 = 2πie.

m
Example 2. Evaluate the following integral:
1
z

o
cos z dz
C z

.c
where C is the ellipse 9x2 + 4y2 = 1.
Sol. Pole is given by z = 0. The given ellipse encloses the simple pole.
∴ By Cauchy’s integral formula,

z z
cos z
C
n
dz = 2πi (cos z)z=0 = 2πi.
ts
ra

Example 3. (i) Use Cauchy Integral formula to evaluate

z
sin πz 2 + cos πz 2
dz
pi

C (z − 1)(z − 2)
where C is the circle | z | = 3. [G.B.T.U. 2010; G.B.T.U. (C.O.) 2011]

z
as

sin πz + cos πz
(ii) Evaluate: dz, where C is the circle | z | = 4. (U.P.T.U. 2008)
C (z − 1) (z − 2)
g

Sol. (i) The integrand has singularities given by


.c

Y
(z – 1) (z – 2) = 0 ⇒ z = 1, 2
The given circle | z | = 3 with centre at z = 0 and radius C≡|z|=3
w

3 encloses both the singularities.


F sin πz + cos πz I
2 2
w

GH z − 2 JK
z z
z=2
∴ sin πz 2 + cos πz 2 O z = 1
X
dz = dz
w

C (z − 1 )(z − 2 ) C1 z−1
F sin πz + cos πz I
2 2
GH z − 1 JK
+ z
C2 z−2
dz

= 2πi
LM sin πz + cos πz OP + 2πi LM sin πz + cos πz OP
2 2 2 2

N z−2 Q N z−1 Q
z= 1 z=2

F 0 − 1IJ + 2πi FG 0 + 1IJ = 2πi + 2πi = 4πi.


= 2πi G
H −1 K H 1 K

www.cgaspirants.com
www.cgaspirants.com

56 A TEXTBOOK OF ENGINEERING MATHEMATICS

(ii) Singularities are given by


(z – 1) (z – 2) = 0 ⇒ z = 1, 2
The given circle | z | = 4 with centre at z = 0 and radius 4 encloses both the singularities.
FG sin πz + cos πz IJ FG sin πz + cos πz IJ
∴ zC
sin πz + cos πz
( z − 1) ( z − 2)
dz = z C1
H z − 2 K dz + H z − 1 K dz
z−1 z
C2 z−2

= 2πi
LM sin πz + cos πz OP + 2πi LM sin πz + cos πz OP
N z−2 Q N z−1 Q
z=1 z=2

= 2πi M
L − 1OP + 2πi L 1O = 2πi + 2πi = 4πi
N − 1Q MN 1PQ

m
Example 4. (i) Evaluate the following integral using Cauchy Integral formula

o
4 − 3z
dz , where C is the circle | z | = 3/2.

.c
C z (z − 1)(z − 2)
(U.P.T.U. 2015)
(ii) Use Cauchy-integral formula to evaluate
ts
z
n
z 1
dz, where C is the circle z − 2 = . (U.P.T.U. 2009)
z 2 − 3z + 2
ra
C 2
Sol. (i) Poles of the integrand are z = 0, 1, 2. These are simple poles.
pi

3 3
Given circle | z | = with centre at z = 0 and radius encloses two poles z = 0 and z = 1.
2 2
as

4 − 3z 4 − 3z
∴ z 4 − 3z
dz = z ( z − 1) ( z − 2)
dz + z z ( z − 2)
dz
g

C z ( z − 1) ( z − 2) C1 z C2 ( z − 1)

LM 4 − 3z OP LM 4 − 3z OP
.c

= 2πi + 2πi = 2πi.


N (z − 1) (z − 2) Q N z (z − 2) Q
w

z=0 z= 1

(ii) Poles of the integrand are given by


w

z2 – 3z + 2 = 0 ⇒ z = 1, 2
1 1
w

Both are simple poles. The given circle | z – 2 | = with centre at z = 2 and radius
2 2
encloses only one of the poles at z = 2.
∴ By Cauchy’s integral formula,

FG z IJ
H z − 1K dz = 2πi LM z OP
zC 2
z
z − 3z + 2
dz = zC z−2 N z − 1Q z=2
= 2πi
FG 2 IJ = 4πi
H 1K
Example 5. Evaluate by Cauchy’s integral formula

zC
dz
z( z + πi)
, where C is | z + 3i | = 1

www.cgaspirants.com
www.cgaspirants.com

FUNCTION OF COMPLEX VARIABLE 57

Sol. Poles of the integrand are z = 0, – πi (simple poles)


The given curve C is a circle with centre at z = – 3i, i.e., at (0, – 3) and radius 1.
Clearly, only the pole z = – πi lies inside the circle.
FG 1IJ
∴ z
C
dz
z( z + πi)
= z
C
H zK
z + πi
dz

= 2π i FG 1IJ | By Cauchy’s Integral formula


H zK z = − πi

2πi
= =–2

m
− πi

z z2 + 1

o
Example 6. Evaluate dz where C is circle,
C z2 − 1

.c
(i) | z | = 3/2 (ii) | z – 1 | = 1 (iii) | z | = 1/2.
(U.P.T.U. 2014) (U.P.T.U. 2014)
Sol. The integrand has singularities given by
z2 – 1 = 0 ⇒ z = ± 1
n ts
ra
(i) The given curve C is a circle with centre at origin Y
(0, 0) and radius 3/2.
C ≡ | z | = 3/2
Both the singularities z = 1 and z = – 1 lie inside the
pi

circle | z | = 3/2.
F z + 1I
2 F z + 1I 2
as

GH z + 1 JK GH z − 1 JK
z z z
X
∴ z2 + 1 z=–1 O z=1
dz = dz + dz
z2 − 1 z−1 z+1
g

C C1 C2

F z + 1I F z + 1I
.c

2 2
= 2πi GH z + 1 JK + 2πi GH z − 1 JK
w

z= 1 z=−1

| By Cauchy’s Integral formula


w

= 2πi (1) + 2πi (– 1) = 0 Y


(ii) The given curve C is a circle with centre at C ≡ | z – 1| = 1
w

(1, 0) and radius 1.


Only the singularity z = 1 lie inside the given
z=1
circle | z – 1 | = 1. O
X
F z + 1I (0, 0) (1, 0)
2
GH z + 1 JK
∴ zC
2
z +1
z2 − 1
dz = z
C z−1
dz

F z + 1I2
= 2πi GH z + 1 JK z= 1
= 2πi | By Cauchy’s Integral formula

www.cgaspirants.com
www.cgaspirants.com

58 A TEXTBOOK OF ENGINEERING MATHEMATICS

(iii) The given curve C is a circle with centre at Y


1 C ≡ | z | = 1/2
origin (0, 0) and radius . Clearly both the
2
singularities z = 1 and z = – 1 lie outside the
1 X
given circle | z | =
. O
2
Hence, by Cauchy’s Integral theorem

z
C
z2 + 1
z2 − 1
dz = 0.

m
Example 7.(i) Use Cauchy’s integral formula to show that

z e zt
dz = 2πi sin t if t > 0 and C is the circle |z| = 3. (U.P.T.U. 2009)

o
C z2 + 1

.c
(ii) Evaluate the following complex integration using Cauchy’s integral formula

z
C
3z 2 + z + 1
(z 2 − 1)(z + 3)
n dz
ts
where C is the circle | z | = 2

Sol. (i) Singularities of the integrand are given by


ra
z2 + 1 = 0 ⇒ z = ± i (order 1)
The circle |z| = 3 has centre at z = 0 and radius 3. It encloses both the singularities z = i
and z = – i.
pi

Fe I zt Fe I zt
GH z + i JK GH z − i JK
z z z z
as

e zt e zt
Now, dz = dz = dz + dz
C z2 + 1 C ( z − i) ( z + i) C1 z−i C2 z+i

Fe I Fe I
g

zt zt
= 2πi GH z + i JK + 2πi GH z − i JK = π (eit – e– it) = 2πi sin t
.c

z=i z=−i
w

(ii) Poles of the integrand are given by


(z2 – 1) (z + 3) = 0 ⇒ z = 1, – 1, – 3 (simple poles)
w

The circle | z | = 2 has centre at z = 0 and radius 2. clearly the poles z = 1 and z = – 1
lie inside the given circle while the pole z = – 3 lie outside it.
w

RS 3z + z + 1 UV
2 RS 3z + z + 1 UV 2

∴ zC
2
3z + z + 1
( z 2 − 1)( z + 3)
dz = z
C1
T ( z + 1)(
z−1
z + 3)
z
W dz + T (z − 1)(z + 3) W dz
C2 z+1

= 2πi
LM 3z + z + 1 OP + 2πi LM 3z + z + 1 OP
2 2

N (z + 1)(z + 3) Q z=1N (z − 1)(z + 3) Q z=−1


| Using Cauchy’s Integral formula
F 5I F 3I F − 1IJ = – πi
= 2πi GH JK + 2πi GH − JK = 2πi GH
8 4 8 K 4

www.cgaspirants.com
www.cgaspirants.com

FUNCTION OF COMPLEX VARIABLE 59

Example 8. Integrate (z 3 – 1) –2 the counterclockwise sense around the circle


|z–1|=1
Y
Sol. Singularities of integrand are given by
C ≡ | z – 1| = 1
(z3 – 1)2 = 0
⇒ (z – 1)2 (z2 + z + 1)2 = 0

− 1± i 3
⇒ z = 1, O
z=1 X
2
Singularities are of second order.
The circle | z – 1 | = 1 has centre at z = 1 and
radius 1. Clearly, only z = 1 lies inside the circle

m
|z–1|=1
RS UV

o
1

z dz
z T (z 2
+ z + 1) 2 W dz

.c
Now, =
C ( z − 1) 2
3
C ( z − 1) 2
LM R UVOP
=
2πi d
n
MN TS 1
1 ! dz ( z + z + 1) 2
2
WPQ ts
Using Cauchy's Integral
formula for derivatives
z=1

L − 2 (2 z + 1) OP = – 4πi FG 3 IJ = – 4πi
ra

= 2πi M
N (z + z + 1) Q 2 H 27 K 93
z=1

z
pi

ez
Example 9. Evaluate: dz, where C is | z | = 4. (U.P.T.U. 2008)
C (z 2 + π 2 ) 2
as

Sol. Singularities of the integrand are given by


(z2 + π2)2 = 0 ⇒ z = ± πi (order 2)
g

The given curve C is a circle with centre at origin and radius 4. The circle encloses both
.c

the singularities.
RS e UV z RS e UV z
w

∴ z ez
dz = z T (z + πi) W dz + T (z − πi) W dz
2

z 2
w

C (z2 + π 2 ) 2 C1 ( z − πi) 2 C2 ( z + πi) 2

LM d R e UOP LM d R e UVOP
w

z z
= 2πi
MN dz ST ( z + πi) VWPQ 2
z = πi
+ 2πi
MN dz ST ( z − πi) 2
WPQ z = − πi

| By C–I formula for derivatives

= 2πi
LM e z
( z + πi − 2) OP + 2πi
LM e z
( z − πi − 2) OP
N ( z + πi) 3 Q z = πi N ( z − πi) 3 Q z = − πi

FG πi − 1IJ + FG πi + 1IJ = i .
=
H 2π K H 2π K π
2 2

www.cgaspirants.com
www.cgaspirants.com

60 A TEXTBOOK OF ENGINEERING MATHEMATICS

Example 10. Use Cauchy’s integral formula to evaluate

zC
e 2z
(z + 1) 4
dz where C is the circle | z | = 2. [U.K.T.U. 2011]

Sol. The integrand has a singularity at z = – 1 which lies within the circle | z | = 2.

z
C
e2 z
( z + 1) 4
dz =
2πi d 3 2 z
3 ! dz 3
RS
(e )
T
UV
W z =−1
=
πi
3
8πi
(8e2z)z = –1 = 2 .
3e

Example 11. Evaluate z C z +12


z
dz , where

m
1
(i) C ≡ z + =2 (ii) C ≡ | z + i | = 1.
z

o
Sol. Poles of the integrand are given by
z2 + 1 = 0 ⇒

.c
z=±i
Integrand has two simple poles z = i and z = – i
(i) The given curve is
n
z+
1
z
=2
ts Y

C1 ≡ | z – i | = 2
ra
2

1
⇒ x + iy + =2 i
x + iy
pi

O
x 2 – y 2 + 2ixy + 1 X
as

⇒ =2
x + iy
C2 ≡ | z + i | = 2
g

⇒ (x2 – y2 + 1)2 + 4x2y2 = 4x2 + 4y2 –i


.c

⇒ x4 + y4 – 2x2y2 +1+ 2x2 – 2y2 + 4x2y2 2


= 4x2 + 4y2
w

⇒ (x2 + y2 ) 2 – 2(x2 + y2 ) +1= 4y2


w

⇒ x2 + y2 – 1 = ± 2y
⇒ x2 + (y ± 1)2 = 2
w

Above eqn. represents two circles with centres (0, 1), (0, – 1) and radius 2.

z C 2
z +1
z
dz = zC1 z +12
z
dz + z C2 2
z +1
z
dz

FG z IJ FG z IJ
= zH C1
z+i
z−i
dz +
K
zC2
H z − i K dz
z+i

= 2πi
FG z IJ + 2πi
FG z IJ
H z + iK z=i
H z + iK z=−i

www.cgaspirants.com
www.cgaspirants.com

FUNCTION OF COMPLEX VARIABLE 61

FG 1IJ + 2πi FG 1IJ = 2πi.


= 2πi
H 2K H 2K Y
(ii) The given curve | z + i | = 1 is a circle with
centre at z = – i and radius 1. Clearly only the pole
O (0, 0)
z = – i lies inside the circle | z + i | = 1 X

FG z IJ C≡|z+i|=1

zC
z
z2 + 1
dz = z C
H z − i K dz
z+i (0, –1) z=–i

= 2πi
FG z IJ
H z − iK

m
z=−i
= πi | By Cauchy Integral formula

o
Example 12. Evaluate by using Cauchy Integral formula

z
.c
z−1
dz , where C is |z – i| = 2.
C ( z + 1) 2 ( z − 2)
Sol. Poles of the integrand are given by


(z + 1)2 (z – 2) = 0
z = – 1, 2
n ts Y

C≡|z–i|=2
ra
(0, 3)
z = – 1 is a double pole while z = 2 is a simple
pole.
pi

(0, 1)
The given curve C is a circle with centre at (0, 1)
2
and radius 2. Clearly, the pole z = – 1 lies inside the z=–1
1
z=2
given circle while the pole z = 2 lies outside it.
as

(–1, 0) O (0, 0) X
Hence, ( 3, 0)

FG z − 1IJ (0, –1)


g

z z−1
z H z − 2 K dz
.c

dz =
C ( z + 1) 2 ( z − 2) C ( z + 1) 2
RS FG IJ UV
w

2πi d z − 1
=
T H
1 ! dz z − 2 KW
w

z=−1

R −1
= 2πi S
UV =–
2πi
.
w

T (z − 2) 2
W z =−1
9

ASSIGNMENT

1. Evaluate
zC
z2 + 5
z−3
dz , where C is the circle | z | = 4.

2. Evaluate
zC
2
ez
z +1
dz over the circular path | z | = 2.

3. Evaluate
zC
3 z2 + 7 z + 1
z+1
dz , where C is the circle | z | = 1.5.

www.cgaspirants.com
www.cgaspirants.com

62 A TEXTBOOK OF ENGINEERING MATHEMATICS

4.

5.
Evaluate
cos z
z−π zdz , where C is the circle | z – 1 | = 3.
C
Evaluate the complex integration

(i)
zC
|RS cos πz + sin πz U|V dz where C is the circle | z | = 3.
2

T| ( z + 1)( z + 2) |W
2

(ii)
zC
sin πz2 + cos πz2
( z − 1) ( z − 3)
dz where C: |z| = 2 (M.T.U. 2013)

6. (i) Evaluate zz dz
( z − 1)( z − 3)
(a) | z | = 3
C
, where C is the circle

(b) | z | = 3/2.

m
ez
(ii) Evaluate dz , where C is the circle | z | = 2.
C ( z − 1)( z − 4)

o
(iii) Evaluate using Cauchy’s integral formula:
e2 z
z
.c
dz where C is the circle | z | = 3.
C ( z − 1)( z − 2)

ts
(iv) State Cauchy’s integral formula. Hence evaluate: (G.B.T.U. 2011, 2012)

z exp ( iπz )
dz
n
C ( 2z 2 − 5z + 2 )
ra
where C is the unit circle with centre at origin and having positive orientation.

7. (i) Evaluate
z ez
z( z + 1)
1
dz , where C is the circle | z | = .
4
pi

(ii) Using Cauchy Integral formula, evaluate


z dz
where C ≡ | z | = 2.
as

C z2 − 1

(iii) Evaluate z 2z + 1
dz where C is | z| =
1
.
g

2 2
C z +z

z
.c

cos πz
8. Evaluate dz around a rectangle with vertices
z2 − 1
C
w

(a) 2 ± i, – 2 ± i (b) – i, 2 – i, 2 + i, i.
w

ez
9. Integrate 2
around the contour C, where C is
z +1
w

(i) | z – i | = 1 (ii) | z + i | = 1

10. Show that


z
C
ez

z
dz = 2πi, C ≡ | z | = 1. Hence show that

z0

ecos θ cos (sin θ) dθ = 2π and
z 2π

0
ecos θ sin (sin θ) dθ = 0

11. Evaluate
z
z2 + 9
C
(i) | z – 3i | = 4
dz
, where C is

(ii) | z + 3i | = 2 (iii) | z | = 5

www.cgaspirants.com
www.cgaspirants.com

FUNCTION OF COMPLEX VARIABLE 63

12. Evaluate:

(i) z
C
2
z+4
z + 2z + 5
dz ; C ≡ | z + 1 – i | = 2 (ii)
z
C
z3 − 6
2z − i
dz ; C ≡ | z | = 1

(iii)
z
C
tan z
z2 − 1
dz ; C ≡ | z | = 3/2 (iv) z C
2 z2 + z
z2 − 1
dz ; C ≡ | z – 1 | = 1.

13. Evaluate by Cauchy-Integral formula:


zC
z2 + 1
z2 − 1
dz, where C is

(i) | z – 1 | = 1 (ii) | z + 1 | = 1 (iii) | z – i | = 1.


14. Evaluate the following integrals:

z cos 2πz
z z 4 − 3 z2 + 6

m
(i) dz; C ≡ | z | = 1 (ii) dz; C ≡ | z | = 2
C (2 z − 1) ( z − 3) C ( z + i)3

z cosh z

o
(iii) dz; C ≡ | z | = 1/2
C z4

z FGH
.c
sin 2 z
15. Evaluate dz , where C is the circle | z | = 1.
C π IJ 3

16. (i) Evaluate


z
z−

C
6 K
e−2 z dz
( z + 1)3
n
, where C is the circle | z | = 2.
ts
ra

(ii) Evaluate the integral


z e2z
( z + 1) 5
dz around the boundary of the circle |z| = 2.
pi

(U.P.T.U. 2015)

17. (i) Evaluate:


z e 3z
dz, where C is the square with vertices at ± 1 ± i.
as

C ( z − log 2)4

(ii) Evaluate:
z dz
, where C ≡ | z | = 1. (G.B.T.U. 2013)
g

C z 2 ( z 2 − 4) e z

z
.c

ez
18. Evaluate dz, where C is
C z (1 − z)3
w

1 1
(i) | z | = (ii) | z – 1 | = (iii) | z | = 2
2 2
w

sin 2 z
19. Integrate around the contour C, where C is a rectangle with vertices at 3 ± i, – 2 ± i.
( z + 3) ( z + 1)2
w

20. Evaluate:

(i) | z | = 3
z C
z3 − z
( z − 2)3
dz, where C is

(ii) | z – 2 | = 1 (iii) | z | = 1
21. Using Cauchy-integral formula, evaluate:

(i)
z
C
cos z
( z − πi)2
dz; C≡|z|=5 (ii)
z
C
ez
z3
dz ; C≡|z|=1

(iii) z
C
ez
( z − 1) ( z2 + 4)
2 dz; C≡|z–1|=
1
2

(iv)
z
C
e zt
( z 2 + 1) 2
dz; C ≡ | z | = 3, t > 0.

www.cgaspirants.com
www.cgaspirants.com

64 A TEXTBOOK OF ENGINEERING MATHEMATICS

22. Evaluate
z
sin z
z2 − iz + 2
(i) | z + 2 | = 2
C
dz, where C is

(ii) A rectangle with vertices at (1, 0), (1, 3), (– 1, 3) and (– 1, 0)


(iii) A rectangle with vertices at (2, 0), (2, 3), (– 2, 3) and (– 2, – 3).
23. Evaluate the integrals

24.
(i)
z e z + sin πz
( z − 1) ( z − 3)2 ( z + 4)
C
Show that
dz, C≡|z|=2 (ii)
z
C
z+1
z2 − 9
dz; C ≡ | z + 3 | = 1

(i)
z dz
=
π
; C≡|z–i|=2

m
2 2 16
C ( z + 4)

(ii) z ez
dz =
FG 11 − 4IJ πi ; C ≡ | z | = 2
He K

o
2 3
C z ( z + 1)

z dz

.c
(iii) = 0; C ≡ | z – 1 | = 4
C ( z 2 + 4) 3

25. Evaluate
z C
z
( z − 6 z + 25)2
2
ts
dz by Cauchy integral formula, where C is | z – 3 – 4i | = 4.

z z z
n
P( z ) P( z ) P( z )
26. Let P(z) = a + bz + cz2 and dz = dz = dz = 2πi where C is the circle | z | = 1.
ra
2
z C z C z3
Evaluate P(z).

z
pi

3z 2 + 7z + 1
27. If f(ξ) = dz, where C is the circle x2 + y2 = 4, find the values of f(3), f ′(1 – i) and
C z−ξ
as

f ″(1 – i).

28. Evaluate:
zC
(1 + z) sin z
(2 z − 3) 2
dz, where C ≡ | z – i | = 2 counter-clockwise. (U.P.T.U. 2014)
g
.c

Answers
1. 28πi 2. 2πi sin 1 3. – 6πi 4. – 2πi
w

5. (i) – 4πi (ii) πi


w

2 2π
6. (i) (a) 2πi (b) – πi (ii) – πie (iii) 2πi (e4 – e2) (iv)
3 3
w

7. (i) 2πi (ii) 0 (iii) 2πi


8. (a) 0 (b) – πi
9. (i) π(cos 1 + i sin 1) (ii) – π(cos 1 – i sin 1)
π π
11. (i) (ii) – (iii) 0
3 3
π π
12. (i) (3 + 2i) (ii) – 6πi (iii) 2πi tan 1 (iv) 3πi
2 8
13. (i) 2πi (ii) – 2πi (iii) 0
2πi
14. (i) (ii) – 18πi (iii) 0 15. πi
5
4 πi
16. (i) 4πie2 (ii) 2
3e

www.cgaspirants.com
www.cgaspirants.com

FUNCTION OF COMPLEX VARIABLE 65

πi
17. (i) 72πi (ii) –
2
18. (i) 2πi (ii) – πie (iii) πi (2 – e)
πi
19. (4 cos 2 + sin 2) 20. (i) 12πi (ii) 12πi (iii) 0
2
6πe
21. (i) 2π sinh π (ii) πi (iii) (iv) πi (sin t – t cos t)
25
2πi 2πi
22. (i) 0 (ii) sinh 2 (iii) (sinh 2 + sinh 1)
3 3
πie 2πi 3π
23. (i) (ii) 25. .
10 3 128

m
26. P(z) = 1 + z + z2 27. f(3) = 0, f ′(1 – i) = 2π(6 + 13i), f ″(1 – i) = 12πi
πi 5 FG3 3 IJ

o
28. cos + sin
2 2 H2 2 K

.c
1.28 REPRESENTATION OF A FUNCTION BY POWER SERIES

A series of the form


∑a z n
n
or

∑a
n
n (z
ts
− a) n whose terms are variable is called a power
n=0
ra
n=0
series, where z is a complex variable and an, a are complex constants. The second form can be
reduced to first form merely by substitution z = ζ + a or by changing the origin.
pi

Every complex function f(z) which is analytic in a domain D can be represented by a


power series valid in some circular region R about a point z0. Both the circular region R and
the point z0 lie inside D. Such a power series is Taylor’s series. If f(z) is not analytic at a point
as

z0, we can still expand f(z) in an infinite series having both positive and negative powers of
z – z0. This series is called the Laurent’s series.
g

1.29 TAYLOR’S SERIES [U.P.T.U. (C.O.) 2008]


.c

If f(z) is analytic inside a circle C with centre at a, then for all z inside C,
w

(z − a) 2 (z − a)n n
f(z) = f(a) + (z – a) f ′(a) + f ″ (a) + + f (a) + ⋅⋅⋅
2! n!
w

Or
w


f (n) (a)
f(z) = ∑
n =0
an (z − a) n , where an =
n!
.

Proof. Let z be any point inside the circle C. Draw a circle C1 with C
centre at a and radius smaller than that of C such that z is an w
interior point of C1. Let w be any point on C1, then
z−a
|z–a|<|w–a| i.e., <1 z
w− a C1 a

Now,
1
=
1
=
1
1−
z−a LM OP −1

w − z (w − a) − ( z − a) w − a w− a N Q

www.cgaspirants.com
www.cgaspirants.com

66 A TEXTBOOK OF ENGINEERING MATHEMATICS

z−a
Expanding RHS by binomial theorem as < 1, we get
w− a
1 1 z−aLM z−a FG IJ 2
F z − a IJ n OP
= 1+ + + +G + ...(1)
w− z w− a w− aMN w− a H K H w − aK PQ
z−a
This series converges uniformly since < 1. Multiplying both sides of eqn. (1) by
w− a
1
f (w) and integrating term by term w.r.t. w, over C1, we get
2πi
1
z f (w)
dw =
1 f (w)
z
dw +
z−a f (w)
dw + z
( z − a) 2
z f (w)
dw

m
2πi C1 w − z 2πi C1 w − a 2πi C1 (w − a) 2 2πi C 1 (w − a) 3

+ ⋅⋅⋅ +
( z − a) n
z f (w)
dw + ⋅⋅⋅ ...(2)

o
2πi C1 (w − a) n + 1

.c
( z − a) 2 ( z − a) n n
⇒ f(z) = f(a) + (z – a) f ′(a) + f ″ (a) + ⋅ ⋅ ⋅ + f (a) + ⋅⋅⋅ ...(3)

which is the required Taylor’s series for f(z) about z = a.


Cor. 1. Putting z = a + h in (3), we get
n 2!
ts n!
ra

h2 hn n
f(a + h) = f(a) + hf ′(a) + f ″(a) + ⋅⋅⋅ + f (a) + ⋅⋅⋅
2! n!
pi

Cor. 2. If a = 0, the series (3) becomes


z2 zn n
as

f(z) = f(0) + zf ′(0) + f ″(0) + ⋅⋅⋅ + f (0) + ⋅⋅⋅


2! n!
This series is called Maclaurin’s series.
g

1.30 LAURENT’S SERIES [U.P.T.U. (C.O.) 2008]


.c

If f(z) is analytic inside and on the boundary of the annular (ring shaped) region R bounded by
w

two concentric circles C1 and C2 of radii r1 and r2 (r1 > r2) respectively having centre at a, then
for all z in R,
w

∞ ∞
f(z) = ∑ an (z – a)n + ∑ b (z − a) −n
w

n
n=0 n=1

where, an =
1
2πi z C1
f(w)
(w − a)n +1
dw ; n = 0, 1, 2, ...

and bn =
1
2πi z
(w − a) − n +1
C2
f(w)
dw ; n = 1, 2, 3, ...

Proof. Let z be any point in the region R, then by Cauchy’s integral formula for double connected
region, we have

f(z) =
1
2πi 1 w − z
C z
f (w)
dw −
1 f (w)
2πi 2 w − z
C
dw z ...(1)

www.cgaspirants.com
www.cgaspirants.com

FUNCTION OF COMPLEX VARIABLE 67

For the first integral in (1), w lies on C1


C1
z−a C2
∴ |z–a|<|w–a| i.e., <1
w− a
z

Now
1
=
1
=
1
1−
z−a FG IJ −1 r1 a
r2
w − z (w − a) − ( z − a) w − a w− a H K
LM FG IJ 2 O R
+ ⋅ ⋅ ⋅P
1 z−a z−a
= 1+ +
w− a w− a MN
w− a H K PQ
1
Multiplying both sides by f (w) and integrating term by term w.r.t. w, along the
2πi

m
circle C1, we get
1
z f (w) 1
z f (w) z−a
z f (w) ( z − a) 2
z f (w)

o
dw = dw + dw + dw
2πi C1 w − z 2πi C1 w − a 2πi C1 (w − a) 2 2πi C1 (w − a) 3

.c
+ ⋅⋅⋅
= a0 + a1(z – a) + a2(z – a)2 + ⋅⋅⋅

= ∑a

n=0
n (z
n − a) n ...(2)
LM∵
MN an =ts 1
2πi z
C1
f (w)
(w − a) n +1
dw, n = 0, 1, 2, ⋅ ⋅ ⋅
OP
PQ
ra
For the second integral in (1), w lies on C2
w− a
∴ | w – a | <| z – a | i.e., <1
pi

z−a

Now
1 1 1 w−a FG IJ −1
as

= =− 1−
w − z (w − a) − ( z − a) z−a z−a H K
1 w− a LM
w− a FG IJ 2 OP
g

=– 1+ + + ⋅⋅⋅
z−a z−a MN
z−a H K PQ
.c

1
w

Multiplying both sides by – f (w) and integrating term by term w.r.t. w, along the
2πi
circle C2, we get
w


1
z f (w)
dw =
1

1
z f (w) dw +
1

1
z (w – a) f(w) dw
w

2
2πi C2 w− z z − a 2πi C2 ( z − a) 2πi C2

+
( z − a)
1
3

1
2πi z
C2
(w – a)2 f(w) dw + ⋅⋅⋅

= b1(z – a)–1 + b2(z – a)–2 + b3(z – a)–3 + ⋅⋅⋅

=

∑ b ( z − a)
n=1
n
−n
...(3)
LM∵
MN bn =
1
2πi zC2
f (w)
(w − a) − n +1
dw, n = 1 , 2, 3, ⋅ ⋅ ⋅
OP
PQ
Substituting from (2) and (3) in (1), we get
∞ ∞
f(z) = ∑
n=0
an ( z − a) n + ∑ b ( z − a)
n=1
n
−n

www.cgaspirants.com
www.cgaspirants.com

68 A TEXTBOOK OF ENGINEERING MATHEMATICS

Note 1. In case f(z) is analytic inside C1, then bn = 0 and an =


1
2πi z
C1
f (w)
(w − a) n+1
dw =
f n(a)
n!
and Laurent’s series reduces to Taylor’s series.
Note 2. If C is any simple closed curve which lies in the ring-shaped region R and encloses the circle C1,
then

z C1
f (w)
(w − a) n+1
dw =
z
C
f (w)
(w − a) n + 1
dw

and z C2
f (w)
(w − a) − n+1
dw =
z
C
f (w)
(w − a) − n + 1
dw

m
∴ Laurent’s series can be written as

z
C

1 f (w)
f(z) = ∑ an ( z − a) n , where an = dw .

o
2πi C (w − a) n + 1
n=−∞

.c
Example 1. Expand
1
n EXAMPLES

in the region
ts
z 2 − 3z + 2
ra

(a) | z | < 1 (b) 1 < | z | < 2 (U.P.T.U. 2015)


(c) | z | > 2 (d) 0 < | z – 1 | < 1. (G.B.T.U. 2006, 2008, 2010)
pi

1 1 1 1
Sol. Here f(z) = = = − | Partial Fractions
z2 − 3z + 2 ( z − 1)( z − 2) z − 2 z − 1
as

(a) When | z | < 1


1 1
g

f(z) = +

FG z IJ 1− z
.c

− 2 1−
H 2 K
[Arranged suitably to make the binomial expansions valid]
w

1 FG
z IJ −1
1
∑ FGH 2 IJK + ∑

z
n ∞
w

zn
=–
2
1−
2H K + (1 – z)–1 = –
2 n=0 n=0
w

This is a series in positive powers of z, so it is an expansion of f(z) in Taylor’s series


within the circle | z | = 1.
(b) When 1 < |z| < 2
1 1 1 FGz IJ −1
1FG 1 IJ −1
∴ f(z) =
FG z IJ −
FG 1 IJ =−
2
1−
H2 K −
z H
1−
z K
− 2 1−
H 2 K H
z 1−
z K
∑ FGH 2 IJK ∑ FGH z IJK
∞ n ∞ n
1 z 1 1
=– −
2 n=0
z n=0
This is a series in positive and negative powers of z, so it is an expansion of f(z) in
Laurent’s series within the annulus 1 < | z | < 2

www.cgaspirants.com
www.cgaspirants.com

FUNCTION OF COMPLEX VARIABLE 69

(c) When |z| > 2

f(z) =
1 1 1FG 2 IJ −1
1 FG
1 IJ −1

FG 2
JKI

FG 1
JKI
=
z H
1−
z K −
z
1−
H
z K
H
z 1−
z
z 1−
H z

∑ FGH z IJK ∑ FGH z IJK


∞ n ∞ n
1 2 1 1
= −
z n=0 z n=0
This is Laurent’s series within the annulus 2 < | z | < R, where R is large.
(d) When 0 < |z – 1| < 1
1 1 1 1
f(z) = − =− − = – (z – 1)–1 – [1 – (z – 1)]–1

m
( z − 1) − 1 z − 1 1 − ( z − 1) z − 1

1
=– − ∑
( z − 1) n .

o
z − 1 n=0

.c
This is also Laurent’s series within the annulus 0 < | z – 1 | < 1.

ts ∑ (n + 1)(z + 1) n
Example 2. Show that when | z + 1 | < 1, z–2 =1+ .
n=1
n
1 1 1
Sol. f(z) = z–2 = 2
= 2
= = [1 – (z + 1)]–2
z [( z + 1) − 1] [1 − ( z + 1)]2
ra

= 1 + 2(z + 1) + 3(z + 1)2 + 4(z + 1)3 + ⋅⋅⋅


[By binomial theorem, since | z + 1 | < 1]
pi


=1+ ∑ (n + 1)( z + 1) n
.
as

n=1
π
Example 3. Expand cos z in a Taylor’s series about z = .
4
g

Sol. Here f(z) = cos z, f ′(z) = – sin z, f ″(z) = – cos z, f ″′(z) = sin z, ...
.c

FG π IJ = 1 FG π IJ = − 1 FG π IJ = − 1 FG π IJ = 1
∴ f
H 4K 2
, f′
H 4K , f″
H 4K , f ″′
H 4K , ...
w

2 2 2
Hence cos z = f(z)
FG z − π IJ FG z − π IJ
w

2 3

F πI F πI F πI H 4K F πI H 4K FG π IJ + ...
= f G J + Gz − J f ′ G J +
H 4 K H 4 K H 4 K 2 ! f ″ GH 4 JK + 3 !
w

f ″′
H 4K
1 L F π I 1 FG z − π IJ + 1 FG z − π IJ + ⋅ ⋅ ⋅OP 2 3
= M
2 MN
1 − Gz − J −
H 4 K 2 ! H 4 K 3 ! H 4 K PQ
sin z
Example 4. Expand the function about z = π.
z−π
Sol. Putting z – π = t, we have
sin z sin (π + t) − sin t 1 t3 t5 F I
z−π
=
t
=
t
=–
t
t− +
3! 5!
⋅⋅⋅ GH JK
t2 t4 ( z − π) 2 ( z − π) 4
= −1+ − + ⋅⋅⋅ = − 1 + − + ⋅⋅⋅
3! 5! 3! 5!

www.cgaspirants.com
www.cgaspirants.com

70 A TEXTBOOK OF ENGINEERING MATHEMATICS

z
Example 5. Expand f(z) = about z = – 2.
(z + 1)(z + 2)
Sol. To expand f(z) about z = – 2, i.e., in powers of z + 2, we put z + 2 = t.
z t−2 2−t 2−t
∴ f(z) = = = = (1 – t)–1
( z + 1)( z + 2) (t − 1) t t(1 − t) t
2−t
= (1 + t + t2 + t3 + ⋅⋅⋅) for 0 < | t | < 1
t
1 2
= (2 + t + t2 + t3 + ⋅⋅⋅) = + 1 + t + t2 + ⋅⋅⋅
t t
2
= + 1 + (z + 2) + (z + 2)2 + ⋅⋅⋅ for 0 < | z + 2 | < 1
z+2

m
which is Laurent’s series.
Example 6. Expand the following function in a Laurent’s series:

o
ez 1

.c
(i) f(z) = about z = 1. (ii) f(z) = for | z – 1 | < 1
(z − 1) 2 z(z − 1) (z − 2)

Sol. (i)

Put
f(z) =
ez
( z − 1) 2
z – 1 = t then
n
z=1+t
ts
LM1 + t + t + t + ⋅ ⋅ ⋅OP = e L 1 + 1 + 1 + t + ⋅ ⋅ ⋅O
ra
1+ t 2 3
e e
∴ f(z) =
t2
=
t2 MN 1 ! 2 ! 3 ! PQ MN t t 2 ! 3 ! PQ 2

LM 1 O
pi

1 1 z−1
=e + + + + ⋅ ⋅ ⋅P .
MN (z − 1) 2
z −1 2! 3! PQ
as

1 1 1 1
(ii) f(z) = = − + |Partial fractions
z ( z − 1) ( z − 2) 2 z z − 1 2 ( z − 2)
g

1 1 1
= − +
.c

2 ( z − 1 + 1) z − 1 2 ( z − 1 − 1)
1 1 1
w

= {1 + (z – 1)}–1 − − {1 – (z – 1)}–1 |∵ |z – 1| < 1


2 z−1 2
w

∞ ∞
1 1 1
=
2 ∑ (− 1) n ( z − 1) n − −
z−1 2 ∑ (z − 1) n
w

n=0 n=0
This is a series in positive and negative powers of (z – 1) hence it is an expansion of f(z)
in a Laurent’s series for |z – 1| < 1.
Example 7. Expand the following function in a Laurent’s series about the point z = 0:
1 − cos z
f(z) = .
z3

Sol. f(z) =
1 − cos z 1
= 3 1− 1−
z2 z4
+
LM
− ⋅⋅⋅
|RS |UVOP
z 3
z 2! 4! MN |T |WPQ
1 F z − z + z – ⋅ ⋅ ⋅I = 1 − 1 z + 1 z
2 4 6
=
z3
GH 2 ! 4 ! 6 ! JK 2 ! z 4 ! 6 ! 3 – ⋅⋅⋅

www.cgaspirants.com
www.cgaspirants.com

FUNCTION OF COMPLEX VARIABLE 71

1
Example 8. Find the terms in the Laurent’s expansion of for the region
z(e z − 1)
0 < | z | < 2π.
1 1
Sol. f(z) = =
z
z(e − 1) LM
z 1+ z +
z
+
2
z3
+ ... − 1
OP
MN 2! 3! PQ
F z + z + ...I = z F1 + z + z + ...I
2 3 −1 2 −1

= z–1 z + GH 2 ! 3 ! JK GH 2 ! 3 ! JK
–2

LM F z z z z I 1 F z z I 2

m
2 3 4 2

MN1 − GH 2 + 6 + 24 + 120 + ...JK + 4 z GH1 + 3 + 12 + ...JK


2
= z–2

o
1 F z I z (1 + ...) + ...OP
− z G 1 + + ...J +
3 4

.c
3 4
8 H 3 K 16 PQ
LM1 − z + z FG 1 − 1IJ − z FG 1 − 1 + 1 IJ
= z–2 2

N 2 H 4 6 K H 8 6 24 K
n 3
ts
F 1 − 1 + 1 + 1 − 1 IJ + ...OP
+z G
H 16 8 24 36 120 K Q
4
ra

LM1 − z + z − z + ...OP = z – 1 z + 1 − z + ...


2 4
−1
2
pi

= z–2 –2
MN 2 12 720 PQ 2 12 720
as

1
The singularities of are given by z = 0, ez = 1 i.e., z = 0, ± 2πi, ± 4πi, ....... .
z
z(e − 1)
Hence the above expansion is valid for the region 0 < | z | < 2π.
g

Example 9. Using Taylor’s theorem, show that:


.c

(z − 1) 2 (z − 1) 3
log z = (z – 1) – + – ... where | z – 1 | < 1.
w

2 3
Sol. f(z) = log z, f(1) = 0 |∵ a = 1 and log 1 = 0
w

1
Now, f ′(z) = , f ′(1) = 1
z
w

1
f ″(z) = – 2 , f ″(1) = – 1
z
2
f ″′(z) = 3 , f ″′(1) = 2
z
−6
f (iv) (z) = 4 , f (iv) (1) = – 6 and so on.
z
We know that,
( z − a) 2 ( z − a) 3
f(z) = f(a) + (z – a) f ′(a) + f ″ (a) + f ″′(a) + ⋅⋅⋅
2! 3!
( z − 1) 2 ( z − 1) 3
= f(1) + (z – 1) f ′(1) + f ″ (1) + f ″′(1) + ⋅⋅⋅
2! 3!

www.cgaspirants.com
www.cgaspirants.com

72 A TEXTBOOK OF ENGINEERING MATHEMATICS

( z − 1) 2 ( z − 1) 3
= 0 + (z – 1) (1) + (– 1) + (2) + ⋅⋅⋅
2! 3!
( z − 1) 2 ( z − 1) 3
= (z – 1) – + – ⋅⋅⋅
2 3
Example 10. Find the Taylor’s or Laurent’s series which represent the function
1
2
when
(1 + z )(z + 2)
(i) | z | < 1 (ii) 1 < | z | < 2 (iii) | z | > 2.

Sol. Let f(z) =


1
=
1 RS
1

z−2 UV
2
(1 + z )( z + 2) T
5 z + 2 1 + z2 W

m
(i) | z | < 1
1 1 1 FG 2− z IJ −1

o
f(z) = .
5 2
1+ z
2 H +
5
(1 + z2)–1
K

.c
| Binomial expansion of (1 + z)–1 is valid only when | z | < 1

FG z IJ
ts
∞ n ∞
1 2− z
=
10 ∑ (− 1) n
H 2K +
5 ∑ (− 1) n
z2n
n
0 0
This is a series in positive powers of z, so it is an expansion of f(z) in a Taylor’s series
ra
within the circle | z | = 1.
∞ ∞

Remark. If | z | < 1, (1 + z)–1 = ∑ (− 1) n z n ; (1 – z)–1 = ∑z n


.
pi

n=0 n=0
(ii) 1 < | z | < 2
as

1 1 1 FG IJ −1
2− z 1 1 FG IJ −1
f(z) = .
5 2
1+ z
2 H K +
5
. 2 1+ 2
z z H K
g

1


FG z IJ n
2– z


FG 1 IJ n
.c

(– 1) n (– 1) n
=
10 0
H 2K +
5 z2 0
Hz K2
w

This is a series in positive and negative powers of z, so it is an expansion of f(z) in a


Laurent’s series within the annulus 1 < | z | < 2.
w

(iii) | z | > 2

FG IJ –1
FG IJ –1
w

1 1 2 1 1 1
f(z) = .
5 z
1+
z H K –
5
( z – 2) 2 1 + 2
z z H K
1
∑ (− 1) FGH z IJK

2 n
n
FG
1 1 2 IJ ∑ (− 1) FG 1 IJ

n
n
=
5z 0

H−
5 z z2 K 0
Hz K 2

This is Laurent’s series within the annulus 2 < | z | < R, where R is large.
Example 11. Find the Taylor’s and Laurent’s series which represent the function
z2 − 1
when (U.K.T.U. 2011)
(z + 2)(z + 3)
(i) | z | < 2 (ii) 2 < | z | < 3 (iii) | z | > 3.

www.cgaspirants.com
www.cgaspirants.com

FUNCTION OF COMPLEX VARIABLE 73

z2 − 1 3 8
Sol. Let f(z) = =1+ − .
( z + 2)( z + 3) z+2 z+3
(i) | z | < 2

3 FG
z IJ −1
8 FG
z IJ −1
f(z) = 1 +
2
1+
2H K −
3
1+
H
3 K
3


(− 1) n
FG z IJ n
8


(− 1) n
FG z IJ n
=1+
2 0
H 2K −
3 0
H 3K
It is a Taylor’s series within a circle | z | = 2.
(ii) 2 < | z | < 3

m
3 2 8FG z IJ −1
FG IJ −1
f(z) = 1 +
1+ −
H
1+
K H K

o
z z 3 3
| Arranging suitably to make the binomial expansion valid for 2 < | z | < 3

.c
3


(− 1) n
FG 2 IJ n
8


(− 1) n
FG z IJ n

H zK H 3K
ts
=1+ −
z 0
3 0
It is a Laurent’s series within the annulus 2 < | z | < 3.
n
(iii) | z | > 3
ra

3 FG
2 IJ −1
8 FG
3 IJ −1
f(z) = 1 +
z
1+
H
z K −
z
1+
H
z K
pi

3


(− 1) n
FG 2 IJ n
8


(− 1) n
FG 3 IJ n
=1+
z H zK −
z H zK
as

0 0
It is a Laurent’s series within the annulus 3 < | z | < R, where R is large.
1
g

Example 12. Expand in the regions


(z + 1)(z + 3)
.c

(i) | z | < 1 (ii) 1 < | z | < 3


(iii) | z | > 3 (iv) 1 < | z + 1 | < 2.
w

Sol. f(z) =
1
=
1 1

1 LM OP
N Q
w

( z + 1)( z + 3) 2 z + 1 z + 3
(i) | z | < 1
w

1 LM
(1 + z) −1 −
1 z FG IJ OP = 1 LM∑ (− 1)
−1 ∞
n
zn −
1

∑ (− 1) n
FG z IJ OP
n
f(z) =
2 MN 3
1+
3 H K PQ 2 MN 0
3 0
H 3 K PQ
It is a Taylor’s series within a circle | z | = 1.
(ii) 1 < | z | < 3
1 1 LM FG
1 IJ −1
1 FG
z IJ −1 OP
MN H K H K
f(z) = . 1+ − 1+
2 z z 3 3 PQ
LM (− 1) F 1I
1 1
∞ n
1

FG z IJ OP
n
=
2 zMN ∑ GH z JK n

3 ∑ (− 1) n
H 3 K PQ
0 0
It is a Laurent’s series within the annulus 1 < | z | < 3.

www.cgaspirants.com
www.cgaspirants.com

74 A TEXTBOOK OF ENGINEERING MATHEMATICS

(iii) | z | > 3
LM FG IJ − 1 FG 1 + 3 IJ OP
1 1 1
−1 −1

MN H K z H z K PQ
f(z) = 1+
2 z z

1 L1 F 1I 1 F 3I O
∞ n ∞ n
= M ∑ (− 1) G J − ∑ (− 1) G J P n n
2 MN z H zK z
0
H z K QP 0

It is a Laurent’s series within the annulus 3 < | z | < R where R is large.


(iv) 1<|z+1|<2
⇒ 1<|u|<2 where z + 1 = u
1 1 LM1 1 1 1 OP LM OP

m
f(z) = − = −
N
2 z+1 z+3 2 u u+2 Q N Q

o
1
.
2 1 1 u FG IJ −1
1


(− 1) n
FG u IJ n
= = = 1+
H K =
H 2K

.c
2 u(u + 2) u(u + 2) 2u 2 2u 0

∑ (− 1) FGH 2 IJK
∞ n
=
1
2( z + 1)
n 0
It is Laurent’s series in the annulus 1 < | z + 1 | < 2.
z+1 n

ts
ra
Example 13. Find the Laurent’s expansion for:
7z − 2
f(z) = (U.K.T.U. 2010)
pi

z − z 2 − 2z
3

in the regions given by:


as

(i) 0 < | z + 1 | < 1 (ii) 1 < | z + 1 | < 3 (iii) | z + 1 | > 3.


Sol. We have
g

7z − 2 1 3 2 1 3 2
f(z) = 3 2
= − + = − +
z − z − 2z z z+1 z−2 ( z + 1) − 1 z + 1 ( z + 1) − 3
.c

(i) 0 < | z + 1 | < 1


w

f(z) = – {1 – (z + 1)}–1 –
3 2 RS FG
z+1 IJ UV −1

z+1 3 T H
1−
3 KW
w

3

( z + 1) n

2
− ∑G
F z + 1IJ
∞ n
w

=– −
z + 1 n=0 3 H 3 K
n=0

This is a series in negative and positive powers of (z + 1) hence it is an expansion of f(z)


in Laurent’s series within the annulus 0 < | z + 1 | < 1.
(ii) 1 < | z + 1 | < 3

f(z) =
1 1 FG
IJ −1
3 RS FG IJ UV
2 z+1
−1

z+1
1−
z+1 H
K − −
z+1 3T H KW
1−
3

1

FG 1 IJ ∞ n
3 2
− ∑G
F z + 1IJ ∞ n
=
z+1 H z + 1K
n=0

z+1 3 H 3 K n=0

www.cgaspirants.com
www.cgaspirants.com

FUNCTION OF COMPLEX VARIABLE 75

This is also a series in negative and positive powers of (z + 1) hence it is an expansion of


f(z) in Laurent’s series within the annulus 1 < | z + 1 | < 3.
(iii) | z + 1 | > 3

f(z) =
1
1−
1 FG IJ −1

3
+
2
1−
3 IJ
FG −1

z+1 z+1 H K z+1 z+1 z+1 H


K
=
1

FG 1 IJ ∞ n

3
+
2

FG 3 IJ ∞ n

z+1 H z + 1K
n=0
z+1 z+1 H z + 1K
n=0

This is a series in negative powers of (z + 1) hence it is an expansion of f(z) in Laurent’s


series within the annulus 3 < | z + 1 | < R where R is large.

m
1
Example 14. (i) Obtain the Taylor’s series expansion of f(z) = 2 about the
z − 4z + 3

o
point z = 4. Find its region of convergence.

.c
1
(ii) Obtain Taylor’s series expansion of f(z) = 2 about the point z = – i. Find the
z +4
region of convergence.
ts (U.P.T.U. 2006)
Sol. (i) If the centre of the circle is at z = 4, then the distances of the singularities z = 1
n
and z = 3 from centre are 3 and 1. Hence if a circle is drawn with centre at z = 4 and radius 1
then within a circle | z – 4 | = 1, the given function f(z) is analytic hence it can be expanded in
ra

Taylor’s series within the circle | z – 4 | = 1 which is therefore the circle of convergence.
pi

Y
as

Cº|z–4|=1
g

O z=1 z=2 z=3 z=4 X


.c
w
w

1 1 1 1 LM1 1 OP1 LM OP
w

f(z) = = − = −
( z − 1)( z − 3) 2 z−3 z−1 N Q
2 z−4+1 z−4+3 N Q
LM1
{1 + ( z − 4)} −1
1R F z − 4 IJ UV
− S1 + G
−1 OP
=
MN
2 3T H 3 KW PQ
1L FG z − 4 IJ OP
∞ ∞ n
f(z) = M ∑ (− 1)
1

2M
n
( z − 4) n − ∑ (− 1) n
H 3 K PQ
N n=0
3 n=0

www.cgaspirants.com
www.cgaspirants.com

76 A TEXTBOOK OF ENGINEERING MATHEMATICS

(ii) If the centre of the circle is at z = – i, then


the distances of the singularities z = 2i and z = – 2i
from centre are 3 and 1 respectively.
Hence if a circle is drawn with centre at z = – i and
radius 1 then within a circle |z + i| = 1, the given
function f(z) is analytic hence it can be expanded in
Taylor’s series within the circle |z + i| = 1 which is
therefore the circle of convergence.
1 1
f(z) = 2
=
z +4 ( z − 2i) ( z + 2i)

FG
1 1 1 IJ

m
= −
H
4i z − 2i z + 2i K
1 L OP

o
1 1
= M −
4i N ( z + i) − 3i ( z + i) + i Q

.c
1 L− 1 R F z + i IJ UV − 1 RS1 + FG z + i IJ UV OP
−1 −1
M
=
4i M 3i T
N
S 1− G
H 3i K W i T H i K W
n ts OP
PQ
1 Li FG z + i IJ + i ∑ ( − 1) FG z + i IJ
∞ n ∞ n
M ∑
ra
n

N H 3i K H i K
=
4i M 3 n=0 n=0
PQ
1 L1 F z + i IJ + ∑ ( − 1) FG z + i IJ OP
pi

∞ n ∞ n
= M ∑ G n

N H 3i K
4 M3 n=0
H i K PQ
n=0
as

Example 15. (i) If the function f(z) is analytic and one-valued in | z – a | < R, prove that
for 0 < r < R,
g

1
z 2π
.c

f ′(a) = P( θ ) e − iθ dθ where P(θ) is the real part of (a + reiθ).


πr 0
w

1 FG 1 IJ ∞
H
z t−
K
(ii) Prove that: e 2 t
= ∑ J n (z) t n , | t | > 0
w

n=−∞

z
w

1 π
where Jn (z) = cos (nθ − z sin θ) dθ [G.B.T.U. (C.O.) 2008]
π 0
Sol. (i) ∵ f(z) is regular in | z – a | < R
∴ f(z) is regular in | z – a | = r |∵ r<R
∴ f(z) can be expanded in a Taylor’s series within the circle | z – a | = r. Thus,

f(z) = ∑a
0
m (z − a) m where z – a = reiθ


= ∑a
0
m r m e miθ ...(1)

www.cgaspirants.com
www.cgaspirants.com

FUNCTION OF COMPLEX VARIABLE 77


⇒ f ( z) = ∑a 0
m r m e − miθ ...(2)

Now, z
C
f ( z)
dz
( z − a) n + 1
= z
0
2π ∞
∑a
0
m r m e − miθ
r
re iθ idθ
n + 1 i( n + 1)θ
e

=

∑a
0
m r m− n i z0

e − i ( m + n ) θ dθ

= 0, for all values of n ...(3)

Particularly, z f ( z)
dz
=0 ...(4)

m
C ( z − a) 2
1
z f ( z) dz 1
z f ( z) + f ( z )

o
We know that f ′(a) = = dz | Using (4)
2πi C ( z − a) 2 2πi C ( z − a) 2

.c
=
1
2πi z 2π

0
f (a + re iθ ) + f (a + re iθ )
2
r e
ts
2 iθ
re iθ idθ

z 2 Re (a + re iθ )
n
1 2π
= dθ |∵ z + z = 2 Re (z)
2π 0 re iθ
ra

=
1
πr z 2π

0
P(θ) e − iθ dθ where P(θ) = Re (a + reiθ).
pi

1 FG
1 IJ
(ii) The function e 2
z t−
tH K is analytic everywhere in the t-plane except at t = 0 and t = ∞
as

i.e., it is analytic in the ring shaped region r ≤ | t | ≤ R where r is small and R is large.
Therefore this function can be expanded in Laurent’s series in the form
g

1 FG1 IJ ∞ ∞
z t−
H K
2 t
∑ an t n + ∑ bn t − n
.c

e =
n=0 n=1
w

z z
1 FG1 IJ 1 FG 1 IJ
where an =
1
e 2
z t−
Ht K dt
and bn =
1
e 2 H
z t−
t K dt
w

n+1 − n+ 1
2πi C t 2πi C t
where C is any circle with centre as origin.
w

Taking C ≡ | t | = 1 so that t = eiθ and dt = ieiθ dθ, we get

an =
1
2πi z 2π

0
e2
z
( eiθ − e − iθ ) ie iθ dθ
e( n + 1) iθ

=
1
2π z
0

e iz sin θ e − niθ dθ =
1
2π z0

e − i( nθ − z sin θ) dθ

=
1
2π z
0

cos (nθ − z sin θ) dθ | Since second part vanishes

⇒ an =
1
π zπ

0
cos (nθ − z sin θ) dθ | Using prop. of definite integrals

www.cgaspirants.com
www.cgaspirants.com

78 A TEXTBOOK OF ENGINEERING MATHEMATICS

1 1 FG IJ
Clearly, the function e 2
z t−
t H K remains unaltered if t is replaced by –
1
so that
t
bn = (– 1)n an. Therefore,
1 FG 1 IJ ∞ ∞
H
z t−
K
e 2 t
=
n=0
∑ an t n + ∑
n=1
bn t − n

∞ ∞ ∞
= ∑
n=0
an t n + ∑
n=1
(− 1) n an t − n = ∑
n= − ∞
an t n

1 FG 1 IJ ∞

m
H
z t−
K
Here, an is Jn (z) hence, e 2 t
= ∑
n=−∞
J n ( z) t n

o
1 π
where, Jn (z) = cos (nθ − z sin θ) dθ.

.c
π 0

FG IJ ∑ a FGH z IJ

1 n 1

where
Example 16. Prove that cosh z +

an =
1
2π z
0

cosh θ cosh (2 cos θ) dθ.
n H z K
= a0 +
1
ts n +
zn K
(M.T.U. 2013)
ra

1 FG IJ
Sol. The function f(z) = cosh z +
z H
is analytic everywhere in the finite part of the
K
pi

plane except at z = 0 i.e., it is analytic in the annulus r ≤ | z | ≤ R where r is small and R is


large. Hence f(z) can be expanded in Laurent’s series in the annulus r < | z | < R. Thus,
as

FG 1IJ ∑ a z + ∑ b z

n

−n
H
cosh z +
z K
= n n
g

0 1

F 1I FG IJ
.c

1
cosh G z + J
H z K dz
z z H K dz
cosh z +
1 1 z
where a = and bn =
w

n
2πi C z n + 1
2πi C z − n+1
where C is any circle lying in the annulus with origin as centre.
w

FG 1 IJ
z H K dz
cosh z +
w

1 z
an =
2πi C z n+1

=
1
2 πi z 0
2π cosh (2 cos θ) ie iθ dθ
e i(n + 1)θ
| Take C as a circle | z | = 1 on which z = eiθ

=
1
2π z 2π

0
cosh (2 cos θ) e–inθ dθ

=
1
2π z 2π

0
cosh (2 cos θ) cos nθ dθ ∵ z0

cosh (2 cos θ) sin nθ dθ = 0

www.cgaspirants.com
www.cgaspirants.com

FUNCTION OF COMPLEX VARIABLE 79

bn = a–n

=
1
2π z 0

cosh (2 cos θ) cos (– nθ) dθ

=
1
2π z 0

cosh (2 cos θ) cos nθ dθ = an

FG 1 IJ ∑ a z + ∑ b z

n

−n

∑ an z n +

∑a z −n
Hence, cosh z +
H z
=
K 0
n
1
n =
0 1
n |∵ a n = bn

∞ ∞ ∞

∑ ∑ ∑ a (z

m
= a0 + an z n + an z − n = a0 + n
n
+ z−n ) .
1 1 1

o
Fa I
z
n 2
1 a n e az
Example 17. If C is a closed contour around origin, prove that G J

.c
= dz
H n !K 2 πi C n ! z n+1

F a I 1 e dθ.
Hence deduce ∑ G J =

H n ! K 2π
0
n 2
n
z 2π

0
2a cos θ
ts
ra
Sol. Let f(z) = eaz
∴ f n(z) = an eaz
∴ f n(0) = an
pi

⇒ an =
1
n! z f ( z) dz
as

2πi C z n+ 1

F a I = 1 1 a e dz
z
n 2 n az

GH n ! JK 2πi n ! z
g

⇒ n+1
C
.c

Fa I
z z
∞ n 2 ∞ n az ∞ n az
1 a e 1 a e
∑ GH n ! JK = ∑ 2πi n ! z dz = 2πi ∑ n ! z dz
w

⇒ n+1 n+ 1
C C
0 0 0
w

=
1
e ∑
a
z
1
dz =
1 R| F a I

e S∑ G J
az

|T H z K
n

z az
∞ n
1 dz U|
V|
w

2πi n! z C 2πi 0
n+1
C
0
n! z W
z
FG IJ
z
1

=
1
e az
e ( a / z) dz
=
1
e
H
a z+
z K dz
2πi C z 2 πi C z

=
1
2 πi z 0

e 2 a cos θ
i e iθ dθ
e iθ
where the circle C is taken as
| z|= 1 so that z = e iθ on C
=
1
2π z 0

e 2 a cos θ dθ
∴ dz = i e iθ dθ

www.cgaspirants.com
www.cgaspirants.com

80 A TEXTBOOK OF ENGINEERING MATHEMATICS


sin θ
Example 18. Prove that for real k, k2 < 1; ∑
n=0
k n sin (n + 1) θ =
1 − 2 k cos θ + k 2

cos θ − k
and ∑
n=0
k n cos (n + 1) θ =
1 − 2k cos θ + k 2
.

Sol.
1 1 k FG IJ −1
1
∑ FGH z IJK

k
n



kn
;|z|>k ...(1)
=
z−k z
1−
z H K =
z n=0
=
n=0
z n+1
Again, put z = e iθ in (1),

1 1

k n e– (n+ 1) iθ

m
= =
z − k e iθ − k n = 0

o

1
⇒ =
cos θ + i sin θ − k n = 0 ∑
k n [cos (n + 1) θ – i sin (n + 1) θ]

.c

(cos θ − k) − i sin θ

1 − 2 k cos θ + k 2
=
n=0
n ∑
k n [cos (n + 1) θ – i sin (n + 1) θ]

Comparing real and imaginary parts of (2), we get the required results.
ts ...(2)
ra
1 1 7
Example 19. (i) Show that cosec z = + z+ z 3 + ...... ; 0 < | z | < π.
z 3! 360
pi

a
(ii) Find the Taylor’s series expansion of f(z) = about the point z0.
bz + c
as

1
Sol. (i) cosec z = has singular points at z = 0, ± nπ.
sin z
g

We expand the series in 0 < | z | < π.


.c

1 1 1
cosec z = = =
sin z z z 3
5
z 7
z2 z4 z6 LM F
I OP
MN GH
JK PQ
w

z− + − + ... z 1− − + + ......
3! 5! 7! 3! 5! 7!
w

1L F z − z + z + ......I + F z − z + z + ......I + ......OP 2

= M1 + G
2 4 6 2 4 6

zM J G J PQ
N H 3! 5! 7! K H 3! 5! 7! K
w

1L z 2
z 4
z O 1 z + RS 1 − 1 UV z + ......
4
= M1 + − + + ......P = + 4
zN 3 ! 5 ! (3 !) Q z 3 ! T (3 !) 5 ! W
2 2

LM OP
(ii) f(z) =
a
=
a
bz + c b( z − z ) + bz + c
=
1
M a
bz + c M 1 + b( z − z ) P
P
0
MN bz + c PQ
0 0
0
0

aL 1 OP b
= M where bz + c = d, = e
d N 1 + e (z − z ) Q
0
0
d

www.cgaspirants.com
www.cgaspirants.com

FUNCTION OF COMPLEX VARIABLE 81


a
=
d ∑
n=0
(− 1) n e n ( z − z0 ) n if | e (z – z0) | < 1

a

F b I n
1
=
bz0 + c ∑
n=0
(− 1) n
GH bz + c JK
0
( z − z0 ) n if | z – z0 | <
e
.

4z − 1
Example 20. Find Taylor’s series expansion of about the point z = 0.
z4 − 1
(U.P.T.U. 2007; M.T.U. 2012)
4z − 1 4z − 1
Sol. f(z) = =

m
z − 1 ( z − 1) ( z + 1) ( z 2 + 1)
4

FG − 2z + 1IJ

o
=
3 1FG+
5 IJ +
H 2K
H K

.c
4 z−1 4 ( z + 1) z2 + 1
Expanding about the point z = 0, we get

f(z) = –
3
n
4
5
(1 − z) −1 + (1 + z) −1 + − 2 z +
4
ts
1
2
FG
H
IJ e1 + z j
K
2 −1

F 1I
ra
∞ ∞ ∞
3 5
∑ zn + ∑ (− 1) n z n + G − 2z + J ∑ (− 1) n z 2 n .
=–
4 n=0
4 n=0
H 2K n=0
pi

ASSIGNMENT
as

Expand the following functions as a Taylor’s series (1–3):


g

1. (i) log (1 + z) about z = 0 [U.P.T.U. (C.O.) 2008]


.c

(ii) tan–1 z in powers of z [U.P.T.U. (C.O.) 2009]


(iii) sin–1 z in powers of z (U.P.T.U. 2007)
w

π π
2. (i) sin z about z = (ii) tan–1z about z = (U.P.T.U. 2015)
4 4
w

z
3. about z = 2.
( z + 1)( z + 2)
w

Expand the following functions in Laurent’s series (4–6):


1 1 1
4. , for | z | > 2 5. , for 1 < | z | < 3 6. , for | z | > 2
z−2 z2 − 4 z + 3 z( z − 1)( z − 2)

2 z3 + 1
7. (i) Find Taylor’s expansion of about the point z = 1.
z ( z + 1)
z
( z − 2)
(ii) Define the Laurent series expansion of a function. Expand f (z) = e in a Laurent series
about the point z = 2. (U.P.T.U. 2009)

7-D:\Engineering Mathematics (Bali)\MTU Sem III-IV\MTU 1-7

www.cgaspirants.com
www.cgaspirants.com

82 A TEXTBOOK OF ENGINEERING MATHEMATICS

( z − 2)( z + 2)
8. Expand f(z) = in the region:
( z + 1)( z + 4)
(a) | z | < 1 (b) 1 < | z | < 4 (c) | z | > 4.
[U.P.T.U. (C.O.) 2008]
1
9. Expand the function f(z) = 2 about (i) z = – 1 (ii) z = 1
z −z−6
z2 − 6 z − 1
10. (i) Find the Laurent’s series expansion of the function f(z) = in the region
( z − 1)( z − 3)( z + 2)
3 < | z + 2 | < 5.
7z − 2
(ii) Find the Laurent’s series expansion of f(z) = in the region 1 < | z + 1| < 3.
z( z + 1) ( z + 2)

m
(G.B.T.U. 2012)
1
11. (i) Obtain the Taylor series expansion of f(z) = about z = 0
z2 + (1 + 2i) z + 2i

o
z

.c
(ii) Expand f (z) = is Laurent series valid for
( z − 1) (2 − z)
(a) | z – 1 | > 1 and (b) 0 < | z – 2 | < 1 (G.B.T.U. 2011, 2013)

(iii) Expand f(z) =

(a) | z – 1 | > 1
z
( z − 1) ( z − 2)
n ts
in Laurent’s series valid for region:

(b) 0 < | z – 2 | < 1 (M.T.U. 2014)


ra

1
12. Find Laurent’s series of f(z) = about its singular points. Determine the region of convergence.
z2 + 1
pi

1
13. Find all possible Taylor’s and Laurent’s series expansions of the function f (z) =
( z + 1) ( z + 2)2
about the point z = 1. Consider the regions
as

(i) | z – 1 | < 2 (ii) 2 < | z – 1 | < 3 (iii) | z – 1 | > 3


1 1
14. The series expansions of the functions and are
g

1− z z−1
FG IJ
.c

1 1 1 1 1
= 1 + z + z2 + ...... and = 1 + + 2 + ......
1− z z−1 z z z H K
w

Adding, we get (1 + z + z2 + ......) +


1 1 1 FG
1 + + 2 + ...... = 0
IJ
z z z H K
w

Is this result true? If not, give the reason.


7 z2 + 9 z − 18
w

15. Expand f(z) = in Laurent series valid for the regions:


z3 − 9 z
(i) 0 < | z | < 3 (ii) | z | > 3 (G.B.T.U. 2013)
z+4
16. If f(z) = , find Laurent’s series expansion in (i) 0 < | z – 1 | < 4 and (ii) | z – 1 | > 4.
( z + 3 ) ( z − 1)2
(M.T.U. 2013)
z
17. Expand f(z) = in Laurent series in 1 < | z | < 2. (G.B.T.U. 2011, 2012)
( z − 1) ( z 2 + 4 )
2

18. Find all Taylor and Laurent series expansion of the following function about z = 0.
− 2z + 3
f(z) = . (U.P.T.U. 2014)
z2 − 3 z + 2

www.cgaspirants.com
www.cgaspirants.com

FUNCTION OF COMPLEX VARIABLE 83

Answers

z2 z3 z4 z3 z5 z3 3 5
1. (i) z – + − + ...... (ii) z − + − ...... (iii) z + + z + ...
2 3 4 3 5 6 40

1 LM1 + F z − π I − 1 F z − π I − 1 F z − π I + ......OP2 3
2. (i)
2 MN
GH 4 JK 2 ! GH 4 JK 3 ! GH 4 JK PQ
F π I F π I 16 − 64π ( z – π/4) + ...
(ii) tan z = tan G J + G z – J .
–1 −1
2

H 4 K H 4 K π + 16 (π + 16) 2 2 2

FG 1 − 1IJ − FG 1 − 1 IJ (z − 2) + FG 1 − 1 IJ (z – 2) – ......
3.
H 2 3K H 2 3 K 3 2 H2 3 K 5 3
2

m
1
f(z) = ∑ G J

F 2I n
1
5. f(z) = − ∑ G J −
F z I 1 ∑ FG 1IJ ∞ n ∞ n
4.
z H zK 6 H 3 K 2z H z K

o
n=0 n=0 n=0

∑ FGH z IJK ∑ FGH z IJK


∞ ∞

.c
n n
1 1 1 1 2
6. f(z) = − +
2z z 2z
n=0 n=0

7. (i) f(z) = 2 z − 2 + ∑

n=0
(− 1) n ( z − 1) n +
n 1
2 ∑
n=0

(− 1) n ts
FG z − 1IJ
H 2 K
n
(ii) f(z) = e


n=0
1 FG
H
2
n! z − 2
IJ
K
n
ra

∑ (− 1) FGH 4 IJK
∞ ∞ n
z
8. (a) f(z) = 1 − ∑
n=0
(− 1) n zn −
n=0
n
pi

1

F 1I
(b) f(z) = 1 − ∑ (− 1) G J − ∑ (− 1) G J n F zI
n ∞
n
n

z H zK H 4K
as

n=0 n=0

(c) f(z) = 1 − ∑ (− 1) FG IJ − ∑ (− 1) FG IJ
∞ n ∞ n
1 1 4 n 4 n
z H zK z H zK
g

n=0 n=0

FG z + 1IJ − 1 ∑ (− 1) (z + 1)
.c

∞ n ∞
1
9. (i) f(z) = –
20 ∑ H 4 K 5
n n
w

n=0 n=0

1 FG z − 1IJ − 1 ∑ (− 1) FG z − 1IJ
∞ n ∞ n

10 ∑ H 2 K
n
(ii) f(z) = –
H 3 K
w

15
n=0 n=0

FG 3 IJ + 1 FG z + 2 IJ + 1 .
w

∞ n ∞ n
1
z + 2 ∑ H z + 2K 5 ∑ H 5 K
10. (i) f(z) =
z+2
n=0 n=0
∞ ∞
9 1 1 8 1
(ii) f(z) = −
z+1 z+1 ∑
n=0
( z + 1) n
∑ (− 1)

z+1
n=0
n
( z + 1) n

1 1
LM ∞
FG z IJ − ∑ (− 1) z OP
n ∞

MN ∑ (− 1) n n n
11. (i) f(z) =
1 − 2i 2i
n=0
H 2i K PQ
n=0
∞ ∞
2
(ii) (a) f(z) =
1
z−1

2
z−1 ∑
1
( z − 1) n
(b) f(z) =
n=0
∑ (− 1) n
( z − 2) n −
z−2
n=0

7-D:\Engineering Mathematics (Bali)\MTU Sem III-IV\MTU 1-7

www.cgaspirants.com
www.cgaspirants.com

84 A TEXTBOOK OF ENGINEERING MATHEMATICS

−1 2 F 1 I ∞
2
n ∞
(iii) (a) f(z) = +
z−1 z−1 ∑ GH z − 1 JK n=0
(b) f(z) =
z−2 ∑
− (− 1) ( z − 2) n n

n=0

1

F z − i IJ ;| z – i | < 2 n

2i ( z − i) ∑
12. (i) f(z) = (− 1) G n

n=0
H 2i K
−1 FG z + i IJ ; | z + i | < 2
∞ n

2i ( z + i) ∑ H 2i K
(ii) f(z) =
n=0

1

F z − 1IJ − 1 ∑ (− 1) FG z − 1IJ – 1 (− 1) (n + 1) FG z − 1IJ
n ∞ n ∞ n

2 ∑
(− 1) G
H 3 K 9∑
n n n
13. (i)
H 2 K 3 H 3 K

m
n=0 n=0 n=0

1

FG 2 IJ n
1 FG z − 1IJ – 1 ∑ (− 1) (n + 1) FG z − 1IJ
∞ n ∞ n

∑ (− 1) n ∑ (− 1) n n

o
(ii)
z−1
n =0
H z − 1K −
3 H 3 K 9
n=0
H 3 K n=0

.c
1

FG 2 IJ n
1 F 3 IJ – 1

F 3 I
n ∞ n
(iii) ∑ (− 1) n
H z − 1K −
z−1 ∑
(− 1) G
H z − 1K (z − 1) ∑ (− 1) (n + 1) GH z − 1JK
n n

14.
z−1
n=0 n=0

No. The first series is valid for | z | < 1 and the second series is valid for | z | > 1. There is no
common point where both the series are valid.
n ts 2
n=0
ra

∑ (− 1) FGH 3 IJK ∑ FGH 3 IJK FG 3 IJ F 3I


∞ n ∞ n ∞ n ∞ n
2 1 z 4 z 2 1 4
15. (i) f(z) = +
z 3
n

3
(ii) f(z) = +
z z ∑ ( − 1)n
H zK +
z ∑ GH z JK
pi

n=0 n=0 n=0 n=0

∑ (− 1) FGH 4 IJK
∞ n
1 z−1 n 1 5
as

16. (i) f(z) = − +


64 16 ( z − 1) 4( z − 1) 2
n=0

F 4 I
g

∞ n
1 1 5
(ii) f(z) = ∑ (− 1) n GH z − 1 JK − +
.c

16( z − 1) 16( z − 1) 4( z − 1) 2
n=0

LM OP
w

F 1I FG 1 IJ
∑ FGH 2i IJK FG z IJ
∞ n ∞ n ∞ n ∞ n
1 1 1
MMN ∑ GH z JK ∑ 1 z 1
17. f(z) =
10 z
+
z
(− 1) n
H zK +
2i

2i ∑ (− 1) n
H 2i K PPQ .
w

n=0 n=0 n=0 n=0

∑ FGH 2 IJK
w

∞ ∞ n
1 z
18. (i) f(z) = ∑
n=0
zn +
2
n=0
;|z|<1

∑ FGH z IJK ∑ FGH 2 IJK


∞ n ∞ n
1 1 1 z
(ii) f(z) = – + ;1<|z|<2
z 2
n=0 n=0

∑ FGH z IJK ∑ FGH z IJK


∞ n ∞ n
1 1 1 2
(iii) f(z) = – − ;|z|>2
z z
n=0 n=0

www.cgaspirants.com
www.cgaspirants.com

FUNCTION OF COMPLEX VARIABLE 85

1.31 ZERO OF AN ANALYTIC FUNCTION [U.P.T.U. (C.O.) 2008]

A zero of an analytic function f(z) is a value of z such that f(z) = 0.


If f(z) is analytic in the neighbourhood of z = a, then by Taylor’s theorem
f(z) = a0 + a1(z – a) + a2(z – a)2 + a3(z – a)3 + ...... + an(z – a)n + ......∞
If a0 = a1 = a2 = ...... = an–1 = 0 but an ≠ 0, then f(z) is said to have a zero of order n at z = a.
The zero is said to be simple if n = 1.
f n (a)
∵ an =
n!
∴ for a zero of order m at z = a,

m
f(a) = f ′(a) = f ″(a) = ...... = f n–1(a) = 0 but f n(a) ≠ 0.
Thus in the neighbourhood of the zero at z = a of order n,

o
f(z) = an(z – a)n + an+1(z – a)n+1 + ......= (z – a)n [an + an+1 (z – a) + ......] = (z – a)n φ(z)

.c
where φ(z) = an + an+1(z – a) + ...... is analytic and non-zero at and in the neighbourhood of
z = a.

1.32 SINGULARITY
n ts
[M.T.U. 2013, U.P.T.U. (C.O.) 2008, 2009]

A singularity of a function f(z) is a point at which the function ceases to be analytic.


ra

1.33 ISOLATED AND NON-ISOLATED SINGULARITY [M.T.U. 2012]


pi

If z = a is a singularity of f(z) and if there is no other singularity within a small circle surrounding
the point z = a, then z = a is said to be an isolated singularity of the function f(z), otherwise it
as

is called non-isolated.
z+1
Example. Consider the function f(z) = .
z( z − 2)
g

It is analytic everywhere except at z = 0 and z = 2. Thus z = 0 and z = 2 are the only


.c

singularities of this function. There are no other singularities of f(z) in the neighbourhood of
z = 0, z = 2. Hence z = 0 and z = 2 are the isolated singularities of this function.
w

Again, consider the function


F πI
w

1
= cot G J
f(z) =
F πI
tan G J
H zK
H zK
w

F πI
It is not analytic at the points where tan G J = 0 = tan nπ i.e., at the points where = nπ
π
H zK z
1
⇒ z= (n = 1, 2, 3, ......)
n
1 1
Thus z = 1, , ,......, z = 0 are the singularities of the function all of which are isolated
2 3
except z = 0 because in the neighbourhood of z = 0, there are infinite number of other singularities
1
z= where n is large. Therefore z = 0 is the non-isolated singularity of the given function.
n

7-D:\Engineering Mathematics (Bali)\MTU Sem III-IV\MTU 1-7

www.cgaspirants.com
www.cgaspirants.com

86 A TEXTBOOK OF ENGINEERING MATHEMATICS

1.34 TYPES OF SINGULARITY

Let f(z) be analytic within a domain D except at z = a which is an isolated singularity. Draw a
circle C with its centre z = a and radius as small as we wish and another large concentric circle
C of any radius R lying wholly within the domain D. Now in the annulus between these two
circles, f(z) is analytic. If z is any point of the annulus, then by Laurent’s theorem,
∞ ∞
f(z) = ∑
0
an ( z − a) n + ∑ b (z − a)
1
n
−n
where 0 < | z – a | < R.


The second term ∑ b (z − a)
1
n
−n
on the RHS is called the Principal Part of f(z) at the

m
isolated singularity z = a. Now there arise three possibilities :
(i) All bn’s are zero ⇒ no term in P.P. (Removable singularity) (M.T.U. 2012)

o
(ii) Infinite number of terms in P.P. (Essential singularity) (M.T.U. 2012)
(iii) Finite number of terms in P.P. (Pole) [U.P.T.U. (C.O.) 2008]

.c

(i) Removable Singularity. Here f(z) = ∑a n (z − a) n which is analytic for | z – a | < R except
n
at z = a. Let ϕ(z) be the sum function of the power series
0
ts ∞

∑a n (z − a) n . Now ϕ(z) differs from


ra
0
f(z) only at z = a, where there is singularity. To avoid this singularity, we can suitably define
f(z) at z = a, so that we have
RSf ( z )
pi

for 0 <|z − a } < R


φ(z) =
Ta 0 for z = a
as

This type of singularity which can be made to disappear by defining the function suitably
is called removable singularity.
sin ( z − a)
g

Example. The function has removable singularity at z = a because


z−a
.c

sin ( z − a)
=
1
( z − a) −
RS
( z − a) 3 ( z − a) 5
+ − ......
UV
T W
w

z−a z−a 3! 5!
( z − a) 2 ( z − a) 4
w

=1– + − ......
3! 5!
has no terms containing negative powers of z – a. However this singularity can be removed
w

and the function made analytic by defining


sin ( z − a)
= 1 at z = a.
z−a

(ii) Essential Singularity. Here the series ∑ ( z − a)
1
−n
does not terminate.

Example. f(z) = sin


FG 1 IJ has essential singularity at z = a, because
H z − aK
sin
FG 1 IJ = 1 − 1 1 1 1
− ......
+
H z − a K z − a 3 ! (z − a)
5 ! ( z − a) 53

has infinite number of terms in the negative powers of z – a.

www.cgaspirants.com
www.cgaspirants.com

FUNCTION OF COMPLEX VARIABLE 87


(iii) Pole. Here the series ∑ (z − a)
1
n
consists of finite number of terms. Then z = a is said to

be a pole of order m of the function f(z). When m = 1, the pole is said to be simple.
sin ( z − a)
Example. f(z) = has a pole at z = a because
( z − a) 4

sin ( z − a)
=
1
( z − a) −
LM
( z − a) 3 ( z − a) 5 ( z − a) 7
+ − + ......
OP
( z − a) 4
( z − a) 4
N 3! 5! 7! Q
1 1 1 1 1
= 3
+ − ( z − a) − ( z − a) 3 + ......

m
( z − a) 3 ! ( z − a) 5 ! 7 !
has finite number of terms (here first two terms only) in negative powers of z – a.

o
Thus if z = a is a pole of order m of the function f(z), then

.c

b1 b2 bm
f(z) = ∑a
0
n ( z − a) n + +
z − a ( z − a) 2
+ ...... +
( z − a) m

=
1
( z − a) m
n LMF∞

MNGH ∑ a
0
n
I
JK ts OP
( z − a) n + m + {bm + bm − 1 ( z − a) + ......+ b1 ( z − a) m }
PQ
ra

1
= ϕ(z)
( z − a) m
pi

Clearly, ϕ(z) → bm as z → a. Hence ϕ(z) is analytic in the neighbourhood of the pole z = a.


as

1.35 THEOREMS

(1) The limit point of the zeros of a function f(z) is an isolated essential singularity.
g

Proof. Let z1, z2, z3, ...... be an infinite set of zeros of f(z). Let z0 be their limit point.
.c

(i) If z0 is a point of the set, then z0 will be a zero of f(z) and will have in its neighbour-
w

hood, a cluster of zeros. But zeros are isolated, so, z0 cannot be a zero of f(z) unless f(z) is
identically zero in D.
w

(ii) If f(z) is not identically zero in D, then z0 is not a zero of f(z). But z0 is surrounded by
many zeros. So z0 is a singularity. Also z0 is not a pole since f(z) does not tend to infinity in the
w

neighbourhood of z0. Therefore z0 is an essential singularity. But the singularity is isolated


since in the neighbourhood of z0, f(z) is analytic. Hence z0 is an isolated essential singularity.
(2) The limit point of the poles of a function f(z) is a non-isolated essential singularity.
Proof. Let p1, p2, p3,...... be an infinite set of poles of f(z). Let p0 be their limit point.
(i) If p0 is a point of the set, then p0 will be a pole of f(z) and will have in its neighbour-
hood a cluster of poles. But poles are isolated, so, p0 cannot be a pole of f(z).
(ii) p0 cannot be a zero of f(z) since the function is not analytic (has poles) in the neigh-
bourhood of p0. So, p0 is an essential singularity. This singularity is not isolated, since these
are poles around p0. Hence p0 is a non-isolated essential singularity.

7-D:\Engineering Mathematics (Bali)\MTU Sem III-IV\MTU 1-7

www.cgaspirants.com
www.cgaspirants.com

88 A TEXTBOOK OF ENGINEERING MATHEMATICS

1.36 DETECTION OF SINGULARITY

(1) Removable Singularity: lim f(z) exists and is finite.


z→ a

z2 – a2
Example. f(z) =
z−a
lim f(z) = 2a
z→ a
So, f(z) has a removable singularity at z = a.
(2) Pole: lim f(z) = ∞.
z→ a

z2 + a2

m
Example. f (z) =
z–a
lim f(z) = ∞.

o
z→ a
So, f(z) has a pole at z = a.

.c
Moreover, the pole is said to be of order n, if there are n terms in the principal part.
e z− a LM1 + (z − a) + (z − a) 2 OP
Example.
( z − a) 2
=

=
1
( z − a)
n
1
2

+
N
1
+
1
ts
2!

+ ......
+ ......
Q
( z − a) 2 ( z − a) 2 !
ra

Since there are only two terms in the negative powers of z – a i.e., there are only 2
(a finite number) terms in the principal part of the function. Hence the function has a pole of
pi

order 2.
(3) Essential singularity: lim f(z) does not exist.
as

z→ a
1
Example. lim e z− a does not exist, so f(z) has an essential singularity at z = a.
g

z→a
.c

EXAMPLES
w

Example 1. Find out the zero and discuss the nature of the singularity of
z−2 1
w

f(z) = 2
sin .
z z−1
Sol. Zeros of f(z) are given by f(z) = 0
w

1
⇒ z – 2 = 0, sin =0
z−1
1
⇒ z = 2, = nπ (n = 0, ± 1, ± 2, ......)
z−1
1
⇒ z = 2, 1 + (n = 0, ± 1, ± 2, ......)

Clearly, z = 1 is an isolated essential singularity.
Poles of f(z) are given by
z2 = 0
⇒ z=0
Hence z = 0 is a pole of order 2.

www.cgaspirants.com
www.cgaspirants.com

FUNCTION OF COMPLEX VARIABLE 89

Example 2. Show that the function ez has an isolated essential singularity at z = ∞.


1
1 ρ 1 1 1
Sol. Put z= e = 1+ + + ...... ∞
ρ ρ 2 ! ρ2
1
We have an infinite number of terms in the negative powers of ρ. So the function e ρ has
an isolated essential singularity at ρ = 0. This implies that ez has an isolated essential singu-
larity at z = ∞.
1
Example 3. Discuss singularity of at z = 2πi.
1 − ez

m
1
Sol. f(z) =
1 − ez

o
For poles 1 – ez = 0

.c
⇒ ez = 1 = e2nπi

Clearly, z = 2πi is a simple pole.
n ts
z = 2nπi (n = 0, ± 1, ± 2, ......)

cot πz
ra

Example 4. Discuss singularity of at z = a and z = ∞ . [U.P.T.U. (C.O.) 2008]


(z – a) 2
pi

cot πz cos πz
Sol. f(z) = 2
=
( z – a) sin πz ( z – a) 2
as

For poles sin πz(z – a)2 = 0


⇒ z = a, πz = nπ (n ∈ I)
g

⇒ z = a, n
.c

Clearly z = ∞ is the limit point of these poles. Hence z = ∞ is a non-isolated essential


singularity. Also z = a, being repeated twice, gives a double pole.
w

z − sin z
Example 5. Discuss the nature of singularity of f(z) = at z = 0.
w

z3

LM z − F z − z + z − z + ......I OP
w

3 5 7
1 1
Sol. f(z) =
z 3 (z – sin z) =
z MN
GH 3 ! 5 ! 7 ! JK PQ
3

1 Fz z z3 5 I 1 − z + z − ......
7 2 4
= G
3
− +
z H 3! 5! 7!
− ......J =
K 3! 5! 7!
Since, there is no term in the principal part of given function hence z = 0 is a removable
singularity.

7-D:\Engineering Mathematics (Bali)\MTU Sem III-IV\MTU 1-7

www.cgaspirants.com
www.cgaspirants.com

90 A TEXTBOOK OF ENGINEERING MATHEMATICS

ASSIGNMENT

1 π
1. Discuss singularity of at z = . [U.P.T.U. (C.O.) 2008]
sin z − cos z 4
2. Discuss the nature of singularity of the function f(z) = z cosec z at z = ∞.
3. What is the nature of the singularity at z = ∞ of the function f(z) = cos z – sin z?

1
4. Discuss the singularity of the function f(z) = at z = 0.
1
cos
z

m
5. Discuss the nature of singularity of f(z) = sin at z = 0. [U.P.T.U. (C.O.) 2009]
z

e1/ z

o
6. Find the singularity of the function g(z) = . [U.P.T.U. (C.O.) 2009]
z2

.c
7. Prove that the singularity of cot z at z = ∞ is a non-isolated essential singularity.

ts
8. Find the nature of singularities of the following functions:

1 – ez 1
n
(i) at z = ∞ [U.P.T.U. (C.O.) 2008] (ii) cosec at z = 0.
1+ e z z
ra

1
9. Discuss singularity of f(z) = sin at z = 1.
1− z
pi

Answers
as

1. Simple pole 2. Non-isolated essential 3. Isolated essential


4. Non-isolated essential 5. Isolated essential
g

6. Isolated essential singularity (z = 0)


8. (i) Non-isolated essential, (ii) Non-isolated essential 9. Isolated essential.
.c

1.37 DEFINITION OF THE RESIDUE AT A POLE


w

Let z = a be a pole of order m of a one valued function f(z) and γ any circle of radius r with
w

centre at z = a which does not contain any other singularities except at z = a, then f(z) is
analytic within the annulus r < | z – a | < R hence it can be expanded within the annulus in
w

a Laurent’s series in the form


∞ ∞
f(z) = ∑
n=0
an ( z − a) n + ∑ b ( z − a)
n=1
n
−n
...(1)

where an =
1
2πi z
C
f ( z) dz
( z − a) n + 1
...(2)

and bn =
1
z f ( z)
γ ( z − a) − n + 1
2 πi
dz ...(3)

| z – a | = r being the circle γ.

www.cgaspirants.com
www.cgaspirants.com

FUNCTION OF COMPLEX VARIABLE 91

Particularly, b1 =
1
2πi z
γ
f ( z) dz

The coefficient b1 is called residue of f(z) at the pole z = a. It is denoted by symbol


Res. (z = a) = b1.

1.38 RESIDUE AT INFINITY

Residue of f(z) at z = ∞ is defined as –


anti-clockwise direction.
1
2πi zC
f ( z) dz where the integration is taken round C in

m
1.39 CAUCHY’S RESIDUE THEOREM OR THE THEOREM OF RESIDUES
[M.T.U. 2013, G.B.T.U. (C.O.) 2011]

o
.c
Let f(z) be one valued and analytic within and on a closed contour C except at a finite number
of poles z1, z2, z3, ......, zn and let R1, R2, R3, ......., Rn be respectively the residues of f(z) at these
poles, then

z C
n
f(z) dz = 2πi (R1 + R2 + R3 + ...... + Rn)
ts
ra

= 2πi (Sum of the residues at the poles within C).

Proof. Let γ1, γ2, γ3, ......, γn be the circles with centres at z1, z2, z3, ......, zn respectively and radii
pi

so small that they lie entirely within the closed curve C and do not overlap. Then f(z) is analytic
within the region enclosed by the curve C and these circles. Hence by Cauchy’s theorem for
as

multi-connected regions, we have

z f ( z) dz = z f ( z) dz + z f ( z) dz + z f ( z) dz + ...... + z f ( z) dz
g

C γ1 γ2 γ3 γn
.c

But by definition of residue,


1
z
w

R1 = f ( z) dz
2πi γ1

z
w

⇒ f ( z) dz = 2πi R1
γ1

z
w

Similarly, f ( z) dz = 2πi R
γ2 2

zγ3
f ( z) dz = 2πi R3

zγn
f ( z) dz = 2πi Rn

Hence, zC
f ( z) dz = 2πi R1 + 2πiR2 + 2πi R3 + ...... + 2πi Rn
= 2πi(R1 + R2 + R3 + ...... + Rn).

7-D:\Engineering Mathematics (Bali)\MTU Sem III-IV\MTU 1-7

www.cgaspirants.com
www.cgaspirants.com

92 A TEXTBOOK OF ENGINEERING MATHEMATICS

1.40 METHODS OF FINDING OUT RESIDUES

(1) If f(z) has a simple pole (i.e., pole of order 1) at z = a, then

Res {f(z)} = Lt (z – a) f(z).


z→a

Since z = a is a pole of order 1, the Laurent’s series becomes


f(z) = a0 + a1(z – a) + a2(z – a)2 + ...... + b1(z – a)–1.
Multiplying both sides by (z – a), we get
(z – a) f(z) = a0(z – a) + a1(z – a)2 + a2(z – a)3 + ...... + b1

m
∴ Lt (z – a) f(z) = b1 = Res {f(z)}
z→ a

o
(2) If f(z) has a pole of order m at z = a, then

.c
1 d m−1
Res {f(z)} = Lt [(z – a)m f(z)]
(m − 1) ! z → a dz m − 1
Since z = a is a pole of order m, the Laurent’s series becomes
n ts
f(z) = a0 + a1(z – a) + a2(z – a)2 + ...... + b1(z – a)–1 + b2(z – a)–2 + ..... + bm(z – a)–m
Multiplying both sides by (z – a)m, we get
ra

(z – a)m f(z) = a0(z – a)m + a1(z – a)m+1 + a2(z – a)m+2 + ......


+ b1(z – a)m–1 + b2(z – a)m–2 + ...... + bm
pi

Differentiating both sides (m – 1) times w.r.t. z and taking the limit as z → a, we get
as

d m−1
Lt [(z – a)m f(z)] = b1(m – 1) !
z→a dz m − 1
g

1 d m− 1
Lt [(z – a)m f(z)] = b1 = Res {f(z)}.
.c

or
(m − 1) ! z → a dz m− 1
Or
w

LM
d m−1 OP
w

1
Res. {f(z)} = {( z − a) m f ( z)}
N
(m − 1) ! dz m − 1 Q z= a
w

(3) If f(z) is of the form given by


φ( z)
f(z) = ; ψ(a) = 0, φ(a) ≠ 0
ψ ( z)
φ(a)
where z = a is the simple pole of f(z), then Residue of f(z) at z = a is = .
ψ ′ (a)
(4) Residue of f(z) at z = a pole {simple or of order m}
1
= Coefficient of in f(a + t) expanded in powers of t, where t, is sufficiently small.
t

www.cgaspirants.com
www.cgaspirants.com

FUNCTION OF COMPLEX VARIABLE 93

(5) Residue of f(z) at z = ∞


= lim { − zf ( z)}
z→∞
Or
1
LM
= – coefficient of
z
in the expansion of f(z) for values of z in the OP
N neighbourhood of z = ∞ Q.
EXAMPLES
Example 1. Determine the poles of the following functions and residue at each pole:
z2 1 1 – e 2z

m
(i) 2 (ii) 4 (iii)
(z − 1)(z − 2) z +1 z4
z2

o
Sol. (i) f(z) = .
( z − 1)( z − 2) 2

.c
Poles are given by
(z – 1)(z – 2)2 = 0 ⇒ z = 1, 2.
n
z2 z2
ts
z = 1 is a simple pole while z = 2 is a double pole.
Residue of f(z) at simple pole (z = 1) is
(1) 2
R1 = lim ( z − 1) . = lim = = 1.
ra
2
z→1 ( z − 1)( z − 2) 2 z → 1 ( z − 2) (1 − 2) 2
Residue of f(z) at double pole (z = 2) is
LM R UVOP
pi

1 d z2
MN ST
R2 = ( z − 2) 2 .
(2 − 1) ! dz ( z − 1)( z − 2) 2 WPQ z=2
as

L d F z I OP = L (z − 1) . 2z − z
=M
2 2 OP LM z − 2z OP
2

MN dz GH z − 1JK QP MN (z − 1) =0
=
2
Q N (z − 1) Q
2
g

z=2 z=2 z=2


1
.c

(ii) f(z) = 4
z +1
w

Poles of f(z) are given by


z4 + 1 = 0
w

⇒ z = (– 1)1/4 = {e(2n+1)πi}1/4
∴ Poles are, z = e(2n+1)πi/4 where, n = 0, 1, 2, 3, ......
w

These are all of order 1 since the four factors occur linearly in z4 + 1.
Since the roots repeat themselves, we can write them more conveniently as
e(2n+1)πi/4 where, n = – 2, – 1, 0, 1
i.e., emπi/4 where, m = ± 1, ± 3. Let denote it by zm.
Residue at (z = zm) is
1 1
R = lim ( z − zm ) . 4 = lim | By L’ Hospital’s Rule
z → zm z +1 z → zm 4 z3
1 zm zm 1
= 3
= 4
=– = − emπi/4 h where, m = ± 1, ± 3.
4 zm 4 zm 4 4

7-D:\Engineering Mathematics (Bali)\MTU Sem III-IV\MTU 1-7

www.cgaspirants.com
www.cgaspirants.com

94 A TEXTBOOK OF ENGINEERING MATHEMATICS

1 − e2 z
(iii) Pole of is evidently z = 0. But this is not of the fourth order since
z4
1 − e2 z 1 LM R 4 z 2 8 z 3 16 z 4 UVOP
MN ST
= 1 − 1 + 2 z + + + + ......
z4 z4 2 6 24 WPQ
FG 2 + 2z + 4 z 2 2 3 IJ
=–
H 3
+
3
z + ......
K ...(1)
z3
Therefore the pole is of order 3.
Residue at this pole is

m
R = lim
RS (1 − e ) z UV
1 d2 2z 3

z→0
T z W
2 ! dz 2 4

o
1 L d R| 1 F I U|VOP

.c
2 2 3
= lim M S
2 ! M dz |T z HG 1 − 1 − 2z −
4z

8z
− ......JK |WPQ
z→0
N 2
2! 3!

1Ld F
= lim M
z→02 N dz H
G
2
n
2
4
3
IO
− 2 − 2 z − z − ......J P
KQ
2
ts
ra

1L 8 2 O 4
= lim M− − . 6 z − ......P = −
z→02N 3 3 Q 3
pi

Example 2. Find the residue at z = 0 of the following functions:


as

1 + ez 1
(i) (ii) z cos .
sin z + z cos z z
g

Sol. (i) z = 0 is a pole of order 1.


.c

z(1 + e z ) 1 + ez 1+ 1
Residue = lim = lim = = 1.
FG IJ
w

z→0 sin z + z cos z z → 0 sin z 1+ 1


z
+ cos z
H K
w

(ii) Expanding the function in powers of z, we have

LM OP
w

1 1 1 1 1
z cos = z 1− 2 + − ...... = z – + – ......
z N2z 4 ! z4 2 z 24 z 3 Q
This is the Laurent ’s expansion about z = 0.
1 1 1 1
The coefficient ofin it is – . So the residue of z cos at z = 0 is – .
z 2 z 2
Example 3. (a) Give an example of a function having residue at infinity yet analytic
there.
z3
(b) Find the residue of f(z) = at z = ∞.
z2 − 1

www.cgaspirants.com
www.cgaspirants.com

FUNCTION OF COMPLEX VARIABLE 95

z2
Sol. (a) f(z) =
( z − α)( z − β)( z − γ )
Residue of f(z) at z = ∞ is

= lim − z .
RS z2 UV
z→∞
T ( z − α)( z − β)( z − γ ) W
−1
= lim
z→∞ FG 1 − α IJ FG β IJ FG 1 − γ IJ = –1
H zK H 1−
z K H zK

m
1
1 FG IJ λ2 λ

o
Now, f
λ
=
H K
(1 − αλ) (1 − βα) (1 − γλ)
. .
=
(1 − αλ)(1 − βλ)(1 − γλ)

.c
λ λ λ

FG 1IJ = 0 (≠ ∞)
At λ = 0, f
H λK
n ts
FG 1IJ is analytic at λ = 0
ra
f

H λK
⇒ f(z) is analytic at z = ∞.
pi

F − z . z I which does not exist


3
(b) Required residue = Lt GH z − 1JK
as

z→∞ 2

F 1I F 1 + 1 + ......IJ = z + 1 + 1
g

3 −1
z
= z G1 − J = z G1 +
Hence, f(z) =
F 1I H K H z z K z z + ......
.c

2 2 4 3
z G1 − J 2 z
H zK 2
w

F 1I
Required residue = – G coefficient of J = – 1.
H zK
w

z ez
w

Example 4. Evaluate dz , where C is the circle | z – 1 |= 3.


C (z + 1) 2
ez
Sol. Here f(z) = has only one singular point z = – 1 which is a pole of order 2 and
( z + 1) 2
it lies inside the circle | z – 1 | = 3.
d d z
Residue of f(z) at z = – 1 is Lt [(z + 1)2 f(z)] = Lt ( e ) = Lt ez = e–1
z→−1 dz z → − 1 dz z→−1

∴ By Residue theorem, we have


zC
ez
( z + 1) 2
dz = 2πi(e −1 ) =
2πi
e
.

www.cgaspirants.com
www.cgaspirants.com

96 A TEXTBOOK OF ENGINEERING MATHEMATICS

z2
Example 5. Determine the poles of the function f(z) = and the residue at
(z − 1) 2 (z + 2)

each pole. Hence evaluate


zC
z2
(z − 1) 2 (z + 2)
dz where C ≡ |z| = 3. (U.P.T.U. 2015)

Sol. The function f(z) has a pole of order 2 at z = 1 and a simple pole at z = – 2.
Residue of f(z) at z = 1 is

d d z2 F I
R1 = Lt
z→1 dz
[(z – 1)2 f(z)] = Lt
z → 1 dz z + 2
GH JK

m
( z + 2) . 2 z − z 2 . 1 z2 + 4 z 5
or R1 = Lt 2
= Lt =
z→1 ( z + 2) z → 1 ( z + 2) 2 9

o
Residue of f(z) at z = – 2 is

.c
z2 4
R2 = Lt [(z + 2) f(z)] = Lt = .

ts
2
z → −2 z → −2 ( z − 1) 9
Since both the poles lie inside the given curve C ≡ | z | = 3,
n
z z2 LM 5 + 4 OP = 2πi.
ra
∴ dz = 2πi ( R1 + R 2 ) = 2πi
C 2
( z − 1) ( z + 2) N9 9Q
| By Cauchy’s Residue theorem
pi

Example 6. Determine the poles of the following function and residues at each pole:
z−1
z
as

f(z) = and hence evaluate f ( z) dz , where C is the circle | z – i | = 2.


(z + 1) 2 (z − 2) C

(U.K.T.U. 2011)
g

Sol. Poles of f(z) are given by


.c

(z + 1)2 (z – 2) = 0 ⇒ z = – 1 (double pole), 2(simple pole)


Residue of f(z) at z = – 1 is
w

1 LM R
d z−1 UVOP Y

MN ST
2
( z + 1) 2 .
w

R1 = |=
(2 − 1) ! dz ( z + 1) 2 ( z − 2) WPQ z =−1
(0, 3)
C≡
| z–
i
w

L d FG z − 1IJ OP
=M =
LM − 1 OP =
−1
N dz H z − 2 K Q z =−1 N (z − 2) Q
2
z =−1
9
2
(0, 1) z = i

Residue of f(z) at z = 2 is 1
– 1 O (0, 0)
z−1 (– 3, 0) 2 X
R2 = Lt ( z − 2) 2
z→2 ( z + 1) ( z − 2) (0, – 1)
z−1 1
= Lt 2
=
z→2 ( z + 1) 9
The given curve C ≡ | z – i | = 2 is a circle whose centre is at z = i [i.e., at (0, 1)] and
radius is 2. Clearly, only the pole z = – 1 lies inside the curve C.

www.cgaspirants.com
www.cgaspirants.com

FUNCTION OF COMPLEX VARIABLE 97

Hence, by Cauchy’s residue theorem

z C
f ( z) dz = 2πi (R1) = 2πi
FG − 1IJ = – 2πi .
H9K 9
Example 7. Evaluate zC
z 2 − 2z
(z + 1) 2 (z 2 + 4)
dz, where C is the circle | z | = 10.

(U.P.T.U. 2009)
Sol. Singularities are given by
(z + 1)2 (z2 + 4) = 0 ⇒ z = – 1 (double pole), ± 2i (simple poles)
All the poles lie inside the given circle c ≡ | z | = 10.

m
∴ Residue (at z = – 1) is

1 dLM R z2 − 2z UVOP

o
MN ST
R1 = ( z + 1) 2
2 − 1 ! dz ( z + 1) 2 ( z 2 + 4) WPQ

.c
z=−1

LM d F z − 2 z I OP
2
L 2z + 8z − 8 OP
=M
2
14

Residue (at z = 2i) is


=
MN dz GH z + 4 JK PQ
n 2
z=−1 ts
N (z + 4) Q 2 2
z=−1
=−
25
ra

z2 − 2 z
R2 = lim ( z − 2i) 2
z → 2i ( z + 1) ( z − 2i) ( z + 2i)
pi

− 4 − 4i 1+ i 7+i
= 2
= =
(2i + 1) (4i) 3i + 4 25
as

Similarly, Residue (at z = – 2i) is


7−i
g

R3 =
25
.c

By Cauchy’s Residue theorem,

z
w

z2 − 2 z
dz = 2πi (R1 + R2 + R3) = 2πi − + +
LM
14 7 + i 7 − i
=0
OP
C ( z + 1) 2 ( z 2 + 4) 25 25 N
25 Q
w

Example 8. Evaluate z 12z − 7


dz , where C is the circle
w

C (z − 1) 2 (2z + 3)
(i) | z | = 2 (G.B.T.U. 2011) (ii) | z + i | = 3.
12 z − 7
Sol. f(z) = .
( z − 1) 2 (2 z + 3)
Poles are given by
3
z = 1 (double pole) and z = – (simple pole)
2
Residue at (z = 1) is
1 d LM R 12 z − 7 UVOP
MN ST
R1 = ( z − 1) 2 .
(2 − 1) ! dz ( z − 1) 2 (2 z + 3) WPQ z= 1

www.cgaspirants.com
www.cgaspirants.com

98 A TEXTBOOK OF ENGINEERING MATHEMATICS

LM d FG 12z − 7 IJ OP LM (2z + 3) . 12 − (12z − 7) . 2 OP


=
N dz H 2z + 3 K Q z= 1
=
N (2 z + 3)2
Q z=1
60 − 10
= = 2.
25
FG 3 IJ
Residue at simple pole z = −
H 2
is
K
FG z + 3 IJ . 12 z − 7
R2 = lim
z → − 3/ 2 H 2 K ( z − 1) (2 z + 3)
2

1 (12 z − 7)
= lim . = – 2.
z → − 3/ 2 2 ( z − 1) 2

m
3
(i) The contour | z | = 2 encloses both the poles 1 and – .
2

o
∴ The given integral = 2πi (R1 + R2) = 2πi (2 – 2) = 0.

.c
(ii) The contour | z + i | = 3 is a circle of radius 3 and centre at z = – i. The distances
3 13
of the centre from z = 1 and – are respectively 2 and

second is > 3 .
2
n 4

∴ The second contour includes only the first singularity z = 1.


ts
. The first of these is < 3 and the
ra

Hence, the given integral = 2πi (R1) = 2πi (2) = 4πi.

Example 9. Evaluate z z−3


dz , where C is the circle
pi

C z 2 + 2z + 5

(i) | z | = 1 (ii) | z + 1 – i | = 2 (iii) | z + 1 + i | = 2.


as

(G.B.T.U. 2013)
z−3
Sol. The poles of f(z) = are given by
g

2
z + 2z + 5
.c

z2 + 2z + 5 = 0 ⇒ z = – 1 ± 2i
(i) Both the poles lie outside the circle | z | = 1.
w

∴ By Cauchy’s integral theorem, we have z z−3


dz = 0
w

2
C z + 2z + 5
(ii) Only the pole z = – 1 + 2i lies inside the circle | z + 1 – i | = 2
w

Residue of f(z) at z = – 1 + 2i is
( z − α)( z − 3) 0
Lt (z + 1 – 2i) f(z) = Lt , where α = – 1 – 2i Form
z → − 1 + 2i 2
z → α z + 2z + 5 0
( z − α) + ( z − 3)
= Lt | By L’ Hospital’s Rule
z→α 2z + 2
α−3 − 1 + 2i − 3 i − 2
= = =
2α + 2 − 2 + 4i + 2 2i

∴ By Cauchy’s residue theorem, 2


z−3
C z + 2z + 5 z dz = 2πi
i−2
2i
= π(i – 2).
FG
H
IJ
K

www.cgaspirants.com
www.cgaspirants.com

FUNCTION OF COMPLEX VARIABLE 99

(iii) Only the pole z = – 1 – 2i lies inside the circle | z + 1 + i | = 2.


Residue of f(z) at z = – 1 – 2i is
( z − β)( z − 3) 0
Lt (z + 1 + 2i) f(z) = Lt , where β = – 1 – 2i Form
z → − 1 − 2i
2
z → β z + 2z + 5 0
( z − β) + ( z − 3)
= Lt | By L’ Hospital’s Rule
z→β 2z + 2
β−3 − 4 − 2i i + 2
= = =
2β + 2 − 4i 2i

∴ By Cauchy’s residue theorem, z 2


z−3
dz = 2πi
i+2 FG
= π(i + 2)
H
IJ
K

m
C z + 2z + 5 2i

z3
Example 10. Find the residue of f(z) = at its pole and hence evaluate

o
4
(z − 1) (z − 2)(z − 3)

z
.c
f ( z) dz , where C is the circle | z | = 5/2.
C
Sol. Poles of f(z) are given by (z – 1)4 (z – 2)(z – 3) = 0 ⇒ z = 1, 2, 3
z = 1 is a pole of order 4 while z = 2 and z = 3 are simple poles.
Residue of f(z) at z = 2 is
n ts
ra

z3 z3 8
R1 = Lt ( z − 2) = Lt = =–8
z→2 ( z − 1) 4 ( z − 2)( z − 3) z → 2 ( z − 1) 4 ( z − 3) (− 1)
pi

Residue of f(z) at z = 3 is
z3 z3 27
R2 = Lt ( z − 3) . = Lt
as

4 4
=
z→3 ( z − 1) ( z − 2)( z − 3) z → 3 ( z − 1) ( z − 2) 16
Residue of f(z) at z = 1 is
g

1 LM R
d3 z3 UVOP
S
.c

R3 = ( z − 1) 4 .
MN T
(4 − 1) ! dz 3 4
( z − 1) ( z − 2)( z − 3)WPQ z=1
w

1Ld R
= M
3
S z 3 UVOP = 1 LM d RSz + 5 + 19 z − 30 UVOP
3

6 MN dz T ( z − 2)( z − 3) WPQ 6 MN dz T z − 5 z + 6 WPQ


w

3 3 2
z=1 z=1

1Ld R 8 UO 1Ld R UVOP


w

3 2
27
= M
6 N dz T
S
3
z+5+ −
z − 3 z − 2 WQ
V P = M
6 MN dz T
S1 − (z −273) + ( z −82)
2 2 2
WPQ
z=1 z=1

1 L d R 54 16 UO
= M S VP = 61 LM (z− −162 48 O
N 3) (z − 2) PQ
− +
6 MN dz T ( z − 3) 3
( z − 2) WPQ 3 4 4
z=1 z=1

1 L − 162 O 27 = 101
= M + 48P = 8 −
6 N 16 Q 16 16
The given curve C ≡ | z | = 5/2 is a circle with centre at (0, 0) and radius 5/2.
Clearly, only the poles z = 1 and z = 2 lie inside this circle.

www.cgaspirants.com
www.cgaspirants.com

100 A TEXTBOOK OF ENGINEERING MATHEMATICS

Hence, By Cauchy’s Residue theorem,

zC
f ( z) dz = 2πi (R + R ) = 2πi 101 − 8 = 2πi − 27 = –
3 1
16 16
FG
27πi
H
8
.
IJ
K
FG
H
IJ
K
Example 11. (i) Find the value of z
C
ze 1/z dz around the unit circle.

(ii) Using Residue theorem, evaluate


1
2 πi z
C
e zt
z 2 (z 2 + 2z + 2)
dz , where C is the circle

|z| = 3. (U.P.T.U. 2009)


1/z 1/z
Sol. (i) The only singularity of ze is at the origin. Expanding e , we have

m
LM
ze1/z = z 1 +
1 1 1
+ 2 + 3 + ...... = z + 1 +
1 1 OP
+ 2 + ......
N Q

o
z 2z 6z 2z 6z

.c
1 1
Residue at origin = coefficient of = .
z 2

Hence, the required integral = 2πi


n FG 1IJ = πi.
H 2K
ts
ra

(ii) Singularities are given by


z2 (z2 + 2z + 2) = 0 ⇒ z = 0, – 1 ± i
pi

z = 0 is a pole of order 2. z = – 1 ± i are simple poles. All these poles lie inside the circle
|z| = 3.
Residue (at z = 0) is
as

1 d LM |RS e zt |UVOP LM d |R e zt
|UVOP
R1 = z2 .
|T |WPQ = S
MN dz |T 2πi (z + 2z + 2) |WPQ
g

(2 − 1) ! dz MN
2π i z 2 ( z 2 + 2 z + 2)
z=0
2
z=0
.c

LM
1 ( z 2 + 2 z + 2) t e zt − e zt (2 z + 2) O 1 F t − 1I
G J
w

=
2πi MN ( z 2 + 2 z + 2) 2
PPQ =
2πi H 2 K
z=0
w

Let – 1 + i = α and – 1 – i = β then


w

Residue at (z = α = – 1 + i) is

R2 = Lt ( z − α) .
1 e zt
=
1 e αt
=
1 1 (− 1 + i) t
e
LM OP
z→α 2
2πi z ( z − α) ( z − β) 2
2πi α (α − β) 2πi 4 N Q
Residue at (z = β = – 1 – i) is

R3 =
LM
1 1 (− 1 − i) t
e
OP
2πi 4 N Q

www.cgaspirants.com
www.cgaspirants.com

FUNCTION OF COMPLEX VARIABLE 101

By Residue theorem,

1
2πi z
C 2 2
e zt
z ( z + 2 z + 2)
dz = 2πi
LM 1 FG t − 1 IJ +
N 2πi H 2 K 2πi
R|S
1 e ( − 1 + i) t + ( e ( − 1 − i) t
T| 4
U|VOP
W|PQ
t − 1 e−t
= + cos t .
2 2
1
Example 12. Obtain Laurent’s expansion for the function f(z) = at the isolated
z 2 sinh z

singularity and hence evaluate zC z 2 sinh z


1
dz , where C is the circle | z – 1 | = 2.

m
1 2
Sol. Here, f(z) = =
z 2 sinh z z 2 (e z − e − z )

o
.c
2
=
LMF 1 + z + z + z + ......I − F 1 − z + z − z + ......I OP
2 3 2 3

MNGH JK GH JK PQ
z2

=
2! 3!
n
2
2! 3!
ts
F I
ra
3
2z 2 z5
z2 2 z +GH 3!
+
5!
+ ...... JK Y
| =2
–1
pi

|z
1 C≡
=
F z + z + ......I
2 4
GH 3 ! 5 ! JK
as

z3 1 +
O X
z=1
LM1 + F z + z I + ......OP
g

−1
2 4

MN GH 6 120 JK PQ
= z–3
.c

F 1 − z − z + z + ......I
w

2 4 4
= z–3 GH 6 120 36 JK
w

1 1 7
w

= − + z + ......
z3 6 z 360
Only pole z = 0 of order two lies inside the circle C ≡ | z – 1 | = 2.
1 1
Residue of f(z) at (z = 0) is = coeff. of in the Laurent’s expansion of f(z) = – .
z 6
By Cauchy’s Residue theorem,

zC 2
z sinh z
dz
= 2 πi −
FG 1 IJ = – πi .
H 6K 3

www.cgaspirants.com
www.cgaspirants.com

102 A TEXTBOOK OF ENGINEERING MATHEMATICS

ASSIGNMENT
Determine the poles of the following functions and the residue at each pole:
2z + 1 z+1 ez
1. 2 2. 3. .
z −z−2 z2 ( z − 2) z2 + π 2
Evaluate the following integrals using Cauchy’s residue theorem:

4.
z LMMN
C
cos πz2 + sin πz2
( z + 1) ( z + 2)
OP
dz ; C ≡ | z | = 3
PQ
5.
z LMMN 3 z2 + z + 1 OP dz , where C is the circle | z | = 2.

m
C ( z2 − 1) ( z + 3) PQ
z z2 + 2 z − 2

o
6. dz , where C is a closed curve containing the point z = 4 in its interior.
z−4

z
C

.c
1 − 2z
7. dz , where C is the circle | z | = 1.5.
z( z − 1)( z − 2)

z
C

8.
C
z
( z − 1)( z − 2) 2
dz , where C is the circle | z – 2 | = 21 .
n ts
9.
z sin πz2 + cos πz2
dz , where C is the circle | z | = 3.
ra

C ( z − 1)2 ( z − 2)

10.
z 5z − 2
dz ; C ≡ | z | = 2 11.
z ez − 1
dz ; C ≡ | z | = 1/2
pi

C z( z − 1) C z( z − i) 2 ( z − 1)

12.
z FGH z cos z
dz, where C ≡ | z – 1 | = 1 13.
z dz
, where C ≡ | z – i | = 2
as

C π IJ 3
C ( z2 + 4)2
z−
2 K
z
g

3 z2 + 2
14. dz , where C ≡ | z – 2 | = 2
.c

C ( z − 1)( z2 + 9)

z 2z + 1
z dz
w

15. dz , where C ≡ | z | = 1 16. dz , where C ≡| z | = 1.5


C (2 z − 1)2 C ( z2 + 1)( z2 − 4)

z
w

4 z2 − 4 z + 1
17. (i) dz , where C ≡ | z | = 1.
C ( z − 2)(4 + z2 )
w

(ii) z C
24 z − 7
( z − 1) 2 ( 2 z + 3 )
dz, where C is the circle of radius 2 with centre at the origin.

(M.T.U. 2012)

18.
zC
z2 + 4
2
z( z + 2 z + 2)
dz, where C is

(i) | z | = 1 (ii) | z + 1 – i | = 1
(iii) | z + 1 + i | = 1 (iv) | z – 1 | = 5

19.
zC
e −z

z2
dz ; C ≡ | z | = 1 20.
z C
z2 e1/ z dz ; C ≡ | z | = 1

www.cgaspirants.com
www.cgaspirants.com

FUNCTION OF COMPLEX VARIABLE 103

21.
zC
2
1
z sin z
dz where C is the triangle with vertices (0, 1), (2 – 2) and (7, 1). (G.B.T.U. 2012)

z
22. Determine the poles and residues at each pole of the function f(z) = and hence evaluate
z2 − 3 z + 2

zC
f ( z ) dz where C is the circle | z – 2| =
1
2
. (G.B.T.U. 2011)

Answers
1 5 3 3 i
1. z = – 1, 2 ; , 2. z = 0, 2 ; – , 3. z = ± πi ; ±
3 3 4 4 2π
πi
4. – 4πi 5. − 6. 44πi

m
4
7. 3πi 8. – 2πi 9. 4πi(π + 1)

o
10. 10πi 11. 0 12. – 2πi
13. π/16 14. πi

.c
15. πi 16. 0 17. (i) 0 (ii) 0
18. (i) 4πi (ii) – π (3 + i) (iii) π (3 – i) (iv) 2πi

19.

22.
– 2πi

z = 1, 2 ; – 1, 2 ; 4πi.
n 20.
πi
3
. ts 21.
2i (− 1) n
n2 π
ra

1.41 CONTOUR INTEGRATION


pi

We take a closed curve C, find the poles of f(z) within C and calculate residue at these poles.
Then by Cauchy’s residue theorem

z
as

f ( z) dz = 2πi [sum of the residues of f(z) at the poles within C]


C
g

The curve is called a contour.


The process of integration along a contour is called contour integration.
.c

1.42 APPLICATION OF RESIDUE THEOREM TO EVALUATE REAL INTEGRALS


w

The residue theorem provides a simple and elegant method for evaluating many important
w

definite integrals of real variables. Some of these are illustrated below.

z
w


1.42.1. Integrals of the Type F(cos θ, sin θ) dθ , where F(cos θ, sin θ) is a Rational
0
Function of cos θ and sin θ.
Such integrals can be reduced to complex line integrals by the substitution z = eiθ, so
that
dz
dz = ieiθ dθ, i.e., dθ = .
iz
e iθ + e − iθ 1 FG IJ
1
Also, cos θ =
2
=
2 H K
z+
z
e iθ – e − iθ 1 F 1I
G z− J.
2i H zK
sin θ = =
2i

www.cgaspirants.com
www.cgaspirants.com

104 A TEXTBOOK OF ENGINEERING MATHEMATICS

As θ varies from 0 to 2π, z moves once round the unit circle in the anti-clockwise direction.

∴ z
0

F(cos θ, sin θ) dθ = zC
F
Fz+ z
GH 2
−1
,
z − z −1 dz
2i iz
I
JK
where C is the unit circle | z | = 1.
The integral on the right can be evaluated by using the residue theorem.

EXAMPLES

Example 1. Using contour integration, evaluate z 2π dθ


where a > |b|

m
0 a + b cos θ
Hence or otherwise evaluate (U.K.T.U. 2010)

o
(i) z 2π dθ
(ii) z π dθ
; a > |b|

.c
0 2 − cos θ 0 a + b cos θ
Sol. Consider the integration round a unit circle C ≡ |z| = 1 so that z = eiθ

∴ dz = ieiθ dθ = iz dθ
n
⇒ dθ = ts dz
iz
FG IJ
ra

1 iθ 1 1
Also, cos θ = (e + e − iθ ) = z+
2 2 z H K
pi

Then the given integral reduces to

z LM 1 FG dz IJ = z 2z dz FG IJ
H K
as

I=
bF 1I O H iz K
2
bz + 2 az + b iz
a + Gz + JP
C C

N 2H zK Q
g

2
z dz 2
z dz
.c

= =
ib C 2 2a ib C ( z − α) ( z − β)
z + z+1
b
w

a a 2 − b2 a a 2 − b2
where, α= −
+ and β = − −
w

b b b b
Poles are given by (z – α) (z – β) = 0 ⇒ z = α, β
w

Both are simple poles.


Since a > |b| ∴ |β| > 1
Since αβ = 1
∴ |αβ| = 1
|α| |β| = 1
⇒ |α| < 1 |∵ |β| > 1
Hence z = α is the only pole which lies inside the circle C ≡ |z| = 1.
Residue of f (z) at (z = α) is
2
R = Lt ( z − α) f ( z) = Lt ( z − α) .
z→α z→α ib ( z − α) ( z − β)

www.cgaspirants.com
www.cgaspirants.com

FUNCTION OF COMPLEX VARIABLE 105

2 2 (b) 1
= = =
ib (α − β) ib (2 a 2 − b2 ) i a − b2
2

By Cauchy’s Residue theorem,


F I
I = 2πi(R) = 2πi GG 1
JJ
Hi a 2 − b2 K
z
0
2π dθ
a + b cos θ
=

a − b2
2
...(1)

(i) Putting a = 2 and b = – 1 in (1), we get

m
z2π dθ
=

= 2π

o
0 2 − cos θ 2−1

.c
(ii) From (1),

2 zπ

0

a + b cos θ
=

a − b2
2
ts | Using prop. of definite integrals

z
n
π dθ π
⇒ = .
a + b cos θ
ra
0
a − b2
2

Example 2. Evaluate by contour integration:

z
pi

2π dθ
, where a > |b| [U.P.T.U. (C.O.) 2010; G.B.T.U. 2012]
0 a + b sin θ
as

Hence or otherwise evaluate z0


2π dθ
1 − 2a sin θ + a 2
, 0 < a < 1.
g

Sol. Consider the integration round a unit circle C ≡ |z| = 1


.c

dz
so that z = eiθ ∴ dθ = .
w

iz
1 iθ 1 1 FG IJ
w

Also, sin θ = (e − e − iθ ) =
2i 2i
z−
z H K
w

Then the given integral reduces to

I=
z LM
C b F
1 FG dz IJ =
G z − 1z IJK OPQ H iz K
zC bz 2
2iz FG dz IJ
+ 2iaz − b H iz K
2i H
a+
N
=
2
b z C
z2 +
dz
2ia
b
z−1

Poles are given by


2ia
z2 + z−1=0
b

www.cgaspirants.com
www.cgaspirants.com

106 A TEXTBOOK OF ENGINEERING MATHEMATICS

− 2ia − 4a2
± +4
b b2 − ia b2 − a 2
⇒ z= = ±
2 b b
2 2
− ia i a − b
= ± = α, β (simple poles)
b b
2 2 2 2
− ia i a − b − ia i a − b
where, α= + and β= −
b b b b
Clearly, |β| > 1
But αβ = – 1

m
∴ |α β| = 1 ⇒ |α| |β| = 1 ⇒ |α| < 1
Hence z = α is the only pole which lies inside circle C ≡ |z| = 1.

o
Residue of f (z) at (z = α) is
2 2

.c
R = Lt ( z − α) . =
z→α b ( z − α) ( z − β) b (α − β)

=
b
n F 2i
GG
2
a 2 − b2 I
JJ
=
ts
1
i a 2 − b2
H b K
ra

∴ By Cauchy’s Residue theorem,


F I
pi

I = 2πi (R) = 2πi GG 1


JJ = 2π
Hi 2
a −b 2
K a − b2
2
as

∴ z0
2π dθ
a + b sin θ
=

a − b2
2
...(1)
g

If we replace a by 1 + a2 and b by – 2a, then


.c

z 2π dθ
=

=

=

.
w

0 (1 + a 2 ) − 2a sin θ (1 + a ) − 4 a2 2 2 4
1 + a − 2a 2 1 − a2

Example 3. Use contour integration method to evaluate the following integral:


w

z π a dθ
, (a > 0).
w

0 a + sin 2 θ
2

Sol. I=
z
0
π

a2 +
(1
a dθ
− cos 2θ)
2

= 2a
z π

0 2

(2 a + 1) − cos 2θ
Put 2θ = φ, dθ =

2

=a z 2π

0 2

(2 a + 1) − cos φ

I = 2a z 2π

0

(4 a + 2) − ( e iφ + e − iφ )
2
...(1)

www.cgaspirants.com
www.cgaspirants.com

FUNCTION OF COMPLEX VARIABLE 107

dz
But z = eiφ so that dφ = then (1) reduces to
iz

I = 2a
zC
(4 a 2
F
1
.
1 I iz
+ 2) – G z + J
H zK
dz
=
2a
i zC
dz
4a2 z + 2 z − z2 − 1

= 2ai zC 2
z − 2 z(1 + 2 a ) + 1
dz
2
= 2ai z C
dz
( z − α)( z − β)

where, α = (1 + 2a2) + 2a 1 + a 2 and β = (1 + 2a2) – 2a 1 + a 2


Clearly, |α|>1

m
∵ | αβ | = 1 ∴ |β|<1
∴ Only β lies inside C.

o
2ai 2ai 2 ai −i

.c
Residue (at z = β) is = Lt ( z − β) . = = =
z→β ( z − α)( z − β) β−α − 4a 1 + a 2
2 1 + a2
By Cauchy Residue theorem,
n
F −i I
ts
I = 2πi GG JJ = π
.
ra

H2 1+ a 2
K 1 + a2

Example 4. Apply Calculus of residues to prove that


pi

z 2π dφ
= 2
2 πa
where a > 0, b > 0, a > b.
as

2
0 (a + b cos φ ) (a − b 2 ) 3/2

z 2π dφ
z 2π dφ
g

Sol. Let, I= = ...(1)


0 (a + b cos φ) 2 0 RSa + b (e U
)V
2
.c

iφ – iφ
+e
T 2 W
w

dz
Put eiφ = z so that dφ = then,
iz
w

From (1), I= z RS 1
IJ UV
dz
= z − 4izdz
w

C b FG1
2
iz C (bz + 2az + b) 2
2

T a+
2
z+
Hz KW
=–
4i
b2 z FGH
C
z2 +
z dz
2az
+1
IJ
K
2

b
Poles are given by,

FG z 2 2az IJ 2
=0 ⇒ (z – α)2(z – β)2 = 0 where, α + β = –
2a
and αβ = 1.
H +
b
+1
K b

www.cgaspirants.com
www.cgaspirants.com

108 A TEXTBOOK OF ENGINEERING MATHEMATICS

2a 4a2
− + −4
b b2 − a + a 2 − b2
Also, α= =
2 b
4a2 2a
− −4 –
b2 b − a − a 2 − b2
β= =
2 b
There are two poles, at z = α and at z = β each of order 2.
Since, | αβ | = 1
or |α||β|=1
But |β|>1 ∴ |α|<1 |∵ a>b

m
∴ Only z = α lies inside the unit circle | z | = 1.
Residue of f(z) at the double pole z = α is

o
LM R UVOP

.c
1 d − 4 iz
MN ST
= ( z − α) 2 . 2
(2 − 1) ! dz b ( z − α) 2 ( z − β) 2 WPQ z=α

=
LM d R − 4iz UOP
S
MN dz T b ( z − β) WPQ
n
2 V =–
b N
2 .M
z=α
ts
4i L ( z − β) . 1 − z . 2( z − β) O
( z − β) 2 PQ
2

4
z=α
ra

4i L (− β − z) O
=– 2 M
b N ( z − β) Q
P = – b4i ((−αα− −β)β) = b4i ((αα−+ββ))
3 2 3 2 3
z=α
pi

FG − 2a IJ
4i H b K = – ia
as

= .
b F2
GH b a − b IJK
2 3 2 2 3/2
2 (a − b )
2
g

I = 2πi M
L − ia OP = 2πa .
.c


N (a − b ) Q (a − b )
2 2 3/ 2 2 2 3/ 2
w

Example 5. Apply Calculus of residues to prove that

z π cos 2θ dθ πp 2
w

= (0 < p < 1).


0 1 − 2p cos θ + p 2 1 − p2

z z
w

π cos 2θ dθ 1 2π cos 2θ dθ
Sol. I= 2 =
0 1 − 2 p cos θ + p 2 0 1 − p( e iθ + e − iθ ) + p2

=
1
2
real part of z 2π

0
e 2 iθ
(1 − pe iθ )(1 − pe − iθ )

=
1
2
real part of z C
(1 − pz) 1 −
z2
FG
H
p
z
IJ
K
dz
iz
writing e iθ = z, dθ =
dz
iz

=
1
2
real part of z C
− iz 2
(1 − pz)( z − p)
dz

www.cgaspirants.com
www.cgaspirants.com

FUNCTION OF COMPLEX VARIABLE 109

=
1
2
real part of zC
f ( z) dz where, f ( z) =
− iz 2
(1 − pz)( z − p)
Poles of f(z) are given by (1 – pz)(z – p) = 0.
1
Thus z = and z = p are the simple poles. Only z = p lies within the unit circle C as
p
p < 1.
The residue of f(z) at z = p is
− iz 2 ip2
= lim (z – p) f(z) = lim (z – p) =−
z→ p z→ p (1 − pz)( z − p) 1 − p2

m
Hence by Cauchy’s residue theorem, we have

z f ( z) dz = 2πi × [Sum of residues within the contour]

o
C

F I = 2πp

.c
ip2 2
= 2πi −GH 1− p 2 JK 1 − p 2
which is purely real.

Hence, I=
1
2
real part of
n
zC
f ( z) dz =
πp2
ts
1 − p2
.

Example 6. Use Complex integration method to prove that


ra

z 2π sin 2 θ 2π
dθ = 2 (a − a 2 − b 2 ) , where 0 < b < a.
pi

0 a + b cos θ b

Sol. Let I=
z 2π sin 2 θ
z 2π 1 − cos 2θ
as

dθ = dθ
0 a + b cos θ 0 2(a + b cos θ)

z 2π 1 − e 2iθ
g

= Real part of dθ
0 2a + 2b cos θ
.c

1 FG
1 dz IJ
Put z = eiθ so that cos θ = z+
Hand dθ =
K
w

2 z iz

z 2π 1 − e 2iθ
z 1 − z2FG dz IJ =
z 1 − z2
w

Then
0 2a + 2b cos θ
dz =
C F 1 I H iz K
2a + b G z + J
C i(bz 2 + 2az + b)
dz

H zK
w

where C is the circle | z | = 1.


The poles of the integrand are the roots of bz2 + 2az + b = 0, viz.

− 2 a ± 4 a 2 − 4b2 − a ± a 2 − b2
z= =
2b b

− a + a 2 − b2 − a − a 2 − b2
Let α= and β =
b b
Clearly, | β | > 1 so that z = α is the only simple pole inside C.

www.cgaspirants.com
www.cgaspirants.com

110 A TEXTBOOK OF ENGINEERING MATHEMATICS

Also, bz2 + 2az + b = b(z – α)(z – β)


Residue at z = α is
1 − z2 1 − z2 1 − α2
Lt ( z − α) . = Lt =
z→a ib( z − α)( z − β) z → α ib( z − β) ib(α − β)
FG 1 − αIJ
=
α
H α K = α(β − α) |∵ αβ = 1
ib(α − β) ib(α − β)
2 2
α a− a −b
=– =
ib ib2

m
∴ By Residue theorem,

z 1 − z2 a − a 2 − b 2 2π FH IK

o
dz = 2 πi . = 2 a − a 2 − b2
C i(bz 2 + 2az + b) ib2 b

.c
Hence I = Real part of
z 1 − z2 2π
dz = 2 a − a 2 − b2 . FH IK
ts
2
C i(bz + 2az + b) b

z 2π cos 2θ
n
Example 7. Using complex integration method, evaluate dθ .
0 5 + 4 cos θ
ra

(M.T.U. 2012, G.B.T.U. 2010)

z 2π e 2iθ
pi

Sol. Let I = Real part of dθ


0 5 + 2 ( e iθ + e − iθ )
as

= Real part of z
C 1
z2

5 + 2Gz + J
H zK
FG dz IJ
F I H iz K
writing e iθ = z
∴ dθ =
dz
g

iz

z
.c

1 z2
= Real part of 2
dz
i C 2z + 5z + 2
w

Singularities are given by


w

1
2z2 + 5z + 2 = 0 ⇒ z=– ,–2
2
w

1
z= − is the only pole which lies inside the unit circle C ≡ |z| = 1.
2
FG 1 IJ
H
Residue of f (z) at z = −
2
is
K
R=
FG z + 1 IJ . z 2
z2 1 1 FG IJ FG 2 IJ = 1
z→−
Lt
1
2
H 2 K i (2z + 1) (z + 2) = Lt
z→−
1
2
2i ( z + 2)
=
2i 4 H K H 3 K 12 i
Hence by Cauchy’s Residue theorem,

I= z
C
f ( z) dz = 2πi
FG 1 IJ = π .
H 12 i K 6

www.cgaspirants.com
www.cgaspirants.com

FUNCTION OF COMPLEX VARIABLE 111

Example 8. Evaluate: z
0
π 1 + 2 cos θ
5 + 4 cos θ

Sol. Let I= z 0
2π 1 + 2 cos θ
5 + 4 cos θ
dθ = Real part of
z
0
2π 1 + 2e iθ
5 + 4 cos θ

= Real part of z
C
1 + 2z

5 + 2G z + J
H zK
FG dz IJ
F 1 I H iz K
Putting e iθ = z
∴ dθ =
dz
iz

= Real part of
1
i z
C 2
1 + 2z
2z + 5z + 2
dz

m
Poles are given by
1
(2z + 1) (z + 2) = 0 ⇒ z=– , – 2 (simple poles)

o
2

.c
1
z=– lies inside unit circle C ≡ | z | = 1
2
FG IJ FG IJ FG 1 IJ
Residue at z = −
H
1
2 z→−
K
= Lt z +
1

Hence by Cauchy’s Residue theorem,


1 1
.
1 + 2z
H
2 i (2 z + 1) ( z + 2)
n
2
=
K ts H 2i K z→ −
Lt
1
2
1 + 2z
z+2
=0
ra

I = 2πi (0) = 0

z2π 1 + 2 cos θ
dθ = 0
pi

0 5 + 4 cos θ

z π 1 + 2 cos θ
as

⇒ dθ = 0 | Using property of definite integrals


0 5 + 4 cos θ

z 2π
g

Example 9. Evaluate by Contour integration: e cos θ cos (sin θ – nθ) dθ.


0
.c

Sol. Let I= z 2π
e cos θ [cos (sin θ – nθ) + i sin (sin θ – nθ)] dθ
w

= z 2π
e cos θ . ei(sinθ–nθ) dθ =
z 2π iθ
e e . e − in θ dθ ...(1)
w

0 0

dz
w

Put eiθ = z so that dθ = then,


iz

Poles are given by


I= z C
ez .
1
zn
.
dz
iz
=–i zC z n+1
ez
dz

z=0 [of order (n + 1)]


It lies inside the unit circle.
Residue of f(z) at z = 0 is

R=
1 dn LM R|Sz n +1 − ie z U|VOP =
LM
− i dn
(e z )
OP =
−i
.
(n + 1 − 1) ! dz n MN T| z n +1 W|PQ z= 0
N
n ! dz n Q z=0
n!

www.cgaspirants.com
www.cgaspirants.com

112 A TEXTBOOK OF ENGINEERING MATHEMATICS

∴ By Cauchy’s Residue theorem,

I = 2πi
FG − i IJ = 2π
H n !K n !
Comparing real parts, we have

z0

e cos θ cos (sin θ – nθ) dθ =

n!
.

Example 10. Evaluate the integral: z 0


π cos 2 3 θ
5 – 4 cos 2 θ
dθ (U.P.T.U. 2015)

1
z 2π cos 2 3θ 1
z 2π 1 + cos 6θ

m
Sol. Let, I= dθ = dθ ...(1)
2 0 5 – 4 cos 2θ 4 0 5 – 4 cos 2θ
Consider the integration round a unit circle c ≡ |z| = 1 so that z = eiθ

o
dz

.c
∴ dz = ieiθ dθ = iz dθ ⇒ dθ =
iz
FG IJ
Also, cos 2θ =

1 6 1
1 2iθ
2
n (e + e – 2iθ) =

FG
1 2
2 z
1
z + 2

IJ
ts
H K
and cos 6θ = z + 6
H K
ra

2 z
Then the given integral (1) reduces to
pi

F1 + z + 1 I
12

GH 2z JK dz 1
1
z 6

z z12 + 2 z6 + 1
as

I= ⋅ =– dz
4 c F
5 – 2G
z + 1 I iz
4 16i c FG 5
z5 z4 – z2 + 1
IJ
H z K J 2 H 2 K
g

1
.c

Singularities are, z = 0 (order 5), z = ± 2, ± (order 1)


2
w

1
Clearly, z = 0 and z = ± lie inside C.
2
w

1
Now we will find residues at z = 0 and z = ± .
w

z
12 6
+ 2z + 1
12
z + 2z + 1
6
LM1 – FG 5 z 2
–z
4 IJ OP –1
Let f(z) =
5 FG 4 5 2 IJ =
z5 N H2 KQ
z
H z – z +1
2 K
=
( z 6 + 1) 2 LM1 + 5 z 2
– z4 +
25 4
z + z 8 – 5 z 6 + ...
OP
z
5
N 2 4 Q
1
Residue of f(z) at z = 0 is the coefficient of in this laurent series expansion. Hence
z
25 21
R1 = Residue of f(z) at z = 0 = – 1 + =
4 4

www.cgaspirants.com
www.cgaspirants.com

FUNCTION OF COMPLEX VARIABLE 113

1
R2 = Residue of f(z) at z =
2

= Lt.
FG z – 1 IJ . ( z 6 + 1) 2
z→
1 H 2K z 5
( z 2 – 2) z −
FG 1 IJ FG z + 1 IJ
2 H 2 K H 2K
( z6 + 1) 2 27
= Lt. =–
1 F
– 2) G z +
1 I 8
z5 ( z2
H 2 JK
z→
2

1
R3 = Residue of f(z) at z = –

m
2

= Lt
FG z + 1 IJ ⋅ ( z 6 + 1) 2
H 2K z FG IJ FG z + 1 IJ

o
z→ –
1 5 1
( z 2 – 2) z –
H K H 2K

.c
2
2
( z 6 + 1) 2 27

Now, by Cauchy-Residue theorem,


= Lt
z→ –
n 1
2 z5 ( z 2 – 2) z –
FG
H ts 1
2
IJ
K
=–
8
ra

1 π 21 27 3π FG IJ
I= −
16 i
[2π i (R1 + R 2 + R 3 )] = −
8 4

4
=
16
.
H K
pi

ASSIGNMENT
as

Evaluate the following integrals by using contour integration:

z z
g

2π dθ 2π dθ
1. (i) (U.K.T.U. 2011) (ii) (U.P.T.U. 2015)
.c

0 5 − 3 cos θ 0 2 + cos θ

zπ dθ
z π dθ
w

2. (i) (ii)
0 5 + 4 cos θ 0 17 − 8 cos θ

z
w

π a dθ
(iii) [G.B.T.U. (C.O.) 2011]
0 1 + 2a2 − cos 2θ
w

3. (i)
z
0
2π dθ
5 + 4 sin θ
(G.B.T.U. 2011) (ii)
z
0
π dθ
3 + sin 2 θ
(G.B.T.U. 2013)

4. (i)
z
0
2π dθ
(5 − 3 cos θ) 2
(ii)
z
0
π dθ
(a + cos θ) 2

5. (i)
z
0
π
2
a dφ
a + cos φ 2
(a > 0) (ii)
z
0
2π cos 2θ
1 − 2 p cos θ + p2
dθ , 0 < p < 1

6. (i)
z
0
2π cos 3θ
5 − 4 cos θ
dθ (U.P.T.U. 2009) (ii)
z
0
π cos 3θ
5 − 4 cos θ

www.cgaspirants.com
www.cgaspirants.com

114 A TEXTBOOK OF ENGINEERING MATHEMATICS

(iii)
z 0
2π cos 3θ
5 + 4 cos θ
dθ (U.P.T.U. 2007) (iv)
z0
π cos 2θ
5 + 4 cos θ
dθ (U.P.T.U. 2014)

7.
z π a cos θ
− π a + cos θ
dθ ; a > 1 8.
z0
2π sin 2 θ
5 − 4 cos θ

9.
z0

e − cos θ cos (nθ + sin θ) dθ ; n ∈ I

10. (i)
z0

cos 2n θ dθ ; n ∈ I (ii)
z0
2π dθ
3 − 2 cos θ + sin θ
[G.B.T.U. 2013; U.P.T.U. 2014]

m
Answers
π 2π
1. (i) (ii)

o
2 3

.c
π π π
2. (i) (ii) (iii)
3 15 2 1 + a2

3. (i)

3

(ii)
2 3
π
n
πa
ts π 2πp2
4. (i) (ii) 5. (i) (ii)
ra
2 3/2
32 (a − 1) 1 + a2 1 − p2
π π π π
6. (i) (ii) (iii) − (iv)
pi

12 24 12 12
F I 2π
7. 2πa 1 − GG a
J 8.
π
9. (− 1) n
− 1 JK
as

n!
H a2 4

π (2n) !
10. (i) (ii) π.
g

2n − 1
(2) (n !) 2
.c

1.42.2. Integrals of the Type z ∞ f(x)


dx , where f(x) and F(x) are Polynomials in x
w

−∞ F(x)
x f(x)
such that → 0 as x → ∞ and F(x) has no Zeros on the Real axis.
w

F(x)

Consider the integral


f ( z)
z
dz over the closed contour C consisting of the real axis from – R
w

C F( z)
to R and the semi-circle CR of radius R in the upper half plane.

CR

–R O R X

www.cgaspirants.com
www.cgaspirants.com

FUNCTION OF COMPLEX VARIABLE 115

f ( z)
We take R large enough so that all the poles of in the upper half plane lie within C.
F( z)
By residue theorem, we have

zC
f ( z)
F( z)
LM
dz = 2πi sum of the residues of
N
f ( z)
F( z)
in the upper half plane
OP
Q
zCR
f ( z)
F( z)
dz + z
f ( x)
F( x)
R
dx = 2πi sum of the residues of
–R
f ( z)
F( z)
LM
in the upper half plane
N
...(1) (∵ on the real axis, z = x)
OP
Q
If we put z = Reiθ in the first integral on the left side, then R is constant on CR and as z
moves along C1, θ varies from 0 to π.

z z

m
f ( z) π f (Re iθ )
∴ dz = Re iθ i dθ
CR F( z) 0 F(Re iθ )

o
z π f (Re iθ )
Re iθ idθ → 0
Rf (R)

.c
For large R, is of the order
0 F(Re iθ ) F(R)


F(Re iθ )
Hence from (1), we have
z
0
π f (Re iθ ) n Reiθ idθ → 0 when R → ∞
ts
z
ra
∞ f ( x) LM
dx = 2πi sum of the residues of
f ( z)
in the upper half plane
OP
−∞ F( x) N F( z) Q
pi

EXAMPLES
as

Example 1. Using contour integration, prove that z


−∞
∞ dx
(1 + x 2 ) 2
=
π
2
.
g

z ∞ dx
.c

Hence or otherwise evaluate .


0 (1 + x 2 ) 2

z 1
w

Sol. Consider the integral f ( z) dz where f(z) =


taken round the closed contour
(1 + z 2 ) 2
C
w

C consisting of the semi-circle CR which is upper half of a large circle | z | = R and the part of
real axis from – R to R.
w

Y
For poles, (1 + z2)2 = 0
⇒ z2 = –1
⇒ z = ± i (Poles of order 2) CR
z = – i is outside C.
So z = i is the only pole inside C and is of order 2.
Residue of f(z) at z = i is
LM d R( z − i) 1 UVOP
MN dz ST
2
–R O R X
= .
( z − i) ( z + i) 2
2
WPQ z= i

L − 2 OP
=M =−
i
N (z + i) Q 3
z= i
4

www.cgaspirants.com
www.cgaspirants.com

116 A TEXTBOOK OF ENGINEERING MATHEMATICS

By Cauchy’s residue theorem,


R
z dx
− R (1 + x )
Taking limit as R → ∞
2 2
+ zdz
2
C R (1 + z ) 2 = 2πi
−i
4
=
π
2
FG IJ
H K

z dx
−∞ (1 + x 2 ) 2
+ Lim z dz
R → ∞ C R (1 + z 2 ) 2
=
π
2
...(1)

Now, z
CR
dz
(1 + z 2 ) 2
≤ z
C R |1 +
| dz|
z 2 |2

≤ z
CR
|dz|
{| z|2 − 1}2

m
∵ | z|= R on C R
= z π R dθ and| dz|= R dθ

o
0 (R 2 − 1) 2 also, 0 < θ < π

.c
πR
= → 0 as R → ∞
(R 2 − 1) 2

Hence,

zdx
−∞ (1 + x 2 ) 2
=
π
2
n ts

zdx
=
π ∞ dx π
= . z
ra
Now, 2 0 2 2 ⇒ 0 2 2
(1 + x ) 2 (1 + x ) 4
Example 2. Apply calculus of residues to prove that

z
pi

∞ dx π
2 2 2
= ; a > 0. (M.T.U. 2013)
0 (x + a ) 4a 3

z
as

Sol. Consider the integral f ( z) dz where Y


C
1
g

f(z) = taken round the closed contour C consist-


(a + z 2 ) 2
2
.c

CR
ing of the semi-circle cR which is upper half of a large circle
| z | = R and the part of real axis from – R to R.
w

Poles of f(z) are given by


w

(a2 + z2)2 = 0 –R O R X
i.e., 2 2
a +z =0
w

or z = ± ai each repeated twice.


The only pole within the contour is z = ai, and is of the order 2.
1
Here, f(z) = .
( z − ai) 2 ( z + ai) 2
1 d LM R 1 UVOP
MN ST
Residue (at z = ai) is = ( z − ai) 2 .
(2 − 1) ! dz ( z − ai) 2 ( z + ai) 2 WPQ z = ai

L d R 1 UOP
=M
L − 2 OP
MN dz ST ( z + ai) VWPQ 2
z = ai
=M
N (z + ai) Q
3
z = ai

−2 −1 1
= 3
= 3 3
= .
(2ai) 4a i 4 a3 i

www.cgaspirants.com
www.cgaspirants.com

FUNCTION OF COMPLEX VARIABLE 117

Hence by Cauchy’s residue theorem, we have

z
C
f ( z) dz = 2πi (Sum of residues within C)

i.e., z
−R
R
f ( x) dx + z
CR
f ( z) dz = 2πi
FG 1 IJ
H 4a i K
3

or z R

−R
1
(a 2 + x 2 ) 2
dx + z CR
1
(a 2 + z 2 ) 2
dz =
π
2a3
...(1)

Now, zCR
1
(a + z 2 ) 2
2
dz ≤ z | dz|
C R | a + z 2 |2
2

z | dz|

m

CR (| z|2 – a 2 ) 2

o
π R dθ
= , since z = Reiθ
0 (R 2 − a 2 ) 2

.c
πR
=
→ 0 as R → ∞
(R 2 − a 2 ) 2
Hence taking R → ∞, relation (1) becomes,
n ts
z ∞ 1
dx =
π
ra
2
2 2
−∞ ( a + x ) 2a3

z ∞ 1
dx =
π
pi

or 2 . 2 2
(a + x )
0 4 a3
Example 3. Apply Calculus of residues to prove that
as

z ∞
2 2
x2
2 2
−∞ (x + a )(x + b )
dx =
π
a+b
(a > 0, b > 0).
g

z z2
.c

Sol. Consider the integral f ( z) dz where f(z) =


taken round the
( z 2 + a 2 )( z 2 + b2 )
C
w

closed contour C consisting of the semi-circle CR which is upper half of a large circle | z | = R
and the part of real axis from – R to R.
w

z2
The poles of f(z) = are z = ± ia, ± ib. Of these, z = ia and z = ib lie in the
( z + a )( z 2 + b2 )2 2
w

upper half of the z-plane.


Residue of f(z) at z = ia is
z2
= lim ( z − ia)
z → ia ( z + a )( z 2 + b2 )
2 2

z2 a2 a
= lim 2 = – 2 = 2 .
z → ia z
( + ia)( z 2
+ b ) ia
2 (− a 2
b
+ ) 2i ( a – b2 )
Residue of f(z) at z = ib is
z2
= lim ( z − ib)
z → ib ( z + a )( z 2 + b2 )
2 2

www.cgaspirants.com
www.cgaspirants.com

118 A TEXTBOOK OF ENGINEERING MATHEMATICS

z2 − b2 −b
= lim 2 2 = = .
z → ib ( z + a )( z + ib) (− b + a 2 )(2ib)
2
2i (a 2 − b2 )
By Cauchy’s residue theorem,
LM a − b OP
z−R
R
2
( x + a )( x + b ) 2
x2
2 2
dx + zCR 2 2
( z + a )( z + b )
z2
2 2
dz = 2πi
MN 2i (a − b ) 2i (a − b ) PQ 2 2 2 2

Taking limit as R → ∞

z ∞
2
x2
−∞ ( x + a )( x 2 + b 2 )
2
dx + Lt
R→∞ z CR
z
2
( z + a )( z + b ) 2
2
dz = π M
L a − b OP
Na − b Q
2 2 2 2

z ∞ x2
z z2 π

m
⇒ dx + Lt dz = ...(1)
−∞ ( x 2 + a 2 )( x 2 + b2 ) R→∞ CR 2 2 2 2
( z + a )( z + b ) a+b

z z2
z | z|2

o
Now, dz ≤ |dz|
CR ( z 2 + a 2 )( z 2 + b2 ) CR | z 2 + a 2 || z 2 + b 2 |

.c
≤ z | z|2
| dz|

=
n
R2
(R 2 − a 2 )(R 2 − b2 )
CR

z0
πts
(| z|2 – a 2 )(| z|2 – b2 )

R dθ | ∵ | z | = R on CR
ra

πR 3
= → 0 as R → ∞
(R 2 − a 2 )(R 2 − b2 )

z
pi

x2 ∞ π
∴ From (1), dx = .
−∞ ( x 2 + a 2 )( x 2 + b2 ) a+b
as

Example 4. (i) Apply Calculus of residues to prove that

z∞ dx π 2
g

44
= , (a > 0).
0 x +a 4a 3
.c

(ii) Using contour integration, evaluate z ∞ dx


1 + x4
. (U.P.T.U. 2007)
w

Sol. (i) Consider the integral z f ( z) dz where f(z) =


1
.
w

C z + a4
4

The poles of f(z) are given by


w

z4 + a4 = 0 ⇒ z4 = – a4 = a4eπi = a4e2nπi+πi
or z = ae(2n+1)πi/4 ; n = 0, 1, 2, 3.
Since there is no pole on the real axis, therefore, we may take the closed contour C
consisting of the upper half CR of a large circle | z | = R and the part of real axis from – R to R.
∴ By Cauchy’s residue theorem, we have

z R

−R
f ( x) dx + z CR
f ( z) dz = z C
f ( z) dz

or z R

−R
1
x4 + a4
dx + z
CR
1
z4 + a4
dz = 2πi ∑R +
...(1)

| where ∑R +
= sum of residues of f(z) at poles within C.

www.cgaspirants.com
www.cgaspirants.com

FUNCTION OF COMPLEX VARIABLE 119

πi
The poles z = ae 4 and z = ae3πi/4 are the only two poles which lie within the contour C.
Let α denote any one of these poles, then
α 4 + a4 = 0 ⇒ α4 = – a 4 .

Residue of f(z) (at z = α) is = LM 1 OP =


1
=
α

MN dzd (z + a )PQ
3
4 4 4α − 4a4
z =α
1
∴ Residue at z = aeπi/4 is = − eπi/4
4 a3
1 e −πi/4
and residue at z = ae3πi/4 is = − e3πi/4 =

m
4a3 4 a3
1 LM e iπ/4
− e − iπ/4 1 OP i
π
∑R +

o
∴ Sum of residues = − =– i sin =− = .
2a 3
N 2 2a 3
Q 4 2 2 a3

.c
∴ From (1),

z −R
R
4
x +a
dx
4
+ zCR z +a4
dz
4
= 2 πi
n F –i I= π 2
GH 2 2 a JK 2a 3 3
ts ...(2)

Now,
z 1
dz ≤ z | dz|
≤ z | dz|
ra
4 4 CR | z 4 4 C R | z|4
CR z +a +a | −|a 4 |

= z π Rdθ
| ∵ |z | = R on CR
pi

0 R − a4
4

πR
= → 0 as R → ∞.
as

R − a4 4

Hence when R → ∞, relation (2) becomes


g

z ∞ dx π 2
z ∞ dx π 2
.c

4 4
= or = .
−∞ x +a 2a3 0 x +a44
4 a3

z
w

1
(ii) Consider the integral f ( z) dz where f(z) =
taken round a closed contour C,
1 + z4 C
w

consisting of the semi-circle CR which is upper half of a large circle | z | = R and the part of
real axis from – R to R.
w

1
The poles of f(z) = 4 are obtained by solving z4 + 1 = 0.
z +1
Now z4 + 1 = 0
⇒ z = (– 1)1/4 = (cos π + i sin π)1/4 = [cos (2nπ + π) + i sin (2nπ + π)]1/4
(2n + 1) π (2n + 1) π
= cos + i sin where n = 0, 1, 2, 3.
4 4
| By De Moivre’s theorem
π π 1 1
When n = 0, z = cos + i sin = +i
4 4 2 2

www.cgaspirants.com
www.cgaspirants.com

120 A TEXTBOOK OF ENGINEERING MATHEMATICS

3π 3π 1 1
When n = 1, + i sin
z = cos =− +i
4 4 2 2
5π 5π 1 1
When n = 2, z = cos + i sin =− −i
4 4 2 2
7π 7π 1 1
When n = 3, z = cos + i sin = −i
4 4 2 2
π π
i 3i
Of these, only the poles corresponding to n = 0, 1, viz, z = e 4 and z = e 4 lie in the
upper half of z-plane.
π
π i
z−e 4 0

m
i
Residue of f(z) at z = e 4 is Lt 4
Form
i
π
z +1 0
z→ e 4

o
1
= Lt | By L’ Hospital’s rule
4 z3

.c
π
i
z→e 4

π
=
n
4e
1
3i
π
4
=

3i
1 −3i 4
4
π
e

−9 i
π
ts
ra
Similarly, residue of f(z) at z = e 4 is 41 e 4
1 –3iπ/4
Sum of residues = (e + e–9iπ/4)
4
pi

=
1
cos

− i sin
3π LM
+ cos

− i sin
9π OP
4 4 4 N 4 4 Q
as

1F
= G−
1

i
+
1

i IJ = − i 2
4H 2 2 2 2K 4
g

By Cauchy Residue theorem


.c

z R dx
+ z dz
= 2πi
F− i 2I = π 2
GH 4 JK 2
w

4 4
−R 1+ x CR 1+ z
Taking Limit R → ∞,
w

z ∞ dx
+ Lt z dz
=
π 2
...(1)
w

4 4
−∞ 1+ x R → ∞ CR 1+ z 2

Now, zCR
dz
1 + z4
≤ z
CR | z 4
| dz|
+ 1|
≤ zC R | z|4
| dz|
−1

=
R −1
πR
4
R
z 0
π
dθ |∵ | z | = R on CR

= → 0 as R → ∞
R4 − 1

∴ From (1), z−∞


1+ x
dx
4
=
π 2
2
or z0

Note. The above method can also be applied to some cases where f(x) contains trigonometric functions
1+ x
dx
4
=
π 2
4
.

also.

www.cgaspirants.com
www.cgaspirants.com

FUNCTION OF COMPLEX VARIABLE 121

1.42.2. (a) Jordan’s Inequality


Consider the relation y = cos θ. As θ increases, cos θ decreases and therefore y decreases.

The mean ordinate between 0 and θ =

when θ = 0, ordinate is cos 0 i.e. 1


1
θ z0
θ
cos θ dθ =
sin θ
θ

Y
π sin π/2 2
when θ = , mean ordinate is i.e.
2 π/2 π P
Hence, when 0 < θ < π/2,
2 θ y
Mean ordinate lies between 1 and

m
π
2 sin θ O X
i.e., < <1

o
π θ
This is known as Jordan’s Inequality.

.c
1.42.2. (b) Jordan’s Lemma

If f(z) → 0 uniformly as | z | → ∞, then Lt


n R → ∞ CR z ts
e imz f(z) dz = 0, (m > 0)

where CR denotes the semi-circle | z | = R, I(z) > 0.


ra
Example 5. Apply calculus of residues to evaluate

z ∞ x sin x
dx , a > 0. (G.B.T.U. 2010)
pi

0 x2 + a2

Sol. Consider the integral z z e iz


f ( z) dz where f(z) =
taken round a closed contour
as

C z2 + a2
C, consisting of a semi-circle CR which is upper half of a large circle | z | = R and the part of
real axis from – R to R.
g

For poles, z2 + a 2 = 0
.c

⇒ z = ± ai
z = ai is the only pole which lie inside C.
w

z e iz ai e − a e − a
∴ Residue of f(z) at (z = ai) = lim ( z − ai) . = =
w

z → ai ( z − ai)( z + ai) 2ai 2


∴ By Cauchy Residue theorem,
w

z−R
R
2
x +a
x e ix
2
dx + z
CR
z e iz
2
z +a 2
dz = 2πi
F e I = πie
−a

GH 2 JK –a

Taking limit as R → ∞,

Lt
R → ∞ −Rz R
2
x +a
x e ix
2
dx + Lt z
R → ∞ CR
z e iz
z + a2
2
dz = πie–a

z
z

−∞ 2
x e ix
x +a 2
dx + Lt
R → ∞ CRz z e iz
z + a2
2
dz = πie–a ...(1)

Since → 0 as | z | → ∞, therefore by Jordan’s Lemma,


z + a2
2

www.cgaspirants.com
www.cgaspirants.com

122 A TEXTBOOK OF ENGINEERING MATHEMATICS

Lt
R → ∞ CR z ze iz
z + a2
2
dz = 0

∴ From (1),
x e ix
−∞ x 2 + a 2 z∞
dx = πi e–a

Comparing imaginary parts,

z∞

−∞
x sin x
x 2 + a2
dx = πe–a

or z
0
∞ x sin x
2
x +a 2
dx =
π −a
2
e .

m
Example 6. Evaluate by using Contour integration z ∞

0
cos ax
x2 + 1
dx ; a ≥ 0.

o
(U.P.T.U. 2006, G.B.T.U. 2011)

.c
Sol. Consider the integral z f ( z) dz where f(z) =
e iaz
taken round a closed contour C,
z2 + 1
C

ts
consisting of a semi-circle CR which is upper half of a large circle | z | = R and the part of real
axis from – R to R.
n
For poles, z2 + 1 = 0
ra

⇒ z=±i
z = i is the only pole which lies inside C.
pi

e iaz e−a
∴ Res. (z = i) = Lt ( z − i) . =
z→i ( z − i)( z + i) 2i
as

∴ By Cauchy Residue theorem,

z
e iax R
dx + z e iax
dz = 2 πi
F I = πe
e−a
GH JK –a
g

−R x2 + 1 CR z2 + 1 2i
.c

Taking Limit as R → ∞,

z e iax

dx + Lt
e iaz
dz = π e–a z ...(1)
w

2
−∞ x + 1 R → ∞ CR z 2 + 1

1
w

Since 2 → 0 as | z | → ∞, therefore by Jordan’s Lemma,


z +1

z
w

e iaz
Lt dz = 0 (a > 0)
R → ∞ CR z2 + 1

∴ From (1), z−∞


∞ e iax
x2 + 1
dx = π e–a

Equating real parts, we get

z ∞

−∞
cos ax
x2 + 1
dx = πe–a

or z 0
∞ cos ax
x2 + 1
dx =
π e− a
2
.

www.cgaspirants.com
www.cgaspirants.com

FUNCTION OF COMPLEX VARIABLE 123

Example 7. Apply calculus of residues to prove that

z
∞ cosh ax

0 cosh πx
1 a
dx = sec , – π < a < π.
2 2
(M.T.U. 2014)

Sol. Consider
R + i and – R + i.
z c
f ( z) dz where f(z) =
e az
cosh πz
, c is the rectangle with vertices at – R, R,

Y
f(z) has simple poles given by
cosh πz = 0
or eπz + e–πz = 0
or eπz = – e–πz = e(2n+1)πi–πz – R+ i R+ i
(2n + 1)i i

m
whence, z = ; n = 0, ± 1, ± 2. Of these poles, only z =
2 2
lies inside c.
FG IJ LM e OP

o
i az
Residue at z =
H =
K M d (cosh πz)P

.c
2 –R O R
N dz Q
X
i
z=

ts
2

e ia / 2 e ia / 2 1 ia / 2
= = = e
n
iπ π πi
π sinh πi sin
2 2
ra

By residue theorem, we get

z z R
z 1
z −R
z 0
pi

f ( z) dz = f ( x) dx + f (R + iy) . idy + f ( x + i) dx + f (− R + iy) . idy


C −R 0 R 1
1 ia/2
as

= 2πi .
e = 2eia/2 ...(1)
πi
or I1 + I2 + I3 + I4 = 2eia/2.

z
g

1 e a( R + iy)
Now, | I2 | = idy
.c

0 cosh π (R + iy)

z
w

1 2e aR| e aiy ||i| dy



0 | e π (R + iy) + e – π (R + iy) |
w

2 e aR dy
= = z2e aR
1

e πR − e − πR
→ 0 as R → ∞ | since – π < a < π.
w

0 e πR − e − πR

In the same way I4 → 0. Hence when R → ∞, we get from (1),

z−∞
∞ e ax
cosh πx
dx + z ∞
−∞ e a( x + i)
cosh π ( x + i)
dx = 2eia/2

or z
−∞
∞ e ax
cosh πx
dx − z ∞

−∞
e ax . e ai
− cosh πx
dx = 2eia/2 [∵ cosh π(x + i) = – cosh πx]

⇒ z −∞
∞(1 + e ia ) . e ax
cosh πx
dx = 2e ia / 2

⇒ z ∞

−∞
(e ia / 2 + e − ia / 2 ) e ax
cosh πx
dx = 2

www.cgaspirants.com
www.cgaspirants.com

124 A TEXTBOOK OF ENGINEERING MATHEMATICS

or
e ax
−∞ cosh πx z
dx =
1
cos
a
2

or
0 e ax
−∞ cosh πx z
dx +
∞ e ax
0 cosh πx
dx =z 1
cos
a
2
...(2)

Putting x = – t in the first integral, we get


z
e ax
−∞ cosh πx
From (2),
0
dx = − z 0


e − at
cosh πt
dt = z0
∞ e − ax
cosh πx
dx

m
∞ e ax + e − ax 1
dx =
0 cosh πx a
cos

o
2

z ∞ cosh ax 1 1 a

.c
or dx = = sec .
0 cosh πx a 2 2
2 cos
n
Example 8. Using contour integration, prove that:
2
ts z
0
∞ log (1 + x 2 )
1 + x2
dx = π loge 2.
ra

Sol. Consider the integral


C z log ( z + i)
f ( z) dz where f(z) =
z2 + 1
taken round a closed con-
pi

tour C which consists of semi-circle CR, the upper half of a large circle | z | = R and the part
of real axis from –R to R.
as

For poles, z2 + 1 = 0 ⇒ z = ± i
Only the pole z = i lies inside C.
g

π
log ( z + i) log (2i) log 2 + i
.c

2
Res. (z = i) = Lt ( z − i) . = =
z→i ( z − i)( z + i) 2i 2i
w

∴ By Cauchy Residue theorem,

Lt z R log ( x + i)
dx + Lt z log ( z + i)
dz =
2 πi FG
H
log 2 + i
π IJ
πi
K FG IJ
w

= π log 2 + ...(1)
R→∞ −R 1 + x2 R → ∞ CR 1 + z2 2i 2 2 H K
LM OP
w

z log ( z + i) z log ( z + i)
Now, Lt = Lt .
z→∞ 2
z +1 z → ∞ z−i Nz+i Q
z log ( z + i)
= Lt Lt = 1.0 = 0
z→∞ z − i z→∞ z+i

Hence, Lt z
z → ∞ CR
z log ( z + i)
1 + z2
dz = 0

⇒ Lt z
z → ∞ CR
log ( z + i)
1 + z2
dz = 0

From (1), z ∞

−∞
log ( x + i)
1 + x2
FG
H
dx = π log 2 +

2
IJ
K

www.cgaspirants.com
www.cgaspirants.com

FUNCTION OF COMPLEX VARIABLE 125

Equating real parts, we get

z 1 log ( x 2 + 1)

−∞ 2 1 + x2
dx = π log 2

⇒ z
0
∞ log (1 + x 2 )
1 + x2
dx = π log 2.

ASSIGNMENT
Evaluate the following integrals using Contour integration:

z ∞ dx
z
∞ dx

m
1. (i) 2
(ii)
0 1+ x −∞ ( x + 1)3
2

z ∞ x2
z
∞ x2

o
(iii) 2 2
dx (G.B.T.U. 2012) (iv) dx ; a > 0
−∞ ( x + 1) −∞ ( x 2 + a2 ) 3

.c
2.
z ∞
2
dx
2 2 2
;a>b>0 3.
z
∞ x2
( x + 1)( x2 + 4)
2
dx (U.P.T.U. 2008)

ts
−∞ ( x + a )( x + b ) −∞

4.
z ∞ x2
dx 5.
z
∞ x2 − x + 2
dx
n
2 2 2
0 ( x + 9)( x + 4) −∞ x + 10 x2 + 9
4

z z x2
ra
∞ x ∞
6. (i) 2 2
dx (ii) dx
−∞ ( x + 1) ( x + 2 x + 2) −∞ ( x + 1) ( x 2 + 2 x + 2)
2 2

[U.P.T.U. (C.O.) 2009]


pi

7.
z ∞ x
dx 8.
z ∞ dx
as

2 2
−∞ ( x + 4 x + 13) −∞ x6 + 1

9. (i)
z ∞ cos mx
dx ; (m ≥ 0, a > 0) (ii)
z ∞ cos mx
dx ; m ≥ 0, a > 0
g

2 2
0 x +a 0 ( a2 + x 2 ) 2
.c

[U.P.T.U. (C.O.) 2008]

z ∞ x sin ax
z ∞ x sin πx
w

10. (i) dx ; a > 0 (ii) dx [U.P.T.U. (C.O.) 2009]


0 x +k2 2 −∞ x2 + 2 x + 5

z z
w

∞ cos x ∞ sin x
11. (i) 2 2 2 2
dx (a > b > 0) (ii) dx (a > b > 0)
0 (x + a ) (x + b ) 0 ( x2 + a2 ) ( x2 + b2 )
w

[U.P.T.U. (C.O.) 2008]

12.

14.
z−∞
∞ sin x
x2 + 4 x + 5
dx

If a > 0, prove that


13.
z

0
3
x sin x
( x + a2 )( x 2 + b2 )
2
dx ; (a > 0, b > 0)

(i)
z ∞

−∞
a cos x + x sin x
x +a 2 2
dx = 2πe–a (ii)
z

−∞
x cos x − a sin x
x2 + a2
dx = 0

eiz
[Hint: Consider f (z) = . At last multiply both Nr and Dr by x + ia and separate real and
z − ia
imaginary parts]

www.cgaspirants.com
www.cgaspirants.com

126 A TEXTBOOK OF ENGINEERING MATHEMATICS

15. Prove that


z ∞

0
cos 2 x
(1 + x ) 2 2
dx =
π
2
FG
e
3
1+ 2
H
IJ LMHint: Take f ( z ) = 1 + e
K MN (1 + z
2iz

2 2
)
so that f ( x ) =
1 + cos 2x
2 2
(1 + x )
OP
PQ
2
16. By integrating e − z round the rectangle whose vertices are 0, R, R + ia, ia, show that

(i)
z 0
∞ 2
e − x cos 2ax dx =
e− a
2
2

π and (ii) z
0
∞ 2
e − x sin 2ax dx = e − a
2

z0
a 2
e y dy .

17. Apply calculus of residues to prove that


z 0

sin x2 dx =
z ∞

0
cos x2 dx =
1
2
π
2
.

Answers

m
π π
1. (i) π/2 (ii) 3π/8 (iii) (iv)
2 8a3

o
π π 5π
2. 3. π/3 4. 5.
ab(a + b) 200 12

.c
π 7π π
6. (i) − (ii) 7. − 8. π/3
5 50 27

9. (i)
π −ma
2a
e (ii)
π e − am (am + 1)
n
4a 3 ts
10. (i)
π − ak
2
e (ii) – πe– 2π

Fe −b
e− a I
ra
π π
11. (i)
2(a 2 G
−b )H b 2

a JK (ii) 0 12. –
e
sin 2

π ( a 2e − a − b2e − b )
pi

13.
2 ( a 2 − b2 )
as

1.42.3. Integrals of the Type


f(x)
F(x) z ∞
dx , when F(x) has zeros on the real axis.
−∞
g

When the poles of f(z) lie on the real axis and also within the semi-circular region, then those
.c

which lie on the real axis can be avoided by drawing small semi-circles Cr , Cr′ etc. about those
poles as centres and small radii r and r′ in the upper half of the plane.
w

This method is said to be ‘indenting the semi-circular contour’.


When the semi-circle of radius R has been indented then f(z) is analytic along this

z
w

modified contour C and the integral f ( z) dz can be evaluated by Cauchy-Residue theorem.

z
C
w

sin mx ∞
Example. Evaluate dx , m > 0. [G.B.T.U. (C.O.) 2008, G.B.T.U. 2007]
x 0
Sol. Since sin mx is the imaginary part of eimx, we consider the function
e imz Y
. φ(z) =
z
This has a simple pole at z = 0, which lies on the C1
real axis. Enclose this singularity by a small semi-circle
C2 : | z | = r. Evaluate the function φ(z) over the contour
C2
C shown in the figure consisting of parts of the real axis
from –R to –r and r to R, the small semi-circle C2 and the
large semi-circle C1. Since the function has no singular- –R –r O r R X
ity within this contour, by Cauchy’s theorem, we have

www.cgaspirants.com
www.cgaspirants.com

FUNCTION OF COMPLEX VARIABLE 127

z C
φ( z) dz = 0

⇒ z −r

−R
φ( x) dx + zC2
φ( z) dz + z
r
R
φ( x) dx + z
C1
φ( z) dz = 0


−R x z
e imx −r
dx +
e imz
C2 z
dz +
r
R e imx

x zdx +
e imz
C1 z
dz = 0 z...(1)

Substituting –x for x in the first integral and combining it with the third integral, we
z
get

z
z z
r
R e imx − e − imx
x
dx +
C2
e imz
z
dz +
e imz
C1 z
dz = 0

z z z

m
R sin mx e imz e imz
or 2i dx + dz + dz = 0 ...(2)
r x C2 z C1 z

o
Now
z z z e imz
dz =
1
dz +
e imz − 1
dz ...(3)

.c
C2 z C2 z C2 z
On C2 , z = reiθ


z z z C2
1
z
dz =
0

π
n re iθ i dθ
re iθ
=−
π

0
idθ = – iπ ts
z z
ra

e imz − 1 | dz|
Also, dz ≤ M = πM
C2 z C2 | z|
pi

where M is the maximum value on C2 of | eimz – 1 | = | eimr (cos θ +i sin θ) – 1 |


Clearly, M → 0 as r → 0

z
as

e imz
∴ From (3), dz = − iπ
C2 z
g

Putting z = Reiθ in the integral over C1, we get ...(4)

z z z
.c

imz imR (cos θ + i sin θ)


e π e π
dz = iθ
Re iθ i dθ = i e imR cos θ . e–mR sin θ dθ
C1 z Re
w

0 0
Since, | eimR cos θ| ≤1

z z z
w

imz
e π π/2
∴ dz ≤ e − mR sin θ dθ = 2 e − mR sin θ dθ
C1 z 0 0
w

sin θ 2 π
Also, continually decreases from 1 to as θ increases from 0 to .
θ π 2
π sin θ 2 2θ
∴ For 0 ≤ θ ≤ , ≥ or sin θ ≥
2 θ π π

π
z e imz
z
dz ≤ 2 z 0
π/2
e −2 mR θ / π dθ = −
LM
N
π −2mRθ / π
mR
e
OP
Q
π/2

0
=
π
mR
(1 – e–mR)

As R → ∞, (1 – e–mR) → 0
mR


e imz
C1 z
dz = 0z

www.cgaspirants.com
www.cgaspirants.com

128 A TEXTBOOK OF ENGINEERING MATHEMATICS

Hence from (2), on taking the limit as r → 0 and R → ∞, we get


2i
∞ sin mx

x
dx − iπ = 0 z
z
0
∞ sin mx π
or dx = .
0 x 2

ASSIGNMENT
Apply calculus of residues to prove that:

1. (i) z ∞ xp − 1
1+x
dx =
π
sin π p
,0<p<1 (M.T.U. 2013) (ii) z ∞ xa − 1
1− x
dx = π cot a π, 0 < a < 1

m
0 0

(iii)
z ∞ x a −1
dx =
π
cosec
πa
,0<a<2
FG IJ
H K

o
2 2 2
0 1+ x

z z
.c
∞ cos x ∞ cos x π
2. (i) dx = 0 (ii) dx = sin a, (a > 0)
0 x −∞ a2 − x 2 a

3. (i)
z ∞ sin mx
2
x( x + a ) 2
dx =
2a
π
2
(1 – e–ma); a > 0 (ii)
z ∞

ts
sin πx
x(1 − x2 )
dx = π
n
0 0

z ∞ log x π
z ∞ (log x) 2 π2
ra
4. (i) 2 2
dx = − (ii) 2
dx =
0 (1 + x ) 4 0 1+x 8

z∞ xa FG IJ π πa
pi

5. (i) dx = (1 − a) sec ;–1<a<3


0 (1 + x 2 ) 2 H K 4 2

z xa F 2aπ IJ cosec (aπ); – 1 < a < 1


as

∞ 2π
(ii) dx = sin G
2
0 x − x +1 3 H 3K
z
g

∞ cos 2ax − cos 2bx


6. dx = π(b – a) if a ≥ b ≥ 0.
x2
.c

0
w

TEST YOUR KNOWLEDGE


w

1. Define analytic function and state the necessary and sufficient condition for function to be analytic.
(M.T.U. 2012)
w

2. If f(z) = u + iv is analytic, then show that the family of curves u(x, y) = c1 and v(x, y) = c2 are
mutually orthogonal. (M.T.U. 2012)
3. Using the Cauchy-Riemann equations, show that f(z) = | z |2 is not analytic at any point.
(M.T.U. 2013)
4. Find the constants a, b and c such that the function f(z) = – x + xy + y + i (ax + bxy + y2) is
2 2 2

analytic. (M.T.U. 2013)

5. Evaluate
z 1+i

0
z 2 dz. (U.P.T.U. 2008)

6.

7.
Evaluate the integral
z3 z
dz where C : | z | = 1.

Define isolated and non-isolated singular points.


C
e iz
(M.T.U. 2013)

(M.T.U. 2012)

www.cgaspirants.com
www.cgaspirants.com

FUNCTION OF COMPLEX VARIABLE 129

8. Define removable and essential singular points with example. (M.T.U. 2012)
9. Define singular point of an analytic function. Find nature and location of the singularity of
z − sin z
f(z) = (M.T.U. 2013)
z2
10. Find the values of a and b for which the function f(z) = cos x (cosh y + a sinh y) + i sin x
(cosh y + b sinh y) is analytic.
11. If f(z) = u + iv is an analytic function and u = x2 – y2 – y then find its conjugate harmonic function
v(x, y).
12. If f(z) = u + iv is an analytic function and v = y2 – x2 then find its conjugate harmonic function u(x, y).
x 2 − y2
13. If u = is the real part of analytic function f(z) = u + iv then find f(z) in terms of z.
( x 2 + y2 ) 2

m
14. Evaluate
z C
dz
z−2
around the circle | z – 2 | = 4.

o
15. Evaluate (5z 4 − z 3 + 2) dz around the unit circle | z | = 1.

.c
C

16. If F(α) =
z C
5z 2 − 4 z + 3
z−α
ts
dz which C is the ellipse 16x2 + 9y2 = 144, then find F(2).

z
n
dz
17. Evaluate where C is | z – 3i | = 4.
C z2 + 9
ra

F z + 1I 3
18. Find residue of f(z) = GH z − 1 JK at z = 1.
pi

2z + 1
19. Find residue of f(z) = at the pole z = –1. (M.T.U. 2014)
z2 − z − 2
as

z2
20. (i) Find residue of f(z) = at the pole z = –1.
( z + 3 z + 2) 2
2
g
.c

z2
(ii) Find residue of f(z) = at the pole –1. (U.P.T.U. 2014)
z2 + 3 z + 2
w

z 4 − 3z
w

21. Evaluate dz where C is any simple closed path such that 1 ∈ C, 0 ∉ C.


C z2 − z
z − sin z
w

22. Find the nature of singularity of f(z) = at z = 0.


z3

23.

24.
Evaluate
z z−3
z 2 + 2z + 5
C
dz when C ≡ | z | = 1.

Let u(x, y) = 2x(1 – y) for all real x and y. Find a function v(x, y) so that f(z) = u + iv is analytic.

25. Let I =
z f (z)
C ( z − 1) ( z − 2)
dz where f(z) = sin

anti-clockwise. Find the value of I.


πz
2
+ cos
πz
2
and C is the curve | z | = 3 oriented


26. Let ∑b
n=−∞
n z n be the Laurent’s series expansion of the function
1
z sinh z
, 0 < | z | < π, then find

b–2, b0 and b2.

www.cgaspirants.com
www.cgaspirants.com

130 A TEXTBOOK OF ENGINEERING MATHEMATICS

27.

28.
Let f(z) =
15

∑z
n=0
n
for z ∈ . If : | z – i | = 2, then evaluate
z
( z − i )15
C
dz.
f (z)

Let u(x, y) be the real part of an entire function f(z) = u(x, y) + i v(x, y) for z = x + iy ∈ C. If C is the

positively oriented boundary of a rectangular region R in R2 then evaluate


z FGH
C
∂u
∂y
dx −
∂u
∂x
IJ
dy .
K
eiz
29. Consider the function f(z) = . Find the residue of f at the isolated singular point in the
z( z2 + 1)
upper half plane {z = x + iy ∈ : y > 0}.

30. Let S be the positively oriented circle given by | z – 3i | = 2. Then evaluate


z dz
.

m
2
S z +4

31. Let f(z) be an analytic function. Then evaluate


z 2π
f ( eit ) cos (t) dt.

o
0

.c
1 1
32. Let f(z) = then find the coefficient of in the Laurent’s series expansion of f(z)
2
z − 3z + 2 z3

33.

34.
for | z | > 2.

conjugate harmonic function v(x, y).


Define Harmonic function.
n ts
If u(x, y) = x3y – xy3 is the real part of analytic function f(z) = u(x, y) + i v(x, y), then find its
(M.T.U. 2014)
ra

Answers
pi

1 1 2 2
4. a=− , b = − 2, c = 5. − + i 6. –πi
2 2 3 3
9. removable singularity at z = 0 10. a = –1, b = –1 11. 2xy + x + c
as

1
12. 2xy + c 13. +c 14. 2πi 15. 0
z2
g

16. 30πi 17. π/3 18. 6 19. 1/3


.c

20. (i) – 4 (ii) 1 21. 2πi 22. removable singularity


23. 0 24. x2 – (y – 1)2 25. – 4πi
w

26. b–2 = 1, b0 = –1/6, b2 = 7/360 27. 2πi (1 + 15i) 28. 0


1 π
w

29. − 30. 31. πf ′(0) 32. 3


2e 2
33. x4 + y4 – 6x2y2 + c.
w

www.cgaspirants.com

You might also like